You are on page 1of 96

Surgery Clerkship

Questions
1. What is the MCC of achalasia?
a. Majority of cases are idiopathic but the 2nd MCC is adenocarcinoma of the esophagus & after that it is
Chaga’s dz
2. What is a long term management in these pts?
a. Long term esophagoscopy surveillance since 10% of pts w/ achalasia develop SCC of the esophagus that
goes unnoticed since they already suffer from dysphagia.
3. “Worst HA of my life”
a. Subarachnoid hemorrhage , if it also presents w/ refractory hypotension think pituitary apoplexy &
administer steroids
b. Pts may also c/o neck stiffness & photophobia, may be 2/2 to rupture of a berry aneurysm
4. Desmoid tumor, osteoma & colon cancer
a. Gardner’s syndrome
5. What is used for confirming the dx of hemorrhagic pancreatitis?
a. CT scan w/ IV contrast
6. Liver abscess w/ “anchovy paste”
a. Entamoeba histolytica which also causes flask shaped GI ulcers & is tx w/ metronidazole
7. Pancreatic mass, gallstones, diabetes & steatorrhea/diarrhea….
a. Somatostatinoma which is a tumor of the delta cells of the endocrine pancreas which produce
somatostatin. Abnormally elevated somatostatin can cause
i. DM by inhibiting insulin secretion
ii. Steatorrhea by inhibiting CCK & secretin release
iii. Gall stones by inhibiting CCK release therefore no gallbladder contraction thereby formation of
gallstones
8. Excruciating pain w/ bowel movement is suggestive of….
a. Anal fissure; dx based on PE & initial tx are stool softeners & NO to relax the sphincter
9. What lab tests are used in evaluation for presence of H pylori?
a. Histologic evaluation of bx is gold standard
b. Urease detection via urea breath test, also used to monitor response to abx tx
10. What is a major SE of ERCP?
a. Pancreatitis in 10% of pts!!!
11. How is this diagnosis confirmed?
a. Spiral CT scan
12. On the 5th post op day after a laparotomy the wound is draining clear pink fluid. The medical student
removes the dressing & asks the pt to sit up. When the pt complies the wound opens widely & small
bowel falls out. What is the dx & how would you manage it?
a. This is bowel evisceration (the complication of dehiscence) & this is a medical emergency where you
cover the bowel w/ warm & moist dressing & prep the pt for surgery.
b. The saline needs to be warm to prevent hypothermia
13. A pt who suffered multiple traumas after a head on collision requires vascular surgery, neurosurgery &
orthopedic surgery for an open femoral fracture. They are trying to decide who goes first….what is true
regarding timing of the femoral fx?
a. This must be treated & closed within 6 hours otherwise pt will have lifelong osteomyelitis &
complications
b. Also remember that any OPEN wound that presents after >6 hrs should NOT be closed
14. Damage to what nerve may occur in a pt who is having resection of a tumor of the submandibular
salivary gland
a. Damage to the hypoglossal nerve (CN12) which would result in tongue palsy
b. Tongue will point TOWARD the lesion whereas the Uvula points UUUWAY from the lesion (CN9)
15. A newborn has not passed their first stool. On a KUB you see that there is feces obstructing the left colon.
Pt begins to have bilious vomiting. What is the likely dx?
a. Meconium ileus
16. What is the treatment for a pt w/ a confirmed PE?
a. Place the pt on heparin or any other anti-coagulants which will prevent formation of new clots.
b. Anti-coagulants will not bust an existing clot but it does not matter in the case of PE since the lung has the
most macrophages & other cells that will degrade the clot quickly.
17. What if the previous pt developed a second PE while on appropriate anti-coagulants?
a. Place a vena cava filter
18. College football player gets tackled & later c/o knee pain. His knee is swollen & there is pain on direct
palpation on the medial aspect of the knee. w/ the knee flexed at 30 degrees passive abduction elicits
pain in the same area & the knee can be abducted further out than the contralateral knee. What is going
on?
a. All of this suggests tearing of the medial (tibial) collateral ligament which is further supported by
increased abduction of the knee.
b. A swollen knee is a poor man’s MRI; if you see a swollen knee it suggests pathology of the knee.
c. If you are tackled on the lateral aspect of the knee then the force is transferred to the medial ligament
which will tear.
19. When placing a pt on a ventilator what FiO2 should you use & how should you proceed?
a. Start w/ FiO2 of 100% & quickly titrate down & use the lowest possible FIO2 to maintain a PaO2 >50 & a
SaO2 >90% to avoid oxygen toxicity
b. FIO2 <60% is considered safe
20. How can we decrease FiO2 but still increase PaO2?
a. Add PEEP or CPAP which allow to reduce FIO2 & still support PaO2
21. What is the inspiratory/expiratory ratio used on ventilators?
a. 1:2 meaning 1 sec of inspiration for every 2 seconds during expiration
b. If 12 breaths are set per minute then 5 seconds per breath then its like 1.8 seconds of inspiration & 3.2
seconds of expiration
22. What are the SE associated w/ PEEP?
a. Barotrauma like pneumothorax due to the increase in positive pressure
b. Increase pulmonary vascular resistance & decreased venous return to heart
23. What is the MCC of cardiac neoplasms?
a. Metastases from other tissue such as lung, breast, lymphoma…. Account for 75% of cardiac neoplasms
24. What is the most common primary cardiac neoplasm?
a. Atrial myxomas are the most common primary cardiac neoplasm which are benign growth that usually
arise in the interatrial septum around the fossa ovalis.
b. Tx is surgical excision
25. T/F the presence of an ET tube prevents aspiration from occurring
a. False!!
26. When are tracheostomy performed & why?
a. Tracheostomy is performed when a pt is going to be ventilator dependent > 2 weeks. These help prevent
tracheomalacia & nosocomial infections
b. Also done for pts who need an airway and have maxofacial injuries
27. What pressure is considered pulmonary HTN?
a. A mean pulmonary arterial pressure > 25mmHg
28. T/F almost all ICU pts are placed on H2 antagonist or PPI
a. True!!! This is because we want to prevent the development of stress ulcers which are common in ICU
pts who suffer many repeated stressors. Stress ulcers are difficult to manage & present as copious
vomiting of blood
b. Stress ulcers are superficial localized gastric/duodenal ulcers w/ surrounding erythema 2/2 to ↓ splanchnic
flow
29. Pt presents w/ new onset of severe stabbing pain in the anterior & back of the chest. When you ask her
to point where it hurts on the back she points between her scapula. The pt is diaphoretic & is
hypertensive. A CXR shows a widened mediastinum. What do you suspect & how would you proceed in
confirming the dx in the pt?
a. Severe tearing/ripping/stabbing anterior & posterior chest pain presenting in hypertensive, diaphoretic
pt w/ a widened mediastinum is suggestive of aortic dissection. The next step is either a TEE which can
be done bedside or a CT scan to confirm the dx of acute aortic dissection.
30. What are the two types of aortic dissections?
a. Stanford type A is a dissection in the proximal ascending aorta which more commonly causes anterior
chest pain
b. Stanford type B is a dissection in the distal aorta that commonly causes interscapular back pain.
31. What is the tx for aortic dissection?
a. Initiate medical tx immediately w/ IV beta blockers to lower HR & diminish LV ejection force &
nitroprusside to lower systolic BP < 120
b. For type A dissections surgical intervention is required
c. For type B medical management w/ the BB and nitroprusside only
32. Medullary thyroid cancer, pheochromocytoma & hyperparathyroidism
a. MEN-2a Medullary
thyroid

Hyper-PTH Hyper-PTH

pheo pheo

33. Hypokalemia refractory to IV potassium supplementation


a. Hypo-Mg
34. Premature newborn w/ pneumatosis on KUB
a. Necrotizing enterocolitis; only operate if there is a perf
35. Child w/ abdominal mass that crosses midline
a. Neuroblastoma curses midline & peak incidence around 2-3
b. Wilm’s tumor is <5 yo & the mass stays in one side & is associated w/ hematuria & sometimes HTN
36. A window cleaner falls three stories directly on his feet. Radiographs show bilateral commuted fx of
both calcaneus. What is the next step in management?
a. Radiograph of the vertebrae checking for compression fx of the thoracic & lumbar spine!!!!
37. While trying to intubate an awake pt who suffered GSWs the pt vomits & aspirates particulate matter.
What is the next step once this has happened?
a. Use bronchoscopy for lavage & removal of particulate matter which will be followed by bronchodilators
& respiratory support
i. In the past steroids have been used but now it is contraindicated for aspiration
b. We avoid this in elective surgery by asking pts to be NPO
38. In the 2 post-operative days after an abdominal procedure a pt is afebrile but develops a fever of 102F
on day 3. What is the most likely cause of the fever?
a. Since the first 2 days the pt is afebrile then it is less likely to be due to atelectasis & more likely to be due
to UTI.
b. UA & urine cultures are the best tests
39. In the first 4 post-operative days after an abdominal procedure a pt is afebrile but develops a fever of
102F on day 5. What is the most likely cause of the fever?
a. A fever this far out post-surgery is most likely to be due to DVT.
40. How will you confirm the dx?
a. Doppler studies of the deep veins
b. Physical exam is worthless!!!
41. In the first 6 post-operative days after an abdominal procedure a pt is afebrile but develops a fever of
102F on day 7. What is the most likely cause of the fever?
a. This late onset of fever post operation is most likely due to infection of the wound itself. PE should show
erythema & possible pus.
42. Pt who had major abdominal surgery had a normal post-operative course. 10 days post op the pt begins
to spike recurrent fevers of 102 & 103. What is the most likely dx?
a. Abscesses likely subphrenic, pelvic or other places.
43. What is the mnemonic used to remember the etiology of fever regarding the timeline post-surgery?
a. The 5 Ws: Wind (Atelectasis), Water (UTI), Walking (DVT), Wound, Wonder-where (abscess)
44. A man w/ advanced pulmonary TB is undergoing laparotomy for perforation of terminal ileum
secondary to intestinal TB. Half way through the case it becomes progressively difficult to bag him. The
BP steadily declines while CVP steadily rises; there is no evidence of abdominal bleeding. What is the
management for this pt?
a. The pt clearly has damaged lungs 2/2 to long standing pulmonary TB. Since the pt is undergoing an
operation he is likely intubated & given positive pressure which may likely cause a tension
pneumothorax therefore the immediate solution is a lifesaving hole through the diaphragm to relieve the
pressure. The reason you won’t do a needle in the 2nd intercostal space is because the pt is draped & this
is occurring during surgery.
45. Currant jelly stools w/ abdominal colic in a toddler
a. Intussusception, usually preceeded by viral URI.
b. Dx and tx done w/ air enema
46. hearing loss, tinnitus & vertigo
a. Menniere’s dz
47. Adolescent boy w/ nasal obstruction & recurrent epistaxis
a. Juvenile nasopharyngeal angiofibroma
48. 4 yo child sitting upright in tripod position, drooling & has a “hot-potato” voice
a. Epiglottitis…..must intubate but do so in an OR setting w/ a surgeon present. Avoid irritating the airway
49. Angina, syncope & CHF
a. Aortic stenosis; surgical replacement of valve once P gradient . 50mmHg or symptomatic
b. Unlike the aortic valve, the mitral valve is repaired rather than replaced
50. Supracondylar fx & contracture of forearm flexors
a. Volkmann’s contracture is a permanent flexion contracture of hand w/ blue, cold hands which lack a
radial pulse. This is caused 2/2 obstruction of brachial artery
51. Tibia fx, “pain out of proportion”, pain on passive foot movement & palpable pulses
a. Compartment syndrome….hallmark is pain on passive foot movement
52. 25 yo male w/ liver mass w/ fibrous septae & NO hx of cirrhosis or hepatitis
a. Fibrolamellar HCC
53. EKG w/ flattening of T waves & U waves
a. Hypokalemia
b. Peaked T waves is suggestive of hyperkalemia
54. Central pontine myelinosis presents as…
a. Presents as acute paralysis, dysarthria & dysphagia in a pt w/ chronic hyponatremia. Do not correct at a
rate faster than 8-10 mmol/L per day
55. Polydipsia, polyuria & constipation
a. Hypercalcemia
56. Factor 8 deficiency
a. Hemophilia A, administer synthetic F8 or DDAVP which will stimulate vWF & F8 release
57. Abdominal pain, fever, hypotension, HYPERkalemia & HYPOnatremia…
a. Adrenal insufficiency (Addisonian crisis; aldosterone def)…remember that aldosterone acts on the
principal cells to ↑ reabsorption of Na thereby creating a negative lumen which forces secretion of K. w/
aldosterone deficiency there is no reabsorption of the Na & thereby accumulation of the K
b. May also be AI 2/2 TB infxn of the adrenal glands
58. Massive UOP & HYPERnatremia….
a. ↓ ADH secretion or resistance = diabetes insipidus
59. 18 hrs after major surgery a pt becomes disoriented. What is the first thing you want to check?
a. Blood gases to check for pulmonary insufficiency (likely atelectasis) & hypoxia
60. What does acute coronary syndrome refer to?
a. Either unstable angina or an MI since it is the manifestations of a plaque rupture & coronary occlusion
61. How can you distinguish unstable angina from an MI?
a. Only an MI results in elevation of cardiac enzymes
62. What is the difference in the pathophysiology of stable vs. unstable angina?
a. With stable angina there is a fixed atherosclerotic plaque that when patients exert themselves result in
increased oxygen demand which is NOT met by increased perfusion
b. With unstable angina the oxygen demand is unchanged (at rest) but the supply is decreased
secondary to reduced resting coronary flow
i. Unstable angina results in angina while resting
ii. Unstable angina is significant since it indicates that stenosis has enlarged via thrombosis, rupture of
the plaque or hemorrhage which may lead to total occlusion & an MI
63. What is the presentation of a pt w/ unstable angina?
a. Pts w/ chronic resting angina that increases in frequency, duration & intensity
b. Pts w/ angina at rest
c. Pts w/ new-onset of angina that is severe & worsening
64. T/F thrombolytic therapy has NOT been shown to be beneficial in tx of unstable angina
a. True!
65. What is the management of pts presenting w/ unstable angina?
a. Hospital admission & cardiac monitoring
b. Aspirin
c. Beta blockers are first line since they reduce myocardial oxygen demand by reducing HR, BP &
contractility
d. Nitrates which relieve sx by generalized vasodilation thereby reducing preload & myocardial oxygen
demand
e. Low-molecular weight heparin like enoxaparin or unfractionated heparin for at least 2 days via IV
f. Glycoprotein IIb/IIIa inhibitors (Abciximab, tirofiban) especially if pt is undergoing PTCA or stenting
66. What drug has been shown to reduce the risk of death & occurrence of CVA by 25% if given as long-term
management after an MI or unstable angina tx?
a. Statins were shown by the CARE trial to reduce the risk of death & stroke & need for revascularization
67. What other coronary pathology, on top of MI & unstable angina, may result in angina at rest?
a. Varian/Prinzmetal’s angina is transient coronary vasospasms that are accompanied by a fixed
atherosclerotic plaque resulting in resting angina & ventricular dysrhythmias
68. What are the ECG findings associated w/ Prinzmetal’s angina?
a. The hallmark is transient ST segment ELEVATION suggesting transmural ischemia
69. What is the definitive test for dx of Prinzmetal’s angina?
a. Coronary angiography displaying coronary vasospasm when the pt is given IV ergonovine which
provokes chest pain
70. What is the treatment for Prinzmetal’s angina?
a. CCB & nitrates which both act as vasodilators have been shown to be helpful in tx of the vasospasms
71. Increased urine osmolality & hyponatremia w/ low serum osmolality
a. SIADH
72. IV abx, fever & profuse watery diarrhea
a. Clostrdium difficile pseudomembranous colitis; rarely bloody diarrhea
b. Associated w/ cephalosporins, clindamycin & amoxicillin
c. Tx w/ oral vancomycin or metro
73. Bleeding gums & wound dehiscence
a. Vitamin C deficiency
74. Fever, central line & hyperglycemia
a. Central line infection  sepsis/bacteremia
b. MCC of nosocomial bacteremia = coagulase neg staph like staph epidermidis or saprophyticus
75. On the 7th post-operative day after inguinal hernia repair a pt develops fever. The inguinal incision is
erythematous & warm. There is no pus draining. What is the tx?
a. A fever 7 days post op in a pt w/ an infected looking incision is suggestive of infection. If there is no pus
draining then administer abx that cover skin flora.
b. If there is an abscess present you must tx w/ an I&D & provide appropriate abx
76. How can you confirm the presence of an abscess?
a. If you are not sure whether or not this is an abscess get an US or CT w/ contrast
77. Within 8 hrs after birth a newborn is found to have excessive salivation. What is the next step?
a. Excessive salivation in a newborn equals esophageal atresia therefore place a soft NG tube until it meets
resistance & a radiograph will be done showing that the NG tube is blocked.
b. Baby-gram will likely show air in the GI tract which is due to a tracheoesophageal fistula w/ esophageal
atresia.
78. What is the next step once the esophageal atresia is confirmed?
a. Must rule out other congenital anomalies since this is commonly part of VACTER = vertebral anomalies,
anal anomalies, cardiac anomalies, tracheo-esophageal fistulas & renal anomalies
b. If the other anomalies are present place a GJ tube
79. A 2 month old baby boy is brought in for chronic constipation. The kid has abdominal distention & KUB
shows gas & dilated bowels throughout the GI tract but w/ large distention of the distal colon. Rectal
exam is followed by explosive expulsion of stool w/ remarkable improvement of the abdominal
distention. What are you worried about in a baby w/ chronic constipation?
a. Aganglionic megacolon also known as Hirschprung’s dz which presents after several weeks to months
post birth. The center piece for this presentation is a baby or child w/ chronic constipation.
80. What is the first step in confirming the dx?
a. Full thickness bx of the rectal mucosa which will show the absence of neurons
81. Appendectomy followed by fever & abdominal pain on post-op day 8.
a. This is most likely due peritoneal abscess
b. Fever after 1 day = atelectasis
c. 2-3 days = UTI
d. 5 days = DVT
e. 7 days = wound infection
f. >7 days = abscess
82. Advancing crepitus, fever & blood blister
a. Necrotizing fasciitis, tx w/ surgical debridement, IV penicillin and hyperbaric chamber
83. High INTRA-operative fever
a. malignant hyperthermia tx w/ oxygen, dantrolene and discontinue surgery/succinylcholine
84. high fever 45 mins post-surgery
a. sepsis…get blood cx & then start empiric abx
85. Otorrhea & battle’s sign after MVA
a. Basilar skull fx, get CT & AVOID prophylactic abx
86. ulcer & decreased pain w/ food
a. duodenal ulcer
87. vomiting, retching & epigastric pain
a. Mallory-Weiss tear which is a tear of the submucosal ARTERY near the GE junction
88. Confusion, ataxia & Ophthalmoplegia
a. Wernicke’s encephalopathy tx w/ thiamine FIRST & later glucose
89. A 24 yo female c/o numbness & tingling in her hands, particularly at night. She is a secretary & admits to
typing a lot at work. When asked to hand her hands limply in front of her numbness & tingling are
reproduced over the distribution of the median nerve. What is the next step in the management?
a. This is classic carpal tunnel syndrome which is diagnosed w/ wrist radiographs, including carpal tunnel
views.
b. Seen in young women w/ repetitive hand work w/ distribution over the median nerve
90. How is this treated?
a. First line treatment w/ splints & anti-inflammatories
91. The same pt present s several months later saying that the tx has not helped so you decide to correct it
w/ surgery. What is done before surgery?
a. Electromyography prior to carpal tunnel surgery
92. Fever on post-op day #1, w/ “bronze” weeping, tender wound
a. Clostridial wound infection ; get tissue for Gram staining, IV penicillin & possible hyperbaric chamber
93. 18 yo woman w/ bloody nipple discharge
a. Intra-ductal papilloma; no masses on mammogram therefore dx w. galactogram or retro-areolar
resection
94. Irritability, diaphoresis, weakness, tremulousness, & palpitations
a. Insulinoma….hypoglycemia w/ ↑↑↑ C-peptide levels
95. Hematochezia & tenesmus
a. Rectal cancer….unlike other CRC, rectal cancer requires resection, chemotherapy & radiation
b. If it is carcinoma in situ there is no need for radiation or chemo
96. Upper GI bleed, jaundice & RUQ pain
a. Hemobilia which implies bleeding into the biliary tree which occurs when there is a fistula between a
splanchnic vessel & the biliary system
97. A 28 yo AA female presents w/ a 4 wk h/o a dry cough & DOE. CXR shows a pulmonary granuloma & PPD
is 6 mm. She also c/o painful nodules on the anterior portion of her shins. What is the likely dx?
a. Sarcoidosis is a chronic systemic granulomatous dz characterized by non-caseating granulomas which
most commonly presents in young (<40) AA females. 25% of pts also have erythema nodosum which
is an inflammation of the fat cells under the skin (panniculitis) presents as painful nodules over the
shins.
b. Definitive dx requires transbronchial bx showing noncaseating granulomas
98. What CXR is a hallmark of this dz?
a. Bilateral hilar adenopathy is hallmark of this dz but is not specific & is seen in 50% of cases.
99. What is the most common eye involvement seen in pts w/ Sarcoidosis?
a. Anterior uveitis which may result in significant visual impairment
100. What serum marker is unique to Sarcoidosis?
a. Elevated angiotensin-converting enzyme (ACE) is seen in 80% of pts which supports the dx of
Sarcoidosis however other pulmonary dz may cause elevation of this enzyme.
101. What is the tx for Sarcoidosis?
a. Most cases resolve spontaneously in 2 years & do not require tx
b. Systemic corticosteroids is the tx of choice & indications for use is symptoms of lung dz or PFT
deterioration
102. What is the MCC of death due to Sarcoidosis?
a. Cardiac dz is the MCC of death in these pts although it is not a common finding
103. What are the two classic descriptors of an acute MI?
a. Substernal chest pain persisting for longer than 30 minutes & diaphoresis
b. The pain is often described as “crushing” or “elephant standing on my chest”
c. Other sx are “sense of impending doom”, weakness, fatigue & N/V
104. Where does the chest pain radiate w/ an MI?
a. The pain radiates to the LEFT neck, shoulder & arm
105. How does a right ventricular infarct present?
a. Hypotension
b. Elevated jugular venous pressure
c. Hepatomegaly
d. CLEAR LUNGS
e. Inferior ECG changes
f. Essentially it presents as right CHF w/ clear lungs
106. What is the diagnostic gold standard for MI?
a. Elevated cardiac enzymes, most specifically troponin I & T & less so is CK MB
107. When does each of the cardiac enzymes increase, peak, return to normal & how often should you
check them?
Increase Peak Returns to normal Monitor every
CK-MB 4 hours 24 hours 48 to 72 hours 8 hours
Troponin 3 hours 24 hours 5 – 14 DAYS 8 hours
108. What are the ECG markers associated w/ an MI?
a. Peaked T waves occur very early & are usually not seen by the time the pt is in the hospital
b. ST segment elevation which suggests transmural ischemia
c. Inverted T waves which are sensitive but not specific
d. Pathologic Q waves suggest necrosis & are not present until late & may indicate a previous MI
e. ST segment depression is seen in subendocardial ischemia/infarct
109. What is the treatment for an acute MI?
a. MONA = morphine, oxygen, nitrates & aspirin plus
b. tPA (alteplase) or PTCA depending on availability
110. What trial has shown that PTCA is superior to tPA in tx of a MI?
a. PAMI trial
111. When is tPA indicated for tx of MI?
a. Administer as soon as possible up to 24 hours but best within 6 hours of onset of angina
b. If two ECG leads showing ST segment elevation & pt is refractory to nitroglycerin
112. What are some contraindications to alteplase?
a. Uncontrolled HTN >180/110; in this case you must first control he HTN
b. Trauma resulting in hemorrhage
c. Active PUD
d. Recent invasive procedure or surgery that may result in bleeding
e. Dissecting aortic aneurysm
113. What beta blocker is most effective in long term management of pts w/ post-MI LV dysfunction?
a. Carvedilol was shown by the CAPRICORN trial to reduce the risk of death in pts w/ post-MI LV
dysfunction
114. What is the treatment for ventricular tachycardia in a hemodynamically stable patient?
a. IV Amiodarone is used for stable pts w/ VT
b. Cardioversion is used in unstable pts
115. What is the treatment for Vfib?
a. Unsynchronized cardioversion & CPR
116. What is the mortality rate in MI?
a. 30% mortality rate where half the deaths are pre-hospital
117. T/F some MIs may be asx
a. True! More common in post-op pts, elderly, women & diabetics
118. Pt c/o DOE & fatigue. PE reveals a sternum that is subtly lifted & a loud S2. What do you suspect?
a. These are the signs of pulmonary HTN which is a devastating disease. The sternum is lifted due to RV
hypertrophy & the loud S2 is specifically due to a loud P2.
b. Once the dz progresses there will be RV heart failure & the corresponding JVD, HSM, ascites & edema
119. 9 days after a sigmoid resection for cancer the wound drains a brown fluid that everyone
recognizes as feces. The pt is afebrile & otherwise doing well. How do you manage this pt?
a. If we have luminal contents coming out the abdominal wall we know this is a fistula. We know this is not
really just a perforation or a leak since the pt does not have signs of peritonitis or fever. Since the fistula
is in the sigmoid colon most absorption of nutrition & water has already been done therefore this does
not pose a problem for nutrition or hydration therefore this is more of a cosmetic & convenience
problem. Most fistulas heal by themselves therefore leave it as is & it will get better by itself.
120. An 18 yo woman has a firm rubbery mass in her left breast that moves easily w/ palpation. What
is the next step in management?
a. In this age group a firm rubbery mass that is moveable is ALWAYS fibroadenoma which may either be
confirmed w/ an FNA or a sonogram…
b. Remember that we do not perform mammography in women under the age of 20 & although an FNA does
not get a lot of tissue we know that since the pt is young the likelihood of breast cancer is very low.
121. A 76 yo man has been known for years to have a wide-pulse pressure & a glowing high pitched
diastolic murmur best heard at the 2nd intercostal space & along the sternal border. Pt has had serial
echocardiograms & in the most recent one there is evidence of left ventricular dilatation. What is the
likely dx?
a. Aortic insufficiency is heard at the base as a diastolic murmur w/ wide pulse pressure
122. Several months after a crushing injury to his arm a pt c/o constant burning agonizing pain that
does not respond to the usual analgesics. Pain is aggravated by the slightest stimulation of the area. The
arm is cold, cyanotic & moist. What is the dx & tx?
a. This is reflex sympathetic dystrophy which is seen post crush injury resulting in burning pain in a cold
cyanotic arm
b. Tx is a sympathetic block which should relieve the sx
123. A 26 yo drug addict develops CHF over a short period of a few days. He has a loud diastolic
murmur on the right second intercostal space. PE done a few weeks ago was completely normal. What is
going on?
a. This is an example of acute aortic insufficiency due to acute endocarditis seen in drug addicts likely due
to staph aureus
b. Although right sided endocarditis is more common in drug users, the right sided acute endocarditis are not
as severe & dangerous
124. Explain why hypertension & hypotension may result in MI
a. Hypertension results in increased afterload which will lead to increased cardiac work  ↑ myocardial
oxygen demand
b. Hypotension results in ↓ coronary perfusion….both morphine & nitrates may result in hypotension &
decreased perfusion
125. How do premature ventricular contractions (PVC) affect cardiac function?
a. PVCs result in lower stroke volume since there is a decrease in diastole  ↓ filling as well as decreased
coronary artery filing time also due to shortening of diastole
b. High frequency of PVCs may predict VFib or VTach
126. How does the QRS due to PVCs differ from a normal QRS?
a. PVCs cause a wide QRS because the conduction is through ventricular muscle rather than normal
pathways therefore it is slower & wider.
127. What is the tx for PVCs?
a. PVCs appear in 50% of men who undergo Holter monitoring & most pts are asx but if the pt is
symptomatic then BB may be used
128. When are anti-arrhythmics contraindicated in tx of PVCs?
a. Post-MI since the CAST trials showed that the use of antiarrhythmic drugs post MI increase risk of death
129. You are called to the nursery to see a newborn boy who has not urinated in the first 2 days of life.
On PE you notice a distended suprapubic mass. On PE his meatus appears normal. What is the next step?
a. This is an example of urinary obstruction in a newborn which may be either to meatal stenosis or
posterior urethral valve. Since the meatus is normal on examination the most likely dx is the PUV. The
next step should be insertion of a Foley to void the bladder (PUV is a one way valve that will allow
catheter placement) later perform a retrograde & voiding cystogram.
130. Minor trivial trauma in a young child found to have microhematuria. Next step is?
a. Urological workup since small trauma should not cause microhematuria
131. Sudden onset of testicular pain in 15 yo w/ no evidence of tenderness in the cord is managed by….
a. Emergency surgery (no other studies) since this is testicular torsion. Perform orthoplexy on other side
132. What if the previous pt was 20 w/ fever & pyuria w/ sudden onset of testicular pain, swollen
testicle & tender cord?
a. If pt has fever & a tender cord it is likely acute epididymitis 2/2 STI; r/o testicular torsion w/ sonogram
133. In the second week in a stormy complicated post-operative period in a 30 yo male that suffered
multiple GSWs to the abdomen he becomes progressively disoriented. CXR shows pulmonary infiltrates
& a low PO2 while on 40% (maximum) oxygen. He has no evidence of CHF since PCWP is 14 mmHg. What
is the likely dx?
a. Adult RDS which is commonly seen in complicated long post-operative pts presenting w/ sudden
disoriented w/ pulmonary infiltrates w/ no evidence of CHF & low PO2.
134. What is a common comorbidity in pts who develop ARDS?
a. Sepsis likely due to a hidden abscess that may be found w/ CT
135. What is the tx for these pts?
a. Respiratory support & PEEP which will help push the oxygen across membranes.
b. Watch out since PEEP can also damage the lung therefore allow some degree of hypercapnea to ensure we
are not too aggressive
136. A 47 yo alcoholic man presents w/ ill-defined upper abdominal discomfort & early satiety. On PE
he has an ill-defined deep epigastric mass. He was d/c from the hospital 4 wks ago for tx of acute
pancreatitis. What is the first step in management & the likely dx?
a. This is a classic presentation of pancreatic pseudocyst where a trauma or inflammation of the pancreas
results in leaking of the pancreatic juice forming a pseudocyst. Confirm the dx w/ either a sonogram or a
CT scan.
137. What is the most common site of pancreatic pseudocysts?
a. Lesser sac
138. What is the management in a pt diagnosed w/ a pancreatic pseudocyst?
a. The general rule is that if it is diagnosed before 6 weeks we leave it alone since the majority heal by
themselves
b. The bad complications such as rupture, bleeding, occur after 6 weeks therefore repeat the US or CT scans
to monitor the cyst for shrinking or growth. Therefore if it is not shrinking by 6 weeks then perform an
endoscopic internal drainage of the cyst (create a cysto-jejunostomy).
139. A 45 yo female who has been a T6 paraplegic for 10 years is held in a meeting for several hours
beyond the time where he would have normally done his in & out self-cauterization of his urinary
bladder. He develops a pounding HA, profuse perspiration & bradycardia w/ a BP of 220/120. What is
going on & how would you manage this pt?
a. This is a classic example of autonomic dysreflexia triggered by the distended bladder (could be any
other visceral stimulation). Pt requires immediate alpha adrenergic antagonist + relief of the bladder;
this only occurs in pts w/ high spinal cord interruptions (T6 & above)
b. Pt may benefit w/ long term CCB
140. 65 yo c/o severe sharp electric pain in his right side of face. Pt did not shave around the area of
pain. No skin changes. Dx?
a. Trigeminal neuralgia tx w/ anticonvulsants or radioablation of pain fibers
141. Long term tx for VUR in a young child who just had a UTI is…
a. Long term abx until the child grows out of the problems
142. A 57 yo alcoholic man was tx for hemorrhagic pancreatitis in the ICU & was transferred to the
floor. 2 wks after the onset of dz the pt begins to spike fevers & abdominal discomfort. What is the likely
dx?
a. This is likely formation of an abscess which will need to be diagnosed w/ a CT & tx w/ percutaneous
drainage
143. What heart valve & defect is seen in pts w/ h/o rheumatic fever?
a. Mitral STENOSIS which is heard during diastole
b. Mitral valve prolapse/regurgitation is heard as an apical holosystolic murmur
144. What is the best surgical tx for mitral stenosis?
a. Unlike tx for aortic valve defects which tx is valve replacement, repair of the mitral valve is preferred
over replacement. The most favored tx is mitral commisurotomy which will open up the stenotic valve.
If repair is not possible then replacement is done.
145. A 53 yo female describes that she wakes up at night w/ her right middle finger acutely flexed &
unable to extend it. She is only able to extend it by using her other hand where she feels an acute painful
snap. What is the dx & first line tx?
a. This is classic trigger finger commonly seen in females in this age group
b. First line tx for trigger finger is steroid injections
146. A 64 yo pt in the CCU post MI develops second degree AV block due to ischemia of the conduction
tracts. What is the treatment?
a. No treatment is indicated for first & second degree AV blocks
b. For pts w/ third degree AV block due to anterior MI emergent placement of a temporary pacemaker is
indicated
147. The same pt who was post MI develops shock w/ a BP 85/55 3 days post MI. On PE there are
distant pulse sounds & pulsus paradoxus. What happened?
a. This patient suffered a free wall rupture which commonly occur 1-4 days post MI due to the weakening
of the myocardium. These develop hemopericardium & cardiac tamponade that may quickly result in
death.
b. 90% mortality rate
148. What is the treatment for the previous pt?
a. Hemodynamic stabilization & immediate pericardiocentesis & surgical repair
149. Name 5 other post-MI mechanical complications
a. Rupture of interventricular septum which can occur up to 10 days post MI
b. Rupture of the papillary muscles resulting in mitral regurgitation which dx is confirmed using an
echo & tx requires valve replacement
c. Ventricular pseudo aneurysm which is incomplete free wall rupture, usually contained by the
pericardium. This is a surgical emergency since it may quickly progress to a free wall rupture
d. Pericarditis treated w/ aspirin….NSAIDS & steroids are contraindicated since they may hinder scar
formation
e. Dressel’s syndrome, also known as postmyocardial infarction syndrome, which is an immunologically
based syndrome consisting of fever, malaise, pericarditis, leukocytosis & Pleuritis weeks to months post
MI.
150. The following drugs are used in post-MI tx in the CCU. Which drugs have been shown to decrease
mortality in these pts?
1. Morphine sulfate
2. Nitroglycerin
3. Beta blockers
4. CCB
5. LMWH
6. ACE inhibitors
7. Statins
8. Aspirin
b. Answer
1. Morphine & nitrates have NOT been shown to reduce mortality but cause VENOdilation
thereby reducing preload & myocardial oxygen demand
2. Beta blockers reduce HR/BP/Contractility thereby ↓ myocardial oxygen demand & DO
REDUCE mortality
3. CCB reduce afterload & contractility but do NOT reduce mortality
4. LMWH, specifically enoxaparin, are used to prevent progression of thrombi but they have
NOT been shown to decrease mortality
5. ACE inhibitors, statins & aspirin DO reduce mortality
151. A mother brings her 6 yo girl because she failed to toilet train. The girl has normal perception of
feeling of need to void, urinates at regular intervals but is constantly wet w/ urine. What is the likely dx?
a. This girl has low implantation of one ureter which is planted below the sphincter therefore she is wet w/
urine all the time. This does not occur to boys therefore even when a ureter inserts low it is still under
control.
b. PE may show a ureter emptying to the vagina
c. IVP or delayed phase CT (Urogram) will show where the ureter drains
152. When is the most critical period for visual development?
a. During the first 3-4 months of life is when visual acuity greatly improves. Any pathologic condition that
disrupts alignment (Strabismus) or retinal stimulation (cataracts) during this period may result in poor
vision as a result of amblyopia
153. Define amblyopia
a. Amblyopia is poor vision as a result of abnormal visual stimulation (misalignment or ↓ stimulation)
resulting in abnormal visual development
b. Amblyopia is the MCC of ↓vision during childhood.
c. Children are most susceptible to amblyopia during the first 3-4 months of life since this is the critical
period for visual development. The severity of amblyopia depends on how early & how long the
refractive error or misalignment was present.
154. How do you manage children at risk for amblyopia?
a. Correct the misalignment or refractive error & then patch the normal eye thereby forcing the shitty eye
to develop
b. The earlier the intervention the better the prognosis
155. How would you confirm strabismus in a child?
a. Shine a bright light & see whether or not the reflection comes off fall the same spot
156. How do you manage a 4 yo who only recently developed crossed eyes but beforehand hand
normal alignment?
a. Kids who early in life have normal alignment but cross eyes later it suggests a refraction problem rather
than actual strabismus therefore this does not require surgical correction but rather glasses which will
correct the crossed eyes
157. A 55 yo man has unequivocal signs of multiple liver metastases but no primary tumor has been
identified by multiple diagnostic studies of the abdomen & chest. The only abnormality of PE he has an
artificial eye. He reports he had a tumor of the eye when he was 22. What is the likely dx?
a. This is an example of melanoma of the choroid plexus of the eye which may present w/ metastases 20
years down the road or immediately after the dx. Melanoma does not follow the normal rules of
metastases & it may present in crazy random places such as the cardiac ventricles or wall of duodenum
158. What is the difference between cardioversion & defibrillation & state when each is indicated
Cardioversion Defibrillation
Mechanism Delivery of shock that is Delivery of shock that is NOT in
synchronized w/ the QRS complex synchrony, purpose is to convert a
in order to terminate dysrhythmias dysrhythmia to normal sinus
rhythm
Indications  AFib  VFib
 Atrial flutter  VT without a pulse
 VT w/ a pulse
 SVT

159. A newborn baby is found to have an imperforate anus. What is the next step?
a. Must look for other anomalies that are part of VACTER & tx whatever is found.
160. A 16 yo boy goes on a binge beer drinking binge for the first time of his life. Shortly after he
develops colicky flank pain that bothers him when he urinates. What is the likely dx?
a. This is obstruction of the uretro-pelvic junction. Normally there is enough lumen for normal urine flow
but when challenged w/ increase diuresis the obstruction causes pain
161. 28 hrs post fx of his right femur 2/2 MVA a pt c/o SOB & dyspnea while resting. On PE you see
that he is tachypneic w/ petechiae on his axilla & legs. What do you suspect?
a. Fat embolism, tx is respiratory support
162. Any adult who presents w/ hematuria without trauma must get a urological work up to r/o
cancer. Describe the steps of this workup.
a. First step is IVP or CT Urogram (delayed phase) which will detect renal tumors & those in the ureters
b. Followed by cystoscopy & cytology since the IVP will not detect bladder cancers
163. A full term newborn baby born to a mother without proper prenatal care presents w/ a scaphoid
abdomen & has severe respiratory insufficiency. Upon auscultation of his left lung you hear
borborgymus. What do you suspect?
a. A scaphoid abdomen w/ respiratory insufficiency & borborgymus (bowel sounds) heard in
the lung field is suggestive of a diaphragmatic hernia which resulted in herniation of the
bowel into the thorax.
b. The respiratory insufficiency is from pulmonary hypoplasia due to the abdominal contents
within the chest preventing adequate development of the lungs.
c. Always on the left due to presence of liver on right, This is usually picked up on prenatal care
164. Why is bag-and-mask ventilation preferred over intubation for correcting the pt hypoxemia?
a. A bag-and-mask is CONTRAINDICATED since it will push air into the GI tract which will further increase
compression of the lungs!! Intubation is the preferred method.
b. Also orogastric suctioning is used to reduce GI distention
165. A young mother c/o of pain along the radial side of the wrist. She admits that the pain depends on
the position of her wrist, specifically wrist flexion & simultaneous thumb extension when she holds her
baby. On PE pain is reproduced by asking her to hold her thumb inside a closed fist & then forcing the
wrist into ulnar deviation. What is going on?
a. This is classic De Quervian tenosynovitis…the pain when a thumb inside a closed fist & ulnar deviation
of wrist is classic diagnostic sign
166. What is the first line tx?
a. Steroid injections
167. What is used for tx in a pt who just developed a stroke?
a. Clot busters such as tPA (alteplase)
b. Days post stroke we can only perform rehab & CT scan to determine extent of damage
168. What is the MCC of cor pulmonale?
a. COPD
169. What blood abnormality is commonly seen in pts w/ COPD?
a. Polycythemia which further worsens the pulmonary HTN & contributes to development of cor
pulmonale
170. T/F upper extremity DVT rarely emboli & rarely cause PE
a. True! Usually seen in IV drug users
171. How is the dx of a PE made?
a. Helical CT w/ contrast is the initial study of choice w/ a sensitivity >90% since it can visualize very small
clots but may miss clots < 2mm
i. Ventilation-perfusion lung scan used to be the test of choice & is used when there are
contraindications to helical CT scan
ii. Ultrasound of the leg may be helpful if it shows a DVT then you can automatically assume the pt
has a PE otherwise if negative it does not help
iii. Pulmonary angiography is the gold standard but it is invasive
iv. D-dimer test is specific fibrin degradation product & high levels can indicate DVT or PE. If D-
dimer levels are normal then you can r/o DVT or PE since it is a very sensitive test.
172. What is one common contraindication to spiral (helical) CT for dx of PE?
a. Renal insufficiency because of the IV contrast required
b. Avoid if Cr > 2
173. How are PE treated?
a. Oxygen supplementation to correct hypoxia
b. Acute anticoagulation therapy w/ heparin to prevent another PE
i. Start immediately on a basis of clinical suspicion do not wait for studies to confirm PE if suspicion is
high
ii. Give one bolus followed by continuous infusion for 5 to 10 days w/ a goal aPTT of 1.5 of control
c. Oral warfarin for long-term treatment w/ a goal INR 2- 3
174. What is used for long term management in pts w/ PE if anticoagulation is contraindicated?
a. IVC filter = Greenfield filter
175. What is the MOA of heparin & 3 contraindications to heparin use?
a. MOA is by forming a heparin-antithrombin complex which inactivates thrombin & other proteases such
as factor Xa.
b. 3 contraindications are uncontrolled HTN, active bleeding, recent stroke or HIT
176. What is the standard recommendation for anyone w/ an abdominal hernia?
a. Have elective surgical repair to prevent the possibility of having a loop of bowel strangulated
177. Name an exception to this rule in infants
a. We do not recommend elective repair of umbilical hernias before the age of 3
178. T/F you need to repair an inguinal hernia found on standard physical exam in a 20 yo
a. True!!!
179. What is the MOA of atrial fibrillations (AFib)?
a. Multiple foci in the atria fire continuously in a chaotic pattern causing irregularly irregular pulse &
instead of intermittent atrial contractions the atria just quivers continuously thereby fucking w/ the
ventricular filling
180. What are the clinical features of AFib?
a. Fatigue, exetrional dyspnea, palpitations dizziness, angina & syncope
181. Pts w/ AFib are at an increased risk for developing….
a. Thromboembolism & hemodynamic compromise due to ↓ diastolic filling ….this is true especially in pts
w/ AFib & the presence of underlying heart disease.
b. The blood stasis which is secondary to ineffective contractions may lead to formation of intramural
thrombi which may embolize to the brain.
c. Most commonly formed in the atrial left atrial appendage
182. What is the management of acute AFib?
a. If pt is hemodynamically unstable then electrical cardioversion to sinus rhythm
b. If pt is stable then CCB are first line w/ a target HR of 60 -100 & then electrical cardioversion after rate
control is achieved
i. BB are second line
c. Anticoagulation for 3 weeks prior to cardioversion & 4 weeks post OR get a transesophageal echo &
if negative for thrombi then cardioversion & 4 weeks of anticoagulants
183. What is the treatment for chronic AFib?
a. Rate control w/ CCB or BB
b. Anticoagulation & aspirin
184. What is the INR goal in these pts?
a. INR goal of 2-3
185. Pt w/ severe DKA presents to the ER w/ severe dehydration & serum K of 5.2. How do you
manage this pt?
a. Immediately give IV insulin & hydration as well as monitor K. The acidosis stimulates release of K from
cells  falsely elevated serum K. Once the acidosis is corrected the K will move back into the cells &
since a lot of K has been excreted by the kidneys the result is hypokalemia therefore be ready to
supplement w/ K.
186. What is the max mEq of K that can be given in one hour?
a. Do not exceed given 10 mEq of K an hour. The catch is for DKA we can give up to 20 mEq.
b. Normally we eat 75 mEq of K a day therefore 10 mEq an hour is more than enough
187. What are we worried about in pts w/ hyperkalemia?
a. It may cause cardiac arrest & arrhythmias
188. What is the most effective way to protect a pt w/ hyperkalemia quickly?
a. Administration of Ca which will stabilize the myocardium
189. A 65 yo man presents w/ a 4 cm hard mass in front of his ear. The mass is deep to the skin & is
fixed. He has constant pain & for the past 2 month he has progressive worsening of ipsilateral facial
palsy. What is the treatment?
a. Total parotidectomy w/ sacrifice of the facial nerve w/ lymph nodes. Afterwards we will perform a
nerve graft where we take a portion of a sensory nerve & plant it on the facial nerve.
190. What is the most common type of parotid tumor?
a. The MCC of parotid tumors are pleomorphic parotid tumors which are benign painless masses which
require superficial or complete parotidectomy
191. A 45 yo lady w/ a h/o recent tooth infection shows up w/ a red hot fluctuant tender mass
occupying the left lower side of the face & part of the neck, including the underside of the mouth. Pt is
febrile. What is the dx & how would you manage it?
a. This is Ludwig’s angina which is an abscess of the floor of the mouth which is dangerous since it may
interfere w/ the airway therefore it requires both I&D & possible tracheostomy to preserve an airway.
192. What is a clue that tells you a question is going to be a neurosurgery question?
a. The question tells you whether the pt is right or left handed
b. The minute it tells you GxPy it suggests an Ob-Gyn question
193. Which vessels feed the occipital lobe?
a. Vertebral arteries feed the back part of the head
194. What study is done to visualize plaques/stenosis in the vertebral arteries in pt’s w/ TIA w/
posterior sx?
a. Begin w/ duplex scanning of carotids as well as an arteriogram
195. Pt suffers a MVA w/ multiple blunt traumas. On arrival pt is grimacing in pain, treated &
stabilized. The next day on rounds you notice the pt has right facial palsy. How should you manage this
palsy?
a. This is an example of facial palsy 2/2 to swelling since yesterday the pt was grimacing. No tx is
necessary since the swelling will come down on its own.
196. A 30 yo woman wakes up w/ right facial palsy. What do you suspect & what is the tx?
a. This is Bell’s palsy which requires either steroids or immediate anti-virals
197. A 2 yo has unilateral right wheezing & a CXR shows a darker right lung. The mom reports an
episode of coughing earlier today. What is the next step?
a. Anytime you hear unilateral ENT problem suspect a foreign body which will not always show on XRAY
therefore perform a rigid bronchoscope & remove the foreign body.
198. A good rule of thumb is that for every ___ mEq of Na above normal is equivalent for loss of 1 liter
a. An increase in 3 mEq of Na is = to loss of 1 L of water
b. i.e. a pt w/ a Na of 155 has lost around 5 L of water
199. How would you tx a dehydrated pt w/ a Na concentration of 155?
a. Slow rehydration over 24 hours to prevent cerebral edema. We want to correct his hypovolemia quickly
but not the high serum tonicity therefore management is 5 liters of ½ normal saline will help correct the
volume but have only a modest impact on the tonicity
b. Do not give pure D5W
200. A 64 yo w/ h/o recurrent sinusitis calls you c/o seeing double. She was diagnosed w/ frontal &
ethmoid sinusitis 7 days ago. What are you worried about?
a. This is an emergency since it suggests involvement of the optic nerve either via orbital cellulitis OR
infection of the cavernous sinus which all these nerves travel through.
201. 8 days after a difficult gastrectomy for cancer the pt begins to leak 2 to 3 liters of green fluid a day
through his abdominal wound. The pt is afebrile & is otherwise normal. How do you manage this pt?
a. Although pt does not have any peritoneal signs there is a fistula that causes a high volume loss. The
appropriate management is to
i. Adequately replace the pt’s fluid & electrolyte loss, most appropriately w/ Ringer’s lactate.
ii. Nutritional replacement since we must place the pt NPO this can be done by either parenteral
nutrition or placing a catheter distal to the fistula (preferred enteral route) & administering
elemental nutrients.
iii. Protection of the abdominal wall by removing the highly acidic fluid that is leaking
b. With appropriate support most of these fistulas heal by themselves
202. What are some things that prevent the fistula from healing
a. FRIEND: foreign body, radiation, inflammation, epithelization, neoplasm, distal obstructin
203. Foul smelling sputum usually suggests….
a. Anaerobic infections, commonly due to aspiration of oral flora
204. What lung segments are most commonly affected by pulmonary aspiration?
a. The right lung, specifically the lower sections of the right upper lobes & the upper sections of the right
lower lobe
205. Fill out the following table comparing the different arrhythmias
Pathophysiology Heart Causes ECG findings Treatment
rate
Atrial   
flutter
Multifocal   
atrial
tachycardia
SVT   
WPW   

Pathophysiology Heart Causes ECG findings Treatment


rate
Atrial One foci in the atrium fires >250  COPD most common  Saw—tooth w/ QRS  If stable then BB
at a rate of 250-300 giving association after every 2nd or 3rd P or CCB
flutter rise to regular atrial  Heart dz wave  If unstable then
contractions  ASD cardioversion
(synchronized)
Multifocal Multiple foci in atria fire at  Pts w/ severe  At least 3 different P  Amiodarone or
random patterns pulmonary dz wave morphologies digoxin
atrial (COPD) needed for dx  Cardioversion is
tachycardia NOT effective
SVT MCC is AV nodal reentrant  Ischemic heart dz  Narrow QRS w/ no P  Stimulate vagus
tachycardia where two  Digoxin toxicity wave since its buried (PNS)
pathways within the AV  Accessory pathway within the QRS b/c stimulation like
node so that reentrant  Excessive caffeine or circuit is short & carotid massage
circuit is within the AV alcohol conduction is rapid or cold water on
node face which
decrease SA &
AV activity,
 IV adenosine
 Cardioversion if
unstable
WPW Accessory conduction  Congenital  Delta wave (upward  Radiofrequency
pathway form atria to deflection before QRS) ablation of the
ventricles causes PVC  Short P-R interval accessory
since it bypasses the AV  Wide QRS pathway
node therefore no delay  Type 1A or 1C
antiarrhythmics
but avoid
digoxin since it is
active on the AV
node
Ventricular Rapid firing of three or 100-250  CAD w/ prior MI is  >3 PVCs in a row  Sustained VT
more PVCs in a row at a MCC (>30 sec) may
tachycardia rate between 100 – 250  Ischemia lead to death &
bpm. AV dissociation is  Cardiomyopathy tx w/
present meaning P waves  Prolonged QT defibrillation
continue unaffected by the syndrome  Nonsustained is
tachycardia asx & needs to be
further
evaluated
206. What is the long term prophylactic tx for SVT?
a. Digoxin is the drug of choice
b. Verapamil & BB are alternatives
207. 43 yo male c/o 1 mo of blood in his stool. What is the next step in management?
a. Any pt w/ positive fecal occult blood testing requires a colonoscopy which is the most specific &
sensitive test for dx of colorectal cancer
b. Sigmoidoscopy is diagnostic for 2/3 of pts w/ CRC since it is shorter
208. How is CEA used in screening for CRC?
a. CEA is NOT used for screening but rather used for baseline & recurrence surveillance of CRC
209. T/F almost all CRC tumors arise from adenomas
a. True!
210. What is the most common site of metastases of CRC?
a. The liver is the most common site of distant spread
211. What age is a RF for CRC?
a. Everyone > 50 yo is at a risk for CRC
212. What characteristics of adenomatous polyps increase the risk for malignancy
a. ↑ size
b. Villous adenomas > tubular adenomas
c. ↑ number of polyps
213. T/F IBD increases the risk for development of CRC?
a. True! UC more so than CD
214. What are you worried about in a pt w/ Torsades de pointes?
a. It leading to VFib & sudden cardiac death.
215. How is this treated?
a. IV Mg or calcium provides cardiac stabilization
216. Name 3 causes of torsades de pointes?
a. Prolonged QT syndrome
b. TCA
c. Electrolyte abnormalities
d. Anticholinergics
e. ischemia
217. An 18 yo boy presents w/ epistaxis, pt denies picking his nose & no source of anterior bleeding is
seen by PE.
a. In this age group the 2 most common causes of epistaxis NOT 2/2 to picking of the nose are either septal
perforation 2/2 coat hanging abuse or a posterior juvenile nasopharyngeal angiofibroma which is a
benign bleeding tumor that happens to younger pts. The problem w/ JVA is that it has relentless growth
& must be surgically removed.
b. With picking the nose the most common site of bleeding from the anterior part of the septum from direct
trauma which may be tx w/ local pressure.
218. How can you distinguish dizziness of cerebellar origin from the inner ear?
a. If the room is steady & the pt is stumbling the problem is from the cerebellum but if pt is standing still & the
room is spinning it suggests the problem is from the inner ears/vestibular apparatus
219. A 72 yo hypertensive male on aspirin for arthritis has a copious nose bleed. Pt has a BP of
220/115 mmHg when seen in the ER. He says he began swallowing blood before it began coming out of
his nose. How should you manage this pt?
a. This is a nosebleed 2/2 HTN which may be life threatening therefore it requires MORE than local
pressure but rather tx w/ posterior packing or emergency ENT consult for operation
220. What is the definition of sudden cardiac death?
a. Unexpected death within 1 hour of sx onset secondary to a cardiac cause
221. What test would you order for a 17 yo female w/ a 1 cm breast lump found on PE?
a. Age is the most important determinant of how invasive a test you need for diagnosis of breast cancer. In
very young pts a FNA is enough whereas in older pts an excisional bx is more often done.
b. Remember that mammography is used to screen for masses that are not felt on PE therefore if this is felt
on PE there is no need for further imaging at the time. On the other hand we should get mammography in
older women before an invasive procedure.
222. By what age do we begin routine mammography screening?
a. Around age 45
223. What are the only contraindications to mammography?
a. Do not perform a mammogram in women under the age of 20 since or lactating women
b. We can perform mammograms in pregnant pts!!
224. How is airway hyper reactivity assessed?
a. Methacholine challenge
225. What is the most common cause of VFib?
a. Most common ventricular tachycardia but in the acute setting ischemia or infarct may precede VFib
226. Which has a higher rate of recurrence, VFib due to MI or VT?
a. VT has a higher rate of recurrence (30%) therefore these pts require a defibrillator or Amiodarone
prophylactic therapy
b. If VFib develops within 48 hours of an acute MI long term prognosis is favorable & recurrence rate is low
therefore no chronic therapy is indicated.
227. What are the PE findings in pts w/ VFib?
a. Pt is unconscious, cannot hear heart sounds, feel pulses or measure BP since the ventricles are quivering
228. What is the first treatment for VFib?
a. Defibrillation (=Unsynchronized cardioversion) & CPR
b. Assess pulse between shocks & continue CPR
229. What if VFib continues?
a. If VFib continues then administer 1 mg IV bolus of epinephrine every 3 minutes to increase myocardial &
cerebral blood flow & decrease defibrillation threshold
230. What if cardioversion is successful?
a. Continuous IV Amiodarone
b. Implantable defibrillator is mainstay of chronic therapy
231. What are the ECG findings in VFib?
a. No P or QRS waves….pretty much no waves since the ventricles are just quivering due to the irregular
rhythm
232. T/F narrow QRS complex tachycardia originate ABOVE ventricles whereas wide complex
tachycardia’s originate within the ventricles
a. True! The wide QRS tachycardia is more omnious since it may progress to VFib
233. What are the normal ranges for the PR, QRS, & QTc interval
a. PR interval (AV node function) 120-200 ms = 3-5 boxes
b. QRS interval 80-120ms = 2-3 boxes
c. QTc interval 360-440 msec = 9-11 boxes
234. What does PR prolongation (>200 msec) suggest?
a. AV node block
235. What is the tx for AV block?
a. For first & second degree AV block no tx is needed
b. For third degree (Complete) AV block a pacemaker is required….here there is no correspondence
between the P & QRS waves
236. Drugs ending in –pril are?
a. ACE inhibitors end w/ the suffix –pril
b. ARB have the suffix –sartan
237. Which receptor is blocked by ARBs, AT1 or AT2?
a. ARBs block AT1 (responsible for vasoconstriction) receptors not AT2 (vasodilation)
238. Name 6 major polyposis syndromes & describe each

Description % progress to CRC Associated with Comment


Familial Autosomal dominant 100% CRC by 3rd or …. Prophylactic
adenomatous dz characterized by 4th decade of life colectomy
polyposis hundreds of polyps in
the colon &
duodenum.
Gardner’s syndrome Polyps plus osteomas, 100% by their 40s  osteomas
dental abnormalities,  Dental
sebaceous cysts & abnormalities
benign soft tissue  Sebaceous cysts
tumors  Benign soft
tissue tumors
 Desmoid tumors
Peutz-Jeghers Multiple hamartmoas Very low malignant  Pigmented spots
throughout GI tract potential around lips, oral
mucosa, face &
genitalia
 Intussusception
or GI bleeding
due to
hamartomas
Turcot’s syndrome Autosomal recessive  Medulloblastoma
inheritance of polyps  Glioblastoma
plus cerebellar multiforme
medulloblastoma or
glioblastoma
multiforme
Familial juvenile Hundreds of juvenile Small risk of CRC  Juvenile polyps Rare
polyposis coli colon polyps w/ only
a small progression of
CRC
Hereditary  Lynch syndrome Early onset CRC 
nonpolyposis CRC 1 (site specific
(without CRC) early onset
adenomatous absence of
polyposis) antecedent
multiple
polyposis
 Lynch syndrome
2 is cancer family
syndrome w/
features of lynch
1 plus increased
number of early
occurrence of
other cancers

239. How is CRC staged?


a. Duke’s staging

Stage 5 year survival T __, N __ , M ___ Description


Stage A 90% T1-2, N 0 M0 Limited to muscularis
mucosa
Stage B 70% T3-4, N0,M0 Past muscularis mucosa but
no nodal involvement
Stage C 40% Any T, N1-3, M0 Positive regional lymph
nodes
Stage D 5% Any T, any N, M1 Distant metastases

240. What is the most common presenting sx of CRC?


a. Abdominal pain b/c remember that CRC is the MCC of large bowel obstruction in adults
241. Compare R vs L sided CRC
Right Left
Obstruction? Less common due to large diameter More common
Common findings  Melena  Bright red blood since it has
 Occult blood in stool since it had less time for it to mix w/
has had time to mix w/ the the stool
stool
 Iron deficiency anemia
Change in bowel habits? Not common Common alternating constipation &
diarrhea w/ pencil stools due to
narrowing of lumen
Most common presentation Anemia Pencil stools w/ obvious presence
of blood in stool
Comment Triad of anemia, weakness & RLQ
mass

242. What is the tx for CRC?


a. Surgical resection of tumor & associated regional lymph nodes
b. CEA levels should be obtained before surgery as a baseline
c. Adjuvant therapy for pts w/ Duke’s stage C colon cancer w/ 5-FU & leucovorin indicated
d. Pts w/ Duke’s stage B or C RECTAL cancer receive both 5 –FU & radiation therapy
e. Radiation therapy is only indicated in the tx of rectal cancer not colon cancer
243. Inflammatory polyps/pseudo-polyps are associated with…
a. UC
244. What is sick sinus syndrome?
a. Sick sinus syndrome is a sinus node dysfunction characterized by persistent & spontaneous sinus
Bradycardia. Sx include dizziness, syncope, fatigue & CHF
245. What is the most effective treatment in a pt w/ underlying heart disease & nonsustanined (<30
sec) VT?
a. Implantable defibrillator…remember that if the pt only had non-sustained VT without underlying heart
dz then he would not need to be treated
246. Cannon A waves in the neck in a pt w/ sudden onset of heart palpitation, dyspnea,
lightheadedness & angina is suggestive of….
a. This is an example of sustained ventricular tachycardia….the cannon A waves in the neck are due to the
AV disassociation therefore atrial contractions during ventricular contraction resulting in back flow
247. Which vagal maneuver is best in tx for ventricular tachycardia?
a. None!! Unlike SVT ventricular tachycardia does NOT respond to vagal maneuvers or adenosine which
both act on the AV node & VT is independent of that node.
248. What is used for tx of sustained VT in hemodynamically stable pts?
a. IV Amiodarone, IV procainamide (class 1a) or IV Sotalol (class 3 K channel blocker)
b. For unstable pts do cardioversion followed by IV Amiodarone
249. What is the long term treatment for pts who suffered a sustained (>30 sec) VT?
a. Intracardiac device like a pacemaker
250. For each of the following cases state what type of hemorrhoids it is
1. A 60 yo man known to have hemorrhoids reports bright red blood on his toilet
paper after evacuation…
2. A 60 yo man known to have hemorrhoids c/o anal itching & discomfort,
particularly toward the end of the day. He has mild perianal pain while sitting
down & finds himself sitting sideways to avoid the pain.
b. Answers
c. The rule of thumb is that internal hemorrhoids bleed but do not hurt while external hemorrhoids hurt but
do not bleed.
1. This is an example of internal hemorrhoids since he is bleeding but no pain (IB = Internal
bleed)
2. This is an example of external hemorrhoids since he is in pain but no bleeding (EH =
External hurt)
251. What is the first step in management of each of the previous cases?
a. The first step in managing all anorectal dz is to r/o cancer via a rectal exam & a rectosigmoid exam
252. What is the tx for internal hemorrhoids?
a. Since these have no pain innervation then we can ligate the hemorrhoids w/ rubber band or laser
ligation.
253. What about tx for external hemorrhoids?
a. These require a formal operation w/ anesthesia since it is innervated w/ pain fibers.
254. A 32 yo woman presents to ER w/ swollen red hot tender eye lids on the left eye. She has fever &
leukocytosis. On prying her eyelids open you see that her pupil is dilated & fixed & she has little ability to
move her left eye. What is the likely dx & what is the first step in management?
a. The fact that the pupil is dilated & fixed w/ limited motion points to orbital cellulitis which is an
ophthalmological emergency since it can lead to blindness.
b. First step is a CT scan to detect whether or not this can be drained
255. A 23 yo woman describes severe pain w/ defecation & blood streaks on the outside of the stool.
B/c the pain she avoids going out the bathroom & that results in hard stools which are even more
painful. PE cannot be done since she refuses anyone to look into the area out of fear of precipitating the
pain. What is the most likely dx?
a. This is classic anal fissure which most commonly occur in young women which presents as painful
defection w/ bright red blood around the stool.
256. What is the first step in management of anal fissures?
a. Just like all anorectal dz we must first r/o cancer therefore do a rectal exam & rectoscopy under
anesthesia due to pain.
257. What is the next step in tx after you r/o anorectal cancer?
a. Stool softeners or topical agent that help relax the sphincter such as nitroglycerin
258. What surgical tx is available for
a. Surgical tx would be either forceful dilatation, injection of botox or lateral internal sphincterotomy.
259. What is used to assess the risk of intra-operative myocardial infarction?
a. Ejection fraction…if it is as low as 35% then it is EXTREMLY high risk pt & the correct answer on the test
will be an answer choice that avoids the operation
i. Normal EF >55%
260. Which is more specific to the liver damage ALT or AST?
a. ALT is more sensitive & specific than AST for liver damage; AST is also found in skeletal & heart muscle,
kidney & brain
b. ALT & AST usually have similar increase in liver damage w/ the exception of alcoholic hepatitis where the
AST:ALT ratio may be >2:1
c. The higher the AST-ALT ratio the more likely alcohol is contributing to the abnormal LFT
261. If ALT & AST are mildly elevated (low hundreds) think ____________, but if they are very elevated
(high hundreds to thousands) think ____
a. Low hundreds think chronic viral hepatitis or acute alcoholic hepatitis
b. Moderately elevated think acute viral hepatitis
c. If severely elevated > 10,000 think hepatic necrosis maybe due to acetaminophen toxicity, ischemia or
shock liver (prolonged hypotension)
262. How does alkaline phosphatase help in narrowing the ddx?
a. Alk phos is found in liver, bone, gut & placenta. Elevated alk phos is seen when there is obstruction to
bile flow in any part of the biliary tree. Normal levels make cholestasis unlikely.
263. What test can help distinguish elevated alk phos due to biliary system from bone, gut or placenta?
a. GGT is specific to the liver system
b. If GGT is negative & alk phos is elevated then consider pregnancy or bone dz (Padget’s)
264. A 3 wk old was BIB mom for non-bilious vomiting. On PE you notice an irritable but hungry infant
& you palpate a small round hard mass in his epigastric region. What do you suspect?
a. Pyloric stenosis presents as projectile NON-bilious vomiting w/ an olive mass in the epigastric region
265. What electrolyte imbalance is seen in these pts after prolonged vomiting?
a. Hypochloremic hypokalemic metabolic alkalosis
266. What is the best diagnostic study?
a. Ultrasound is the best
b. Upper GI study will show a narrow pyloric channel (“string sign”)
267. What is the first step in tx?
a. Start off w/ fluid resuscitation & correction of the hypocholeremic, hypokalemic metabolic alkalosis
268. What is the next step?
a. Partial pyloromyotomy after correction dehydration & electrolyte imbalances
269. A 62 yo man has vague poorly described epigastric & upper back discomfort. On PE he is found to
have a 6 cm pulsatile mass in the abdomen which is tender to palpation. How is this managed?
a. A tender AAA is suggests that this is getting ready to pop therefore confirm dx w/ US but get emergent
vascular repair
b. If pt has excruciating lower back pain w/ a pulsatile mass suggests rupture therefore the right answer is to
call for emergency vascular repair
270. A 73 yo obese mother of 4 children presents w/ 3 day h/o RUQ pain that radiates to her shoulder.
The pain started off as colicky but has been persistent for the past 2 days. The pt has abdominal
tenderness to deep palpation, muscle guarding & rebound tenderness. Pt has fever of 104 w/ chills. WBC
of 22,000 w/ a left shift of 11. Alk phos is elevated. What is the first step in management & what is the
likely dx?
a. This is RUQ pain w/ high fever & leukocytosis in an elder pt classic for acute ascending cholestasis.
b. The first step is US of the RUQ which should show dilated CBD w/ inflammation.
271. How is this treated?
a. NPO, NG tube, IV abx & decompression of the biliary tract w/ ERCP 24-48 hrs later
272. A 42 yo woman hits her breast w/ a broom handle. She presents c/o a 3 cm mass in that breast
that has not gone away in 4 weeks. What is the next step in managing this pt?
a. H/o trauma does NOT r/o the possibility of cancer therefore manage this as usual w/ a mammogram
followed by appropriate tissue bx
b. Many pts don’t realize they have masses until they suffer a trauma & then they attribute the mass to the
trauma
273. What would you expect to see on bx if this is indeed 2/2 to trauma?
a. Macrophages w/ fat globules since this will be fat necrosis
b. On PE there might even be nipple retraction & mammogram may show COARSE calcifications in the mass
but this is still benign
274. A 4 yo spilled draino on his eye. How do you tx it?
a. Wash the eye under running water for at least 30 mins of irrigation
275. What do you do after the irrigation of the eye?
a. Test pH in the eye before d/c since pt may still have alkali which needs to be removed
276. A 58 yo woman discovers a mass in her right axilla. She has a discrete hard moveable 2 cm mass.
PE of her breast is negative & no LAD elsewhere. What is the first step in managing this pt?
a. Mammography to detect breast masses that are not found on PE
b. Proceed w/ tissue sampling if masses are found on radiography or bx of the lymph node,
277. What if the previous pt was a 22 yo woman w/ palpable mass in axilla as well as generalized
LAD?
a. Start thinking lymphoma or possible EBV if HSM
278. T/F PT is not prolonged until most of the liver’s synthetic capacity is lost which corresponds to
advanced liver dz
a. True therefore an abnormal PT is seen in advanced liver dz
b. PT measures the extrinsic pathway which are Factors 2,5,7,10 & fibrinogen; PT is prolonged in pts on
coumadin
279. Pt presents after a MVA w/ suspected intraabdominal bleeding. What is the next step in
confirming the dx?
a. The next step depends whether or not if he is hemodynamically stable. If he does respond to fluid
resuscitation then get a CT abdomen if he is not hemodynamically stable (did not respond o fluids) just
perform a FAST US exam.
b. FAST exam = intraperitoneal free fluid detection test
280. What would be the next step if the FAST exam is inconclusive & pt is at 90/60 mmHg?
a. Get a free peritoneal fluid sampling to confirm the dx whether or not the pt is bleeding
281. What is the next step if the FAST exam does show free intraperitoneal fluid
a. Exploratory laparotomy
282. What is the most common source of significant intraabdominal bleeding in blunt abdominal
trauma?
a. The spleen, these pts usually present w/ left lower rib fx
b. Every effort is made to save the spleen, not like the movies where it is just taken out; if spleen is removed
then must give postop immunization against pneumococcus, HIB & Meningococcus
283. An 18 yo boy presents to the ER w/ a left wrist drop & decreased sensation over his left forearm &
posterior arm. Pt is on crutches for 2 weeks since he sprained his ankle playing basketball. What do you
suspect?
a. Wrist drop suggests decreased function of the extensors which are innervated by the radial nerve.
Furthermore the decrease in sensation over the forearm & posterior arm is also suggestive of injury to
the radial nerve which is commonly seen in pts w/ improperly fitted crutches.
284. During a prolonged abdominal surgery for multiple traumas a pt begins bleeding everywhere
suggesting the development of coagulopathy (excessive bleeding). A few minutes later his temp starts
dropping. What do you do?
a. Intraoperative development of coagulopathy is not uncommon during prolonged abdominal surgery. Tx
is platelet packs & FFP (10 units of each). If pt also develops hypothermia & acidosis then the laparotomy
will be terminated w/ packing of bleeding surfaces & temporary closure. The operation will be resumed
later once the coagulopathy is treated & pt has warmed up.
285. What do you need to do in a pt who needs emergency operation & his Rx list consists of
amlodipine, warfarin, digoxin, lithium & ibuprofen?
a. Give FFP to correct the effects of Warfarin prior to surgery otherwise pt will bleed profusely
286. A toddler has a h/o the flu a week ago & now refuses to move his right leg. He is laying down w/
his hip flexed & externally rotated. What do you suspect?
a. Septic hip or transient synovitis
287. How do you confirm the dx?
a. Aspiration of joint fluid under sedation
b. Transient synovitis will have an ESR <40, lower temperature & pts will be able to bear weight on the joint
288. How is a septic joint managed?
a. Open arthrotomy for drainage
289. What if the previous child c/o localized pain over his tibia?
a. Suspect acute hematogenous osteomyelitis which presents as more localized pain over a bone
290. How is this diagnosed?
a. Bone scan or MRI
b. Radiographs do not show presence of osteomyelitis until 2 weeks later!!
291. A 32 yo male presents to the ER c/o pain in his hand & thumb. He reports he had a skiing injury
yesterday where he fell in the snow & used his hand to break the fall. On PE you notice laxity of his
thumb-metacarpophalangeal joint. What is the likely dx?
a. This is an example of Gamekeeper/Skier thumb which is an injury to the ulnar collateral ligament
sustained by forced hyperextension of the thumb.
292. What is the tx?
a. Casting is first line since if left untreated it can be dysfunctional & may lead to arthritis
293. Pt c/o wrist pain. You ask her to make a fist w/ the thumb inside the fist & deviate the fist in the
ulnar direction. This maneuver reproduces the pain. What is the likely dx?
a. De Quervain tenosynovitis which is commonly seen in young mothers who carry their baby forcing their
hand in wrist flexion & thumb extension to hold the baby’s head.
294. How would you tx this pt?
a. First line therapy is steroid injection…out of all the orthopedic injuries in the wrist only De Quervain &
trigger finger are tx first line w/ steroid injections
295. A 28 yo man is brought in by his mother. 3 mo ago he has had several operations for repair of a
perianal fistula. After each surgery it failed to heal & now has become bigger. He now has multiple
draining fistulas w/ no palpable masses. What is the most likely dx & what is the first step in
management?
a. This is an example of perianal dz in a Chron’s dz patient. The first step in all anorectal dz is to r/o cancer
via rectal exam & follow w/ colonoscopy.
296. Pt reports seeing flashes of light at night as well as “floaters” that have been increasing in
number. Furthermore the pt reports that recently he noticed a “big cloud” at the top of his visual field.
What is the first step in management?
a. This pt has retinal detachment. The flashes of light at night are due to tugging & pulling on the optic
nerve causing the perception of light. As the retina detaches more debris comes off which are the
floaters. The number of floaters suggests the severity of the dz. The cloud is the actual detachment
which if it is at the top of his visual field it really is at the bottom of his retina.
b. RD is an emergency therefore must consult Optho quickly for laser tx to prevent further detachment
297. What is the next step in a 65 yo pt who on PE is found to have a deep pulsatile abdominal mass
between the sternum & the umbilicus?
a. US to dx the AAA
298. T/F all AAA >6 cm require repair
a. True!
299. T/F all AAA less than or equal or 4 cm may be observed
a. True! Watchful waiting for all AAA <4cm
300. A 27 yo woman presents w/ 6 mo h/o HA, visual loss & amenorrhea. On day of admission she c/o
severe HA, marked deterioration of vision & stupor. BP is 75/45 mmHg & fundoscopic examination
shows bilateral pallor of the optic discs. CT shows a subarachnoid hemorrhage. What is going on & what
does this pt need most desperately?
a. This lady has a growing tumor for the past 6 mo but she had deterioration w/ a sudden HA &
neurological deterioration which suggests rupture of the tumor & bleeding into the head. This pt had a
pituitary adenoma & the vessels to this tumor ruptured  pituitary apoplexy  adrenal insufficiency 
shock which may be corrected w/ administration of steroids
b. You should not expect such severe hypotension only from the bleeding since there is not enough room in
the head to fit enough blood to cause hemorrhagic shock
301. A 72 yo man reports leg pain when walking that is relieved w/ rest. On further questioning you
find out that he has to sit down or bend over for the pain to go away where standing up & resting does
not relieve the pain. He can walk for long periods of time w/o pain if he is bent over such as pushing a
shopping cart of riding a bike. What is going on in this pt?
a. This is neurogenic claudication 2/2 to osteophytes growing in the spinal canal & compressing the cord
which is characterized by pain only when walking in certain position (not when bending over) that is
only relieved when sitting or bending over. This is NOT PVD causing claudication since it is mostly
positional
302. A 54 yo right handed laborer notices coldness & tingling in his right hand when he does
strenuous work. What really concerns him is that during the past several episodes he has had transient
vertigo, visual abnormalities & difficulty articulating speech. What is the next step in management?
a. Since there is a combination of arm complaints & posterior part of brain neurological complaints then
you likely have subclavian steal syndrome where there is an atherosclerotic plaque in the origin of the
subclavian artery. This allows enough perfusion for normal function but inadequate perfusion w/
exertion. Once there is an increased requirement for perfusion the perfusion is met by stealing the blood
away from the brain & redirects it to the arm. Claudication of arm + neurological sx
b. Angiogram which will show reverse of flow & a bypass will fix the problem
303. After laparotomy to repair intra-abdominal bleeding post MVA it is difficult to close the
abdominal wall due to tissue swelling. What do you do?
a. This is an example of abdominal compartment syndrome that occurs when lots of fluids & blood have
been given during a prolonged laparotomy. The solution is to place a temporary cover over the
abdominal contents such as an absorbable mesh that later can be grafted over or a non-absorbable
plastic that may be removed at a later date.
304. A 63 yo male c/o several month h/o worsening dyspnea. He admits to using 4 pillows when he
sleeps. He also complains of a non-productive cough that is worse when he lies down. He reports that he
wakes up every 2 hours feeling out of breath. Pt is afebrile, obese, w/ a 40 pack year smoker & h/o
poorly controlled HTN. On PE you notice a left displaced PMI & on auscultations you hear rales at the
lung bases & an S3. What do you suspect?
a. Left sided CHF which would result in pulmonary edema  dyspnea, orthopnea, paroxysmal
nocturnal dyspnea (waking up after 2 hours b/c SOB).
b. Signs of left sided CHF are left displaced PMI due to cardiomegaly.
c. S3 (“ventricular gallop”) which is heard b/c of early rapid filling to a noncompliant left ventricle.
d. Crackles & rales at lung bases
305. What serum marker may be used to differentiate dyspnea caused by CHF from COPD since they
both may present similarly?
a. B-type natriuretic peptide levels > 100 pg/ml are suggestive of CHF since this hormone is released by the
ventricles in response to excessive stretching of the ventricles due to volume expansion & pressure
overload.
306. What is the function of B-type natriuretic peptide?
a. BNP binds atrial natriuretic peptide receptors & increase natriuresis (↓ reabsorption of Na) thereby ↓
EABV / blood volume thereby lowering systemic BP & afterload  ↑ CO
307. What is the preferred tx for small cell lung cancer?
a. Small cell lung cancer is tx w/ chemotherapy & radiation (NOT surgery)…think of small cells as small as
chemicals
b. NSSC of the lung are tx w/ surgery whereas SCC is tx w/ radiation & chemotherapy
308. A 62 yo woman has an exematoid lesion in the areola. It has been present for 3 month. It looks
like some kind of skin condition that has not gone away w/ application of a variety of lotions. No masses
are felt on PE. What is the likely dx & how do you manage this pt?
a. First mammogram followed by obtaining tissue samples
b. This is classic Padget’s dz which is a cancer of the breast that is not palpable as a mass but is infiltrating
tissue of her areola.
309. A 77 yo man w/ h/o CAD & HTN calls you since he suddenly loses sight from his right eye. Pt
denies any other sx. What advice do you have?
a. Since the only function that is affected is sight this is likely an embolic occlusion of the retina which is an
emergency therefore tell the pt to be driven to the ER & en route to breath in a paper bag & have
someone push his eye which might dislodge the clot. The breathing in a paper bag  CO2 retention 
vasodilation of vasculature in the head  easier to dislodge the clot.
b. If pt also had other neurological sx it would likely be a occipital stroke
310. When trying to diagnose a pt w/ pancreatitis, how do you decide whether or not to get serum vs
urine amylase/lipase?
a. Serum levels are elevated within 12 hours but back to normal in 2 days whereas urine levels are
elevated after 2 days & continue to be elevated for quite a while so it depends how long after the onset of
pain you see the pt
311. How do you diagnose pancreatic txp rejection?
a. Must take a bx to assess for rejection but amylase & lipase will also be elevated but not enough for
diagnosing rejection
312. What about assessing for rejection if the pancreas was connected to the bladder?
a. Check urine amylase which will be DECREASED if rejection is occurring
313. T/F S3 is always pathological
a. False! It may be normal in children but it is usually associated w/ CHF in adults
b. S4 is when the atria contracts (atrial systole) into a noncompliant left ventricle.
314. Where & how is S3 best heard?
a. S3 is best heard at the apex using the bell of the stethoscope….S3 may be difficult to hear but it is one of
the most specific signs of CHF
315. Define CHF
a. CHF is a clinical syndrome where the heart is unable to meet the body’s circulatory demands under
NORMAL physiological conditions
316. What are the signs & sx associated w/ right heart failure?
a. Jugular venous distention (JVD)
b. HSM
c. Ascites
d. Edema, specifically pedal
317. Fill out the following table

MOA Echo Causes Treatment


shows…
Systolic  
dysfunction
Diastolic  
dysfunction
High output heart  
failure

MOA Echo Causes Treatment


shows…
Systolic Decreased ventricular ↓ Ejection  Recent MI is MCC  Sodium restriction (< 4
dysfunction contractility fraction (<  Cardiomyopathy (dilated) g/day)
40%)  Myocarditis  Loop diuretics most
effective in relieving sx (no
change in mortality)
 ACE inhibitor reduce
pre/afterload & reduce
mortality
 Beta-blockers ↓ mortality
in pts w/ POST-MI CHF
Diastolic Decreased ventricular Impaired  HTN  myocardial  Few tx available for
dysfunction filling during diastole due relaxation hypertrophy is MCC diastolic dysfunction
to ↑ wall stiffness to  Valvular dz such as aortic therefore just treated
impaired relaxation stenosis/regurgitation symptomatically
 Restrictive
cardiomyopathy
(amyloidosis, Sarcoidosis,
hemochromatosis)
 Hypertrophic
cardiomyopathy
High Increase in cardiac output Both systolic  Chronic anemia
output is needed to meet the & diastolic  Pregnancy
heart requirements of dysfunction  Hyperthyroidism
failure peripheral tissues are present  AV fistulas
 Wet Beriberi (vit B1 def)
 Mitral & aortic
regurgitation

318. What tests should you order for pts w/ new onset of CHF?
a. CXR to check for pulmonary edema, cardiomegaly & r/o COPD.
i. Kerley B lines indicate pulmonary congestion secondary to dilation of pulmonary lymphatic
vessels
b. CBC to r/o anemia as the cause of high output CHF
c. ECG
d. Cardiac enzymes to r/o MI
e. Echocardiogram is the initial test of choice which will estimate ejection fraction, diastolic vs systolic
dysfunction & help distinguish between pericardial, valvular or myocardial processes
319. What does the Frank-Starling relationship refer to?
a. Basically it states that in a normal heart an increase in preload  ↑ contractility
320. Fill out the following table comparing the three types of cardiomyopathy

Dilated Hypertrophic Restrictive


LV EF
Wall thickness
LVEDV
Size of atria
Systolic function
Diastolic function
Clinical presentation
PE findings
Causes
Tx
Dilated (most common) Hypertrophic Restrictive
LV EF Lowered (↓ systolic function) High Normal
Wall thickness Normal Thick Normal
LVEDV High Normal/Low Normal/low
Size of atria Large Large Very large
Systolic function Reduced Increased (high EF) Normal
Diastolic Normal Reduced Reduced
function
Clinical - systolic CHF since the problem is - Triad of: diastolic CHF, syncope, - diastolic CHF
presentation a ↓ in systolic function (↓ EF) angina - edema & dyspnea
- both R & L CHF develops - sudden cardiac death in Exercise intolerance due
athletes to elevated filling
pressure
PE findings  S3 & S4  S4
 Signs & sx of R & L CHF  Systolic murmur that ↑ w/
standing & Valsalva & decreases
w/ squatting, laying down &
hand grip
 Bisferious pulse
Causes  CAD w/ prior MI is MCC  Autosomal dominant  Amyloidosis
 Alcohol inheritance  Sarcoidosis
 Doxorubicin (Adriamycin)  Hemochromatosis
 Chagas, Lyme dz, HIV  scleroderma
 Thyroid dz
 Peripartum cardiomyopathy
 Thiamine def (Beriberi)
Tx Digoxin, diuretics, BB, cardiac
transplantation
 Remove alcohol or PTCA for tx
of ischemia
***hand grip increases murmur due to mitral valve prolapse but decreases hypertrophic murmurs, both decrease w/
squatting & increase w/ standing

321. A young recruit c/o localized pain on his tibia after a forced march. He is tender to palpation over
a specific point of the bone but radiographs are normal. ESR is WNL & pt is afebrile. What is the likely
dx?
a. This is a classic presentation of a stress fx which does not immediately show on radiographs. It takes a
couple of weeks for the radiographs to show the fx
b. Tx is splinting & repeat imaging in 2-3 weeks
c. The key to stress fx is repetitive use of the bone way beyond what it can tolerate such as a forced march or
marathon running.
322. What are the two esophageal hiatal hernias & which one requires surgical repair?
a. Type 1 is a sliding hiatal hernia where both the LES & a portion of the stomach herniate into the thorax
through the esophageal hiatus. This is common & benign finding in pts w/ GERD. This accounts for 95%
of all cases of esophageal hernias.
b. Type 2 is a paraesophageal hiatal hernia which is more rare & here only the stomach herniates into the
thorax where the LES does not. This requires surgical repair since it may become strangulated.
323. A 69 yo woman has a 4 cm hard mass in the right breast w/ ill-defined borders. The mass is
moveable from the chest wall but not moveable within the breast. The skin overlying the breast has
retraction of the nipple w/ an orange peel appearance. Is this likely to be cancerous?
a. This is classic breast cancer, anytime you see retraction of the skin think breast cancer
b. Orange peel appearance is classic for inflammatory breast cancer
324. How are these pts managed?
a. Mammogram & US first
b. Tissue sampling next
c. Axillary node bx
d. Appropriate tx consisting of lumpectomy/modified radical mastectomy + chemo or hormonal therapy +
radiation tx
325. A 44 yo man presents to the ER w/ severe perianal pain. He cannot sit down & reports that BM are
very painful. He has had chills & fever. PE shows a hot, tender, red fluctuant mass between the anus &
the penis. What is the first step of management?
a. Although this is a classic perirectal abscess since this is an anorectal dz the first step is to r/o cancer via
rectal exam & endoscopy.
b. The tx for perirectal abscess is I&D which will result in immediate improvement
c. Think about Hidradenitis suppurativa in pts w/ recurrent perirectal & axillary abscesses 2/2 infxns of
apocrine or sebaceous glands
326. How would management change if the previous pt is an obese male w/ advanced uncontrolled
DM?
a. Pts w/ advanced DM have poor circulation & perianal abscess tend to progress to necrotizing fasciitis
therefore must have close f/u on these patients.
327. The previous pt comes to your clinic 3 months after his presentation to the ER & the draining of
the perianal abscess. He c/o of perianal discomfort & fecal streaks staining his boxers. PE reveals a
perianal opening in the skin & a cord like tract can be palpated. Brownish discharge can be pushed out
of the tract. What is the next step in management?
a. This is classic perianal fistula which only occurs in pts who had a perianal abscess drained or Crohn’s dz.
There is no other way a pt may develop these fistulas therefore you know these pts had a perianal I&D.
b. As always the first step in managing all anorectal dz is to r/o cancer
c. Tx is to “un-roof” the fistula which means removing the epithelial tissue in the tract allowing the
granulation tissue to close it out.
328. What types of cancer occur in the perianal area?
a. Adenocarcinomas & squamous cell carcinomas of the anus
329. Which type of pts are more likely to develop rectal squamous cell carcinomas?
a. SCC is more likely in HIV positive pts or pts who have anal sex (homosexual men)
330. Where does the adenocarcinoma of the rectum metastasize?
a. Adenocarcinoma of the rectum metastasizes only to lymph nodes inside the abdomen
331. What about SCC of the anus?
a. SCC metastasizes to the internal abdominal lymph nodes & to groin lymph nodes
332. A 45 yo out of shape male experiences a loud pop & pain of his ankle while playing a game of
tennis. Pt is still able to move his ankle but it is otherwise in plantar extension. PE reveals a gap in the
tendon in the posterior part of the ankle. What is the likely dx?
a. Rupture of the Achilles tendon
333. How is this tx?
a. Surgery has quicker recovery but casting in the plantar flexion position also works
334. What is the presentation of endocarditis?
a. FROM JANE = Fever, Roth spots (retinal hemorrhage), Osler nodes on fingers, new Murmur, Janeway
lesions (erythematous lesions on palms & soles), Anemia of chronic dz, Nail bed splinter hemorrhages,
Emboli
335. Endocarditis 2 weeks after surgery most likely due to….
a. Staph epidermidis are the MCC of early onset endocarditis sx if it appears within 60 days of surgery
b. Strep viridians (α hemolytic) most common late-onset endocarditis 60 days after surgery
336. MCC of endocarditis in drug users?
a. Staph aureus is MCC of endocarditis in drug users, more commonly right sided
337. What is used for tx of infective endocarditis?
a. Vancomycin OR ceftriaxone + gentamicin for 4 weeks
338. What Rx is used as prophylaxis for endocarditis in pts w/ known valvular heart dz or prosthetic
valves?
a. amoxicillin
339. What two Rx may cause drug induced lupus syndrome?
a. Hydralazine which is an arterial dilator
b. Procainamide which is a class 1a antiarrhythmic
340. A pt presents to the ER w/ chest pain, give at least 10 different things on your ddx for chest pain
a. Heart, pericardium & vascular causes such as stable angina (exertion) , unstable angina (rest),
Prinzmetal’s angina (rest), MI (>30 min), pericarditis, aortic dissection
b. Pulmonary causes such as pleuritis, pneumothorax, pneumonia, asthma attack & PE
c. GI causes such as GERD, esophageal spasms, PUD, esophageal rupture
d. Chest wall causes such as costochondritis, muscle strain, rib fracture, herpes zoster, thoracic outlet
syndrome
e. Psychiatric causes such as panic attacks, anxiety, somatization
f. Cocaine use can cause angina or MI
341. What if the chest pain is relieved w/ sublingual nitroglycerin?
a. It is most likely a cardiac cause
342. What murmur is decreased w/ sustained handgrip & explain the MOA
a. A murmur due to hypertrophic cardiomyopathy DECREASES w/ sustained handgrip since it increases
systemic resistance  ↓ gradient across aortic valve
b. The murmur also decreases when squatting, lying down or raising the legs since it increases preload
which expands the ventricles thereby decreasing the obstruction & murmur
c. The murmur INCREASES w/ Valsalva & standing since it decreases preload
343. When is surgical valve replacement of aortic valve stenosis indicated?
a. When there is a pressure gradient >50 mmHg across the valve OR pt is sx w/ syncope or CHF
344. A 13 yo chubby boy c/o pain in his left knee & groin pain. The pt developed a limp the past 2
weeks. He sits in the office w/ the sole of the foot of the left side pointing toward the other foot
(externally rotation of the leg). PE of left knee is normal & his hip has limited motion & is externally
rotated. Pt denies any trauma. What do you suspect?
a. SCFE which is seen in fat AA adolescents
345. How is it diagnosed?
a. diagnosed w/ radiographic imaging
346. How is this managed?
a. Pinning of the femoral head in place
347. How can the urine color help determine the cause of jaundice?
a. If pt has dark urine it suggests conjugated hyperbilirubinemia since only conjugated bilirubin is water
soluble
b. Pale stools also suggest conjugated hyperbilirubinemia likely due to biliary obstruction
348. Where does the majority of bilirubin come from?
a. 80% of bilirubin are products of hemoglobin breakdown in the spleen  unconjugated bili  binds
albumin but not water soluble  conjugated in liver  secreted to the intestines  intestinal bacteria
convert it to urobilin & urobilinogen  reabsorbed by entero-hepatic circulation
b. The other 20% are due to myoglobin breakdown
349. What is Budd-Chiari syndrome?
a. Liver dz due to occlusion of venous outflow resulting in hepatic congestion microvascular ischemia
b. Dx using hepatic venography & serum ascites albumin gradient >1.1
350. How is Budd-Chiari treated?
a. Medical therapy is usually unsatisfactory therefore surgical placement of stents in the IVC or portocaval
shunts
351. Fill out the following table comparing obstructive to restrictive lung dz
Obstructive Restrictive
FEV1
FEV1/FVC ratio
Peak expiratory flow rate
Residual volume
Total lung capacity
Vital capacity

Obstructive Restrictive
FEV1 Low Normal
FEV1/FVC ratio Low Normal
Peak expiratory flow rate Low Normal
Residual volume High Low normal or high
Total lung capacity High Low
Vital capacity Low Low

352. T/F the most important intervention for COPD is smoking cessation
a. True! Smoking prolongs the survival rate in these pts but DOES NOT reduce it to the level of someone
who has never smoked. Quitting smoking decreases the rate of decline of FEV1 to that of someone who
does not smoke BUT it does not result in complete reversal of sx
353. A 67 yo man presents w/ 4 mo h/o throbbing lower back pain. On PE you notice ecchymosis
around his umbilicus & flanks & you feel a pulsatile mass during his abdominal exam. What is the likely
dx & how would you confirm it?
a. Abdominal aortic aneurysm (AAA) is the most likely dx. I would confirm the dx w/ ultrasound which is
100% sensitive, or CT scan.
b. The ecchymosis on the back, flanks & umbilicus are called the Grey-Turner’s sign & Cullen’s sign
354. What is the treatment for unruptured AAA?
a. If aneurysm is >5cm in diameter or symptomatic then surgical resection w/ synthetic graft placement is
indicated
b. If aneurysm is < 5 cm & asx then periodic imaging is recommended….remember that there is no safe size
& any size may rupture
355. What is the presentation of a ruptured AAA?
a. The classic triad is sudden onset of abdominal pain, hypotension & a palpable pulsatile
abdominal mass which requires emergency laparotomy.
356. Asthma has a PFT that shows obstruction. What change in the PFT would you expect when a pt
w/ asthma exacerbation inhales bronchodilators?
a. ↑ in FEV1 & FEV1/FVC ratio of at least 12% which is the criteria for reversible bronchoconstriction
357. Where is the most common site of AAA?
a. Between the renal (infra-renal) arteries & the iliac bifurcation
358. What is the average age of dx of AAA?
a. AAA incidence increase w/ age most commonly presenting at 65 & rarely before 50
359. What is the MCC of AAA?
a. Atherosclerotic weakening of the aortic wall
360. Name two causes of thoracic aortic aneurysms?
a. Syphilis & connective tissue abnormalities such as Marfan dz
361. Coronary stenosis is significant & can produce angina once it is > ____ %
a. >70%
362. Name 2 prognostic indicators in patients w/ coronary artery disease (CAD)?
a. If the left ventricular ejection fraction is <50% it indicates a poor prognosis
b. If the left main coronary artery is involved it indicates a poor prognosis. This is because the left main
coronary artery supplies 2/3 of the heart w/ blood
i. Two or three vessel CAD has the worse prognosis
363. How is CAD diagnosed?
a. Get a resting ECG as well as a stress ECG which requires the patient to exercise sufficiently to increase his
heart rate to 85% of maximum.
b. If there are ST segment or T wave abnormalities at rest & negative troponin it suggests unstable angina
c. If exercise induces abnormalities it suggests stable angina
364. What ECG finding is suggestive of stable angina during a stress test?
a. ST segment DEPRESSION which is indicative of subendocardial ischemia
b. Other positive findings include ventricular arrhythmias or onset of heart failure (ejection fraction <
50%)
365. What is the next step in managing pts w/ a positive stress test?
a. Cardiac catheterization
366. What non-invasive test is preferred over stress ECG for diagnosis of CAD?
a. Stress echocardiogram where an echo is done before & after exercise since they can directly visual
ventricular function & wall motion abnormalities as well as diagnose valvular disease
367. What can be used in addition to a stress test to enhance its sensitivity?
a. Stress myocardial perfusion imaging after IV administration of thallium 201 during exercise. This
radioisotope will be taken up by VIABLE myocardial cells from the blood & none by the non-viable cells
therefore you can directly visualize areas that are not receiving blood. If the areas of hypoperfusion are
perfused over time then it suggests reversible ischemia which should be rescued w/ PTCA or CABG.
368. What is considered a positive stress test?
a. A stress test is positive if a patient develops ST segment depression, chest pain, hypotension or any
arrhythmias during exercise
369. What is the gold standard for definitive diagnosis of CAD?
a. Coronary angiography is the definitive test for CAD & it is indicated for patients being considered for
CABG
b. Coronary angiography is the most accurate method to determining CAD
370. When is cardiac catheterization indicated?
a. After a positive stress test
b. Pt that has angina despite medical therapy or soon after MI
c. Severely symptomatic patients
d. Evaluation of valvular disease & determine the need for surgical intervention
371. A 4 yo fat infant has bowing on his right tibia below the knee. The child has lateral thrust
(shifting of weight away from midline; penguin walk) when walking. What are you suspecting?
a. Since this is unilateral angulation below the knee in a child > 3 yo I suspect Blount’s dz which is
progressive angulation at the proximal tibia thought to be a result of overload injury to the medial tibial
growth plates resulting in growth on the lateral side but not on the medial  angulation.
b. Blount’s dz should be considered in any child w/ progressive bowing, unilateral bowing or persistent
bowing after 3 years of age.
c. < 3 yo is normal
372. How would you confirm the dx?
a. AP radiographs of the leg
373. How is Blount’s dz tx?
a. Bracing for a year & if no improvement then surgery.
374. College football player gets tackled & later c/o knee pain. Anterior drawer test & Lachmann test
are both positive, what is the dx?
a. Tearing of the ACL
b. If you can push the knee posteriorly (posterior drawer sign) it suggests a torn PCL
375. A 27 yo immigrant from Mexico has a 10 x 10 x 7 cm mass in her left breast. It has been present
for 7 yrs & has been slowly growing to its present size. The mass is firm, non-tender, rubbery completely
moveable & not attached to chest wall or axillary skin. No presence of axillary LAD. What is the next step
in management?
a. This is classic for cystosarcoma phyllodes which is similar to fibroadenoma. This is seen in females in
their late twenties where it grows slowly to a large size. The mass is firm, rubbery & moveable which is
similar to fibroadenoma. Since this has malignant potential FNA will not suffice therefore it requires bx
& resection of the mass
b. Fibroadenomas are seen in younger pts & may be dx w/ FNA or US. Resection is not needed but usually done
for cosmetic purposes.
376. A 34 yo woman in her fifth month of pregnancy reports a 3 cm ill-defined mass in her right breast
that has been present & growing for 5 month. What is the next step in management?
a. Get a mammogram to r/o presence of other masses. Remember that mammograms are NOT
contraindicated during pregnancy.
b. Also must get tissue bx. If this is cancer then she will undergo surgery during her pregnancy just no
radiation
377. A 35 yo woman has a 10 yr h/o tenderness in both breasts related to her menstrual cycle w/
multiple lumps that come & go. She now has a firm round 2 cm mass in her right breast that has not gone
away for 6 wks. What is the next step?
a. The description in the beginning is classic for fibrocystic dz which is seen in females between 20 -40 as
cyclical tender lumps that come & go. The fact that the 2 cm mass has not gone away suggests that it is
likely a cyst but it must be followed therefore first step is mammogram.
378. What is the tx if the mass is indeed a cyst?
a. Aspiration of the cyst
b. If bloody aspirate then send to culture
379. What do you do if after you aspirate the cyst it re-forms in 2 days?
a. Must follow up w/ tissue sampling.
380. A 34 yo woman has been having bloody discharge from the right nipple on & off for several
months. There are no palpable masses. What is the next step in management & the likely dx?
a. The first step is mammogram.
381. What is the most likely dx if the mammogram is negative for masses?
a. This is most likely intraductal papilloma which classically produces bloody nipple discharge in women in
their 30s & will not show up on mammogram
382. How can you confirm the dx of intraductal papilloma?
a. Galactogram or retro-areolar surgical exploration
383. A 26 yo lactating mother has cracks in the nipple & develops a fluctuating red hot tender mass
along w/ fever & leukocytosis. What is the next step in management?
a. This is likely a breast abscess which is ONLY seen in lactating women. Although mammography is
usually the first step to r/o cancer remember that mammography is contraindicated in lactating women
& women < 20yo. When you I&D the abscess get tissue sample to r/o cancer.
384. A 49 yo woman has a firm 2 cm mass in the right breast that has been present for 3 month. What
is the first step in management?
a. Mammography to r/o other masses that are not found on PE
385. What is the next step post mammography?
a. Since this pt is older & therefore at higher risk for breast cancer we will need more tissue (not FNA)
therefore I would get a core tissue bx
386. A 72 yo man is scheduled to have an elective sigmoid resection for recurrent diverticulitis. On
preoperative evaluation it is noted that he has JVD. What is the significance of this PE finding w/ regards
to the surgery?
a. This suggests that the man is in CHF which is a contraindication for elective surgery therefore we must
have medical tx for the CHF before we perform the surgery.
b. JVD in a preoperative setting suggests CHF for testing purposes
387. A middle aged homeless man comes to the ER w/ severe pain in his forearm. He admits to have
drinking a lot of EtOH & passing out on a park bench. There are no signs of trauma but severe tenderness
on palpation of his forearm & worse pain on passive extension of his fingers. Pulses are intact. What is
the likely dx?
a. Compartment syndrome due to prolonged ischemia, when he was intoxicated he likely slept on a twisted
arm resulting in cutting off of circulation to the arm. When he wakes up there is reperfusion of the
forearm resulting in swelling & ↑ pressure in the compartment. The presence of normal pulses DOES
NOT exclude compartment syndrome.
b. Excruciating pain upon passive extension of the digits is hallmark of compartment syndrome.
388. How would you tx this pt?
a. Emergency fasciotomy is the tx for all compartment syndrome
389. A basketball player c/o knee pain, he has seen various physicians which have said that his
radiographs are normal & Rx NSAIDs which have not helped. He describes how he feels as if his knee is
“catching & locking”. On PE his knee is swollen. What is the likely dx & what is the next step?
a. First of all remember that a swollen knee is the poor man’s MRI & it usually suggests real pathology. The
catching & locking of the knee is classic for tearing of the meniscus which will NOT show up on
radiographs. Also the fact that this was difficult to dx further supports this dx.
b. The next step should be an MRI of the knee which confirm the dx of a meniscal tear
390. How should a meniscal tear be treated?
a. Surgical repair. We try to save as much of the meniscus as possible since if there is a complete
mensictomy degenerative arthritis will ensue.
391. Name 3 different options to induce stress in a patient that is not able to perform exercise for a
stress test & describe the MOA?
a. Pharmacological stress test can be performed if a patient cannot exercise
i. IV adenosine or dipyramidole result in coronary vasodilation which works because diseases
coronary artery are already maximally vasodilated & when you cause generalized coronary
vasodilation the diseased arteries receive relatively less blood resulting in ischemia.
ii. IV dobutamine increases myocardial oxygen demand by increasing heart rate, contractility & BP.
392. Name 5 risk factors that need to be modified in patients w/ a positive stress test
a. BP control
b. Smoking cessation which decreases the risk of CAD in half after one year
c. Reduction of serum cholesterol
d. Weight loss
e. Exercise
f. Diet
g. Strict glycemic control has a greater effect on MACROvascular disease rather than microvascular but it
should still be emphasized
393. What is the medical Rx for CAD & describe the MOA of each?
a. Aspirin which is an irreversible thromboxane inhibitor on platelets thereby reduces the risk of
thrombosis
b. Beta blockers which reduce HR, contractility & BP thereby reducing cardiac work  ↓ myocardial
oxygen demand
c. Nitrates relieve angina by causing generalized vasodilation  ↓ preload  ↓ myocardial oxygen
demand
i. Main benefit is symptomatic relief, effect on prognosis is unknown
d. CCB  coronary vasodilation & ↓ afterload
i. CCB is considered a secondary treatment after beta blockers and/or nitrates fail
e. If CHF is present then tx w/ ACE inhibitors & diuretics
394. What are the SE of nitrates?
a. HA
b. Syncope & orthostatic hypotension
c. tolerance
395. When is revascularization w/ CABG or PTCA indicated?
a. When pts have severe angina, >2 vessels involved or any decrease in EF
396. When is CABG preferred over PTCA?
a. > 3 vessels involves
b. High risk disease
c. Left main coronary artery disease
397. T/F Revascularization of a stenotic coronary artery w/ PTCA or CABG significantly reduces
incidence of MI & sx
a. False! The revascularization reduces symptoms but DOES NOT REDUCE INCIDENCE of MI
398. What is a main difference between chest pain due to MI vs. pericarditis?
a. Pericarditis chest pain can be differentiated from a MI because pericarditis is associated w/ inspiration.
Pericarditis pain is also positional meaning it is worse when laying down & improves when sitting up &
leaning forward.
b. Pericarditis may not always present w/ chest pain but if it does it may radiate to the trapezius & neck
399. What ECG finding is specific for acute pericarditis?
a. Diffuse ST elevation & PR depression
400. Describe the three different ASDs & state which is most common
a. Ostium primum is at the most caudal portion of the atrial septum near the endocardial cushion
b. Ostium secundum accounts for 80% of cases & is in the central portion of the septum
c. Sinus venosus defects occur high in the septum
401. When do ASD become sx in pts?
a. Not until their 40s & they may present as DOE & fatigue
402. What are the PE findings of an ASD?
a. Wide fixed splitting of S2
b. Diastolic flow “rumble” murmur across tricuspid valve
403. Describe bronchiectasis & the MCC?
a. Bronchiectasis refers to permanent abnormal dilation & destruction of bronchial walls & cilia. This most
commonly occurs due to CF (50% of cases)
404. What are the clinical features of bronchiectasis?
a. Chronic cough w/ large amounts of mucopurluent FOUL smelling sputum
b. Hemoptysis due to rupture of blood vessels near bronchial wall
c. Recurrent or persistent pneumonia
d. Dyspnea
405. What type of PFT pattern is seen in bronchiectasis?
a. Obstructive pattern
406. What is the diagnostic study of choice?
a. High resolution CT scan showing dilatation of the bronchial walls
407. Transesophageal echocardiogram shows small warty vegetation on both sides of aortic valve.
What is the likely dx?
a. Vegetations on BOTH sides of valve leaflet is called Libman-Sacks endocarditis which most commonly
involves the aortic valve & is seen in pts w/ SLE
408. T/F Libman-Sacks endocarditis rarely gives rise to infective endocarditis
a. True! Although it rarely gives rise to infective endocarditis it is a source of embolization therefore tx
underlying SLE & anticoagulate these pts
409. An 88 yo male c/o right calf pain several hours after undergoing a right femoral artery
embolectomy. He also c/o burning sensation in his right posterior leg. His BP is 150/70 & HR is 100. His
right calf is swollen & tender w/ worsening of pain upon passive extension. Pulses are intact. What is the
likely dx?
a. Compartment syndrome & soft tissue swelling due to ischemia-reperfusion syndrome. Following more
than 4 hrs of ischemia tissues can suffer both intracellular & interstitial edema upon reperfusion which
creates a risk for compartment syndrome
410. After a MVA a pt has an obvious pelvic hematoma. How do you tx him?
a. No tx needed for pelvic hematomas, just leave it alone as long as he is hemodynamically stable
411. What must you do for any pt presenting w/ a pelvic fx?
a. Associated injuries must be ruled out therefore perform a rectal exam, proctoscopy, pelvic exam &
bladder/urethra using retrograde cystogram
412. What is the tx in pelvic fx w/ ongoing significant bleeding?
a. Rule out bleeding elsewhere & then although tx is controversial the best answer is to do pelvic fixators to
prevent further fx of the bone & angiographic embolization of the internal iliac arteries
413. One day post-laparotomy that was done to fix intraabdominal bleeding post GSW, a pt’s belly
becomes distended resulting in cutting of the sutures through the tissue & dyspnea. What is going on?
a. This is an example of abdominal compartment syndrome which may occur due to tissue swelling after
prolonged laparotomy where lots of fluid & blood was given. Must take out sutures & provide a
temporary cover for the abdominal contents
414. What is the MCC of primary liver cancer?
a. HCC
415. What are the two types of HCC?
a. Non-fibrolamellar which is more common & is associated w/ Hep B/C & cirrhosis. The non-fibrolamellar
type is NOT resectable & has a short survival time
i. HCC develops in 10% of cirrhotic pts
b. Fibrolamellar is less common & is not associated w/ hepatitis or cirrhosis. This is more often resectable
& a better prognosis. More commonly seen in adolescents & young adults.
416. What is the most common benign liver tumor?
a. Cavernous hemangioma (=hemangioma) which increases in size during pregnancy & OCP use
417. What benign liver tumor is the only one not associated w/ OCP use?
a. Focal nodular hyperplasia of the liver
418. Pt presents w/ 8 hrs of chest pain, CXR, troponin & EKG are negative. What is the next step in
management?
a. Admit for a stress test to r/o unstable angina
419. T/F urethral injuries occur almost exclusively in men
a. True! Typically associated w/ pelvic fx & may present w/ blood at the meatus or scrotal hematoma
420. What is the next step in managing a male pt in the OR w/ a pelvic fx & blood at the meatus?
a. DO NOT try to place a Foley since it may further compound the injury!!! Instead perform a
retrograde urethrogram….if someone tried to perform a Foley & met resistance it further suggests
urethral injury
421. Burn victim w/ BP 90/70 presents to the ER w/ a HR of 115 & RR 28. What is the first thing you
should do?
a. Secure airway w/ intubation since these pts tend to develop swelling of their airways later on which
makes it hard to intubate
b. Can confirm w/ Fiberoptic bronchoscopy to assess damage
422. What is the most common respiratory complication during the first 24 hrs post-surgery?
a. Atelectasis which impairs gas exchange & predisposes to pneumonia. This is commonly due to guarding
2/2 to pain thereby ↓ ventilation
423. What would you expect to find on ABG in pts w/ atelectasis?
a. These pts start hyperventilating  hypo-CO2  respiratory alkalosis w/ hypoxemia
424. How can this be prevented?
a. Chest physiotherapy & incentive spirometry have increase the functional residual capacity & prevent
atelectasis
b. Simply elevation the head of the bed of a pt vs supine reduces the abdominal pressure acting on the
undersurface of the diaphragm thereby permitting greater alveolar expansion & increasing FRC
425. Who should you consult in a pt who is newly diagnosed w/ T2DM?
a. Consult Ophtho to check the retina for damage
b. This is not required for T1DM
426. A 6 yo child has a mushy fluid filled mass at the base of the neck that has been noted for several
years. The mass is 6 cm in diameter occupies most of the supraclavicular area & seems by PE to go deep
into the neck. What is the likely dx & first step in management?
a. This is a classic example of a cystic hygromas (cystic lymphangioma) which is a congenital lymphatic
malformation which commonly presents at the posterior triangle of the neck. These are benign but
disfiguring.
b. First step is a CT scan since we need to determine how deep the cystic hygroma extends so we can
surgically remove it & determine what surgical field we need to plan for.
427. A 34 yo woman presents to the ER after a MVA w/ a pelvic fx & gross hematuria. Her vitals are
stable. What is the next step in management?
a. Must perform a rectal & vaginal exam but also get a retrograde cystogram & post-void radiography
which might show extraperitoneal leaks that might otherwise be obscured by a bladder full of dye.
428. A 43 yo male presents to the ER after a MVA w/ enlarged scrotum full of blood. Pt is urinating
normally & no pelvic fx is present & a retrograde cystogram is negative. What is the next step in
management?
a. US of testicle to ensure there is no rupture. If the testicle is intact then no intervention is needed.
429. T/F non-alcoholic steatohepatitis has identical histology to alcoholic liver dz but unlike ALD it
does not result in cirrhosis
a. False! Although both ALD & NASH have the same histology, the majority of NASH are benign but 10% do
develop cirrhosis
430. What portion of the thigh does the femoral artery & vein travel?
a. Anterior medial portion of the thigh therefore any injury to the lateral aspect of the thigh does not pose a
risk for vascular injury
b. If a pt has an injury to this aspect of the thigh then you can either get an arteriogram if he is stable or
operation if he is unstable or has a large expanding hematoma; arteriogram is needed only if we are not
sure whether there is injury
431. Which injury would you tx first in a pt who was shot in the upper arm resulting in shattering of
the humerus & damage to the brachial artery & nerve?
a. Although the vascular repair is more urgent, the bone repair will require a lot of maneuvering which may
destroy the vasculature therefore bone repair comes first. Nerve repair & fasciotomy to prevent
compartment syndrome
432. What sites are most common sites of compartment syndrome?
a. Lower leg & forearms have smaller compartments that are likely to develop compartment syndrome
(>30 mmHg)  ischemia
i. 5 Ps: pallor, pulselessness, paralysis, pain, paresthesias
b. Thigh has a large compartment therefore unlikely to cause compartment syndrome.
433. What is the most likely dx in a pt w/ a CT scan showing a solitary lung nodule that is….
1. 1 cm in diameter & calcified?
2. Popcorn appearance
3. Irregular borders/speculated?
4. A solitary pulmonary nodule that grew over a period of days
ii. Answers
1. Calcification suggests benign lung nodule
2. Popcorn appearance suggests hamartoma
3. Irregular borders/speculated suggests invasive carcinoma
4. If it grows so rapidly over period of days it is likely infectious/inflammatory & NOT
malignant but if it grows over periods of months it is likely malignant
**Any pt >50 yo w/ a pulmonary coin lesion on CXR has a 80% chance of the mass to be malignant
434. What are the 2 next steps in managing a pt w/ a solitary pulmonary nodule discovered on CXR?
a. Check for old CXR
b. We always start w/ the non-invasive methods for diagnoses therefore
i. Order a chest CT w/ contrast & if the CT is suspicious for malignant they get a bx or PET scan
ii. Sputum cytology which has a very low sensitivity but since its non-invasive if it appears on exam
order it
435. What are the next steps in confirming the dx?
a. Bx via bronchoscopy for central masses
b. Percutaneous bx for peripheral lung masses
436. What is the surgical tx for a central mass that end up being non-small cell lung cancer?
a. Pneumonectomy (removal of whole lung) for central masses WITHOUT evidence of mets
b. Lobectomy or wedgectomy for peripheral masses without evidence of mets
c. Avoid surgical removal in pts w/ metastases
437. What is the most important consideration before performing a pneumonectomy for a central lung
cancer?
a. The pt’s lung function
438. What is the lung function requirement that suggests a pt will tolerate a pneumonectomy & be
able to survive on one lung?
a. Pt must have a FEV1 > 800 post surgery
439. How do we determine whether or not a pt will have the appropriate FEV1 (>800) post
pneumonectomy?
a. Get pre-op PFT & then perform a V/Q scan & calculate how must each lung is contributing.
440. What are the two types of lung cancers?
a. Small cell lung cancer (SCLC) which accounts for 25% of cases
b. Non-SCLC which accounts for 75% of cases & includes:
i. Squamous cell carcinoma
ii. Adenocarcinoma
iii. Large cell carcinoma
iv. Bronchoalveolar cell carcinoma
441. Smoking is the biggest RF for lung cancer. Which of the previous type of lung cancer has the
lowest association w/ smoking?
a. Adenocarcinoma has the lowest association w/ smoking
442. Which type of lung cancer is the MCC of superior vena cava syndrome?
a. SCLC most often causes SVC syndrome due to obstruction of SVC by mediastinal tumors
b. PE findings are facial fullness, facial & arm edema, JVD, dilated veins in upper extremities & head
443. For each of the following paraneoplastic syndromes state the most commonly associated type of
lung cancer
1. SIADH
2. Ectopic ACTH
3. PTH-like hormone secretion
4. Hypertrophic pulmonary osteoarthropathy
5. Eaton-Lambert syndrome
6. Pancoast tumor
ii. Answers
1. SIADH is inappropriate ADH secretion  edema & HTN. This is most commonly caused
by SCLC
2. Ectopic ACTH  ↑ cortisol secretion (aldosterone regulated by renin)  ↑ Cushing; MCC
SCLC
3. PTH-like hormone secretion most commonly by squamous cell carcinoma
4. Hypertrophic pulmonary osteoarthropathy results in severe long bone pain & commonly
due to adenocarcinoma & Squamous CC
5. Eaton-Lambert syndrome presents like MG but improves w/ increase used & is
commonly due to SCLC
6. Pancoast tumor is compression of C8-T2 due to SCC in the apical lung also presents w/
Horner’s syndrome
***SCLC is responsible for all the paraneoplastic syndromes w/ the exception of Pancoast & PTH-like hormone
secretion
444. What is the difference in tx between SCLC & N-SCLC?
a. SCLC is tx w/ chemotherapy while surgery has a very limited role
b. NSCLC is tx w/ surgery & chemo has a limited role
445. A 34 yo male presents w/ 4 day h/o profuse watery diarrhea. He was treated w/ ampicillin last
week for strep throat. A KUB shows significant distention of his left colon. What do you suspect?
a. Profuse watery diarrhea w/ abdominal cramps, & toxic megacolon in a pt tx w/ ampicillin, clindamycin
or cephalosporins is suggestive of pseudomembranous colitis due to C. difficile
446. How is the dx confirmed?
a. Demonstrate C. difficile toxins in the stool
b. Flexible sigmoidoscopy is more rapid & diagnostic but expensive & invasive
c. KUB to r/o toxic megacolon & risk of perforation
d. Leukocytosis is common
447. What is the tx for C. diff?
a. Oral vancomycin and/or metronidazole (cannot be used in infants or pregnant pts)
448. Pt c/o of RUQ pain for several months. On US of the liver you notice large fluid filled cysts. CBC
shows eosinophilia what do you suspect?
a. Hydatid liver cysts which are caused by infection from the tapeworm Echinococcus granulosus which
results in cyst formation in the liver.
b. Think amebic liver abscess caused by E. histolytica in homosexual (Fecal-oral route) men (9:1 M:F) who also
have intestinal flask shaped ulcers & fever. Tx is IV metronidazole
449. How are these treated?
a. Must remove the cysts w/ caution to avoid spilling contents & give mebendezole post-surgery
450. Pt has multiple liver cysts on ultrasound & Cr of 2.2. What is the likely underlying dx?
a. Polycystic kidney dz which is an autosomal dominant disorder that often involves the liver. The liver
cysts are usually asx & the prognosis in these pts has to do w/ renal function not so much liver
i. Pts also develop brain aneurysms & HTN
451. What liver lobe is the most common location for liver abscess?
a. Right lobe
452. What is found within the falciform ligament?
a. Round ligament (ligamentum teres) of liver which is the obliterated umbilical vein
453. What are the most common masses in the anterior mediastinal compartment?
a. The terrible Ts: Thymoma (most common), thyroid cancer, tumors, teratoma, thoracic aneurysm
454. What 4 structures are found in the middle compartment?
a. Heart, trachea, bronchi & hilar structures
455. What is the most common mass found in the middle mediastinum?
a. Lymphoma
456. What structures are found in the posterior mediastinum?
a. Esophagus, aorta, neurogenic tumors
457. What portion of the clavicle is most often fx?
a. Typically at the junction of the distal & middle third
458. What is the tx for clavicular fx?
a. Just put the arm in a sling
b. Sometimes for young women fixation by surgery is done for cosmetic purposes
459. A 72 yo man is scheduled to have an elective sigmoid resection for recurrent diverticulitis. In the
preoperative evaluation it is noted that he suffered an acute MI 2 months ago. What is the significance of
this finding w/ regards to the surgery & what is the next step?
a. A recent MI (<6 mo) there is nearly a 40% mortality associated w/ the surgery therefore the most
appropriate step is to wait until at least 6 month have passed since MI
b. Waiting past 6 months brings down the risk to 6%
460. An 8 wk old w/ persistent progressive jaundice. LFTs show elevated direct bilirubin. What do
you suspect?
a. Biliary atresia
461. How would you confirm his dx?
a. A HIDA scan done after one week therapy w/ phenobarbital. Phenobarbital stimulates the liver to
produce as much bile as possible & after a week of that stimulation we do a HIDA scan. If the HIDA scan
shows that the bile does not leave the liver it shows that there is a biliary atresia but if the HIDA scan
shows bile in the liver, gallbladder & the ducts then we must think of another dx.
462. What is the tx for biliary atresia?
a. The majority of these pts require a liver transplant
463. A 23 yo man w/ AIDS has a painless ulcer in his mouth that has not healed. This is likely…
a. Squamous cell carcinoma
464. What is the first step in the work up?
a. Triple endoscopy or pan endoscopy to collect bx
465. Unilateral sensorineural hearing loss in a man who denies going to a shooting range. What is the
first step in work up?
a. Unilateral sensorineural hearing loss is suggestive of a tumor therefore the first step is getting a MRI
466. Any slow growing firm mass in front of the ear, around the angle of the jaw, or right behind the
ear is diagnosed as a _____
a. A parotid tumor until proven otherwise.
467. What is the most common parotid tumor?
a. Pleomorphic adenoma which is a benign painless tumor but it does have potential for malignant
transformation.
b. There is only one chance to remove this w/ sparing the facial nerve & that is the first time we operate
because in the virgin territory we can either do a complete or superficial parotidectomy. If we do
something less than the formal superficial parotidectomy then the tumor will grow back & it will be
more difficult to preserve integrity of the facial nerve 2/2 scar tissue.
468. A 68 yo male w/ h/o smoking & drinking presents w/ rotten teeth & c/o a large firm growing
neck mass. What do you suspect?
a. Metastatic SCC from oral mucosa is seen in pts w/ rotten teeth who smoke & drink
469. How would you confirm the dx?
a. FNA of neck mass followed by triple/pan endoscopy where you put the pt under anesthesia & check all
nasopharyngeal areas & bx all masses
b. DO NOT bx the enlarged lymph node since tx will require extensive surgery & we rather have a “virgin”
neck
470. A 13 yo runner c/o knee pain every time he runs. On PE you note that he has swelling of the tibial
tuberosity & you can reproduce the pain by applying resistance when he extends the knee. Dx?
a. Osgood-Schlatter dz whish is tibial apophysitis (inflammation of tuberous) caused by overuse seen in
basketball players & sprinters
471. What is the tx for OS dz?
a. RICE for 2 – 3 wks
472. A 15 mo healthy infant is brought to the ER because he refuses to walk after falling down on a
carpet. Parents report that he was running when he fell but it was not very force full. A radiograph
shows a spiral fx of his tibia but the fibula is intact. What do you suspect?
a. This is an example of a Toddlers fx which is a spiral fx of the tibia but the fibula remains intact.
473. What is the tx?
a. Long leg casting for 3-4 month
474. What is the first thing you think of when you see a child w/ a spiral fracture?
a. Child abuse….unless the fibula is intact & it’s the tibia where you should think toddler fx
475. What are other fractures typical of child abuse that are picked up on a skeletal survey?
a. Metaphyseal fractures
b. Posterior or first rib fractures
c. Multiple fx at various ages of healing
d. Complex skull fx
476. What ocular finding is associated w/ child abuse?
a. Retinal hemorrhage are highly suggestive of child abuse
477. Describe the Salter-Harris classification & its grades
a. Salter-Harris classification is used to describe fractures involving the physis (growth plate)
Grade Acronym Description
I S Same = fx isolated within the physis
II A Above = fx is in the physis & above into the metaphysis
III L Low = Fx is in the physis & beLow in the epiphysis
IV T Through & through = fx is in the physis & through both metaphysis
& epiphysis
V Crush

478. A 22 yo man develops anorexia, followed by vague periumbilical pain that several hours later
becomes sharp, severe constant & well localized to the RLQ. There is abdominal tenderness, rebound &
guarding. Pt has leukocytosis w/ neutrophilia & slight fever. What is the next step in management?
a. For classic appendicitis the next step is operation before perforation. The reason this is classic is because
it began w/ anorexia & vague periumbilical pain which later became localized & sharp in the RLQ. The
peritoneal signs, leukocytosis & fever al support this diagnosis.
479. What would have been the management if this was a not so classic presentation of acute
appendicitis?
a. Get either an abdominal US or a CT scan
b. Must have an expert to be able to read the US otherwise get a CT scan
480. What must you monitor in pts w/ crushing injuries?
a. Monitor levels of serum & urine myoglobin, K & hemoglobin which is released by muscle which may
cause damage to the kidneys & arrhythmias
b. Monitor for compartment syndrome since damage to the muscles may lead to swelling & requiring
fasciotomy
481. How do you tx these pts?
a. Vigorous fluid administration, osmotic diuretics & alkalization of urine to prevent renal damage
b. Fasciotomy to prevent compartment syndrome
482. A 55 yo woman has been evaluated for protracted diarrhea. On further questioning she gives
bizarre h/o episodes of flushing of the face w/ expiratory wheezing. A prominent jugular venous pulse is
noted on her neck. What is the next step?
a. This is classic carcinoid syndrome 2/2 to a carcinoid tumor that metastasized most likely now in the
liver. The carcinoid tumor is a neuroendocrine cell tumor  secretion of serotonin which results in
diarrhea, bronchoconstriction (wheezing), flushing of the face & mechanical abnormalities of the right
sided heart valves.
b. Most carcinoid tumors are in the appendix but carcinoid syndrome is only seen in pts who have
the tumor metastasize outside the intestines
c. The next step is confirming the diagnoses w/ serum determination of 5-HIAA.
483. What should you worry about in a pt that is placed on clindamycin for a week?
a. Clindamycin is the abx that is most associated w/ development of C. difficile.
484. Bonus question: what is the mechanism of action of clindamycin?
a. Clindamycin is a lincosamide abx that is used to tx anaerobic infections & may be useful for MRSA as well. It
binds the 50s ribosomal unit & fucks w/ bacterial protein synthesis.
485. What is the management in a pt who developed diarrhea while on clindamycin?
a. Remove the abx, in this case clindamycin
b. Oral vancomycin & metronidazole
c. Fecal transplant is the correct answer if provided
d. DO NOT use Imodium or anti-diarrheal medication
486. A mother of a 7 yo who spilled draino all over her arms & legs calls you for help. What should you
tell them?
a. Must do massive irrigation to eliminate the offending chemical since chemical burns are “the gift that
keep on giving” since unless it is removed it will keep burning. After she washes it in the shower for >30
minutes then tell them to come to the ER.
b. The common alkali chemicals are the plumber drainers
c. Common acids are from car batteries & are less serious than alkali
487. She asks you if she should tell her to drink vinegar to neutralize the alkali. What do you say?
a. DO NOT attempt to neutralize acids; this results in an exothermic reaction which will further injury, if
there is ingestion perform endoscopy to assess for injury
488. Which of the following studies must be done before a Nissen fundoplication is done for a pt w/
Barrett’s esophagus & long term GERD refractory to medical tx?
1. Barium swallow test
2. Monometry
3. pH monitoring
4. endoscopy
ii. Answer
1. Must do all of the previous studies!!
489. A 47 yo woman describes difficulty swallowing which she has had for many years. She says
liquids are more difficult to swallow than solids & she has to sit up straight & wait for the fluids to make
it through. Occasionally she regurgitates large amounts of undigested food. What is the most likely dx &
how would you confirm it?
a. The minute you hear that liquids are harder to swallow than solids it suggests a motility/functional
problem rather than mechanical. This is most likely to be achalasia which is best diagnosed w/ a workup
consisting of barium swallow, followed by endoscopy followed by monometry studies.
b. By standing up straight the weight of the liquid will overcome the ↑ tension of the LES.
490. How is achalasia treated?
a. Medical tx w/ repeated dilatations of the LES or botox injections
b. Surgical tx Heller myotomy
491. A 24 yo female develops moderate generalized abdominal pain of sudden onset & shortly
thereafter collapses. At time of evaluation at the ER she is pale, tachycardic & hypotensive. The
abdomen is tender & distended & she has a hgb of 7. She denies pregnancy since she has been taking
OCP for the past 10 years. Pt denies any trauma. What is the first step in management?
a. This is a classic presentation of a rupture of a hepatic adenoma which is seen in females who take OCP
for a long time. These adenomas are not cancerous but may rupture. The first thing on your ddx in a
young female in reproductive age should be ectopic pregnancy which will need to be ruled out. The first
step is fluid resuscitation & a CT scan to confirm the presence & rupture of hepatic adenoma.
b. Think rupture of a hepatic adenoma in a young woman who presents w/ diffuse abdominal pain, bleeding &
has been on long term OCP
492. How is fluid resuscitation calculated for burn victims?
a. Body weight in kg * (% of BSA up to 50%) * 4 cc of ringers lactate without sugar to avoid osmotic
diuresis
b. After 50% burn there is a maximum rate of fluid loss therefore for anything > 50% we use 50%
c. Half of this should be given in the first 8 hours & the other half in the next 16 hours
d. They need an additional 2 L of fluid for eating & drinking
493. How much fluid will the burn pt need on the second day?
a. On the second day they require half the fluids of the first day whereas on the third day there should be
massive diuresis of excessive fluids
494. How is extent of body burn calculated in adults?
a. Head = 9%
b. Trunk = 18% front, 18% back
c. Each leg = 18%
d. Each arm = 9%
495. What about in children?
a. In children the head accounts for 18%
b. Trunk & extremities are same in adult
c. In children both legs add up to 3 X 9% whereas in adults it is 4 9s
496. A newborn is born w/ a “C shaped” foot where there is medial curvature of the metatarsals. What
is this called?
a. Metatarsus adductus which is medial curvature of the metatarsals that is caused by intrauterine
constraints. Dorsiflexion should be intact
b. Must exercise & stretch the foot.
497. What do you suspect if the previous pt could not dorsiflex/move his foot?
a. Talipes equinovarus which is clubfoot which is a fixed foot in inversion & plantar flexion w/ no
flexibility.
498. How is club foot treated?
a. Serial plaster casting within the first week of life addressing the most distal deformity to the proximal
deformity
b. If there is no improvement then surgery which is not done until 6 mo
499. A 63 yo alcoholic presents w/ epigastric abdominal pain, N/V & diarrhea. He reports that his
stool is very smelly & looks “fatty”. Pt’s glucose is 280. What is the likely dx?
a. Chronic pancreatitis which would result in the pain as well as the steatorrhea & DM secondary to
endo/exocrine dysfunction of the pancreas
b. Classic triad of steatorrhea, DM & pancreatic calcification on plain films or CT is diagnostic
500. Which is more helpful in confirming the diagnosis of chronic pancreatitis, serum amylase or
lipase?
a. Neither!!! These are not elevated in chronic pancreatitis, only in acute episodes
501. What is the medical management for chronic pancreatitis
a. Alcohol abstinence
b. Bowel rest & narcotic analgesics for pain
c. Pancreatic enzyme & H2 blockers (give simultaneously). The pancreatic enzymes inhibit CCK release &
therefore decrease pancreatic secretions after meals while H2 blockers inhibit gastric acid secretion
thereby preventing degradation of the pancreatic enzyme supplements by gastric acid.
d. Once the pt begins to eat it is better to have frequent small volume low fat meals
502. What surgical tx are available for chronic pancreatitis
a. Pancreaticojejunostomy is the most common procedure for chronic pancreatitis & it is where the
pancreatic duct drainage procedure to decompress the dilated pancreatic duct.
503. What is the Whipple procedure?
a. Whipple procedure which is a pancreaticoduodenectomy where the pancreas, duodenum & other organs
are removed….commonly due to malignant tumors
504. A 16 yo has a 4 wk h/o bone pain in his right femur. There is a mass on the femur that shows a
sharp well defined edge between the mass & bone. What is the most likely dx?
a. Sharp well defined edge between mass & bone suggests a benign bone mass
b. Ill-defined border suggests malignancy
505. Fill out the following table comparing Osteogenic sarcoma to Ewing’s sarcoma
Osteogenic sarcoma Ewing’s sarcoma
Description
Peak age of incidence
Site
Local findings
Systemic findings?
Radiographic findings
Metastases
Dx made by

Osteogenic sarcoma Ewing’s sarcoma


Description Malignant tumor that forms osteoid (new bone) Characterized by small, round blue cell
tumor
Site Metaphysis of long bones, distal femur around Flat bones & diaphysis of tubular bones
knee most common Most commonly around the pelvis
50% occur near the knee
Peak age of Adolescence during growth spurt Adolescence
incidence
Local findings Pain, swelling & soft tissue mass Pain swelling & soft tissue mass
Systemic findings? Uncommon Fever, malaise, weight loss
Leukocytosis & ↑ ESR
Radiographic Periosteal reaction w/ sunburst appearance Periosteal reaction w/ onion skin
findings appearance
Metastases 15% at presentation most commonly to lungs 25% at presentation most commonly to
lungs
Dx made by Bx & MRI Bx & MRI

506. What are the most common bone tumors in adults?


a. Metastatic tumors to the bones rather than primary bone tumors in children
507. Fill out the following table comparing omphalocele & gastroschisis
Omphalocele Gastroschisis
Location of hernia
True hernia sac (peritoneal cover)
Association w/ other congenital
anomalies

Omphalocele Gastroschisis
Location of hernia Midline through the umbilical ring Right paraumbilical area
area
True hernia sac (peritoneal cover) YES, shiny thin membranous sac No, only bowel viscera herniates
Association w/ other congenital YES such as: No association w/ other
anomalies  Congenital heart defects TOF & malformations but increased risk for
ASD. bowel damage
 Beckwith-Wiedemann syndrome
 Trisomy 13

508. How are these treated?


a. Construct a silastic container called “silo” which during the subsequent days we slowly push in the
intestines into the abdomen.
b. In a baby w/ gastroschisis it is necessary to provide venous access for parenteral nutrition since the bowel
don’t function for a month
509. What is the most common cause of intestinal obstruction in the neonatal period?
a. Intestinal atresia which may occur in the small or large bowel presenting as bilious vomiting in a new
born w/ a KUB showing multiple gas bubbles in the GI tract.
510. A 74 yo man has sudden onset of severe tearing chest pain that radiates to the back & migrates
down shortly after its onset. BP is 220/110, he has unequal pulses in his upper extremities & a wide
mediastinum on CXR. EKG & cardiac enzymes are negative for cardiac infarction. What is the next step?
a. Tearing horrible chest pain in an old person w/ a wide mediastinum & HTN along w/ negative cardiac
enzymes & EKG suggests a dissecting aneurysm of the thoracic aorta. Must confirm the dissection w/
either a spiral CT scan (preferred), transesophageal echo (most invasive) or an MRI (takes mucho time)
b. In a trauma setting this would suggests a traumatic transection of the aorta
511. How would you proceed w/ treatment if the spiral CT scan is positive for a dissection of a thoracic
aortic aneurysm?
a. If the dissection is in the ascending aorta (Stanford A) we will proceed w/ emergency surgery
b. If the dissection is in the descending aorta (Stanford B) medical control in an ICU setting is preferred.
512. A 53 yo woman is in the ER c/o extremely severe frontal HA. The pain started an hour ago shortly
after she left the movie theater after watching all 3 lord of the rings. She reports seeing halos around
lights in the parking lot. On PE the right pupil is mid-dilated, does not react to light, & the right cornea is
cloudy w/ a greenish hue & the eye feels hard as a rock. How is this pt managed?
a. This is classic description of acute angle closure glaucoma. The fact that she spent some time in a dark
place where the pupils were dilated thereby closing off the trabecular mesh in the anterior chamber
resulting in accumulation of the fluid allowing the pressure to build up creating an emergency. The
greenish cornea w/ a hard eye is highly suggestive.
b. The first step is giving eye drops that will constrict (SNS antagonists or PNS agonists) the iris which will
promote draining of the fluid or diuretics such as mannitol or acetazolamide.
c. Commonly seen in Asia females around 40-50 yo
d. Constricted iris helps drain the fluid
513. A 34 yo man c/o severe abdominal pain that is sharp & unbearable. The pain is in the lower left
abdomen & radiates to the perineum. Pt denies dysuria. Pt reports N/V. On PE pt is afebrile, cannot sit
still & is constantly moving, pt’s abdomen is soft & non-tender on palpation. What is the next step in
management?
a. This is classic renal stone which will be confirmed w/ a UA, KUB & a non-contrast CT of the abdomen
(more sensitive).
b. US will miss small stones therefore CT is preferred
514. A 62 yo west Texas farmer of Scandinavian decent presents w/ a growing 1.5 cm indolent, waxy
skin mass w/ raised borders on his nasal bridge. There are no enlarged lymph nodes on head & neck.
What is the most likely dx?
a. This is classic description for skin cancer, specifically basal cell carcinoma due to the raised borders. BCC
does not metastasize therefore no involvement of lymph nodes
b. Melanoma is hyperpigmented, asymmetrical w/ irregular borders
c. SCC is described as a non-healing slowly enlarged ulcer w/ heaping borders
515. All BCC of the skin occur above ____ whereas all SCC of the skin occur below ____
a. All BCC occur above the mouth
b. All SCC occur below the mouth, SCC of the skin most commonly affects the lower lip.
516. What if the first step in confirming the dx of a skin cancer?
a. Full thickness skin bx at the edge of the lesion
517. What is the tx for BCC of the skin & how does it differ from SCC of the skin?
a. BCC of the skin requires excision w/ very small expenditure of skin around it, does not involve lymph
nodes
b. SCC of the skin is more aggressive & requires excision of a larger surrounding margin (0.5 cm) as well as
consider involvement of lymph nodes & lymph node dissection
518. What is the MCC of upper GI bleeds?
a. PUD most commonly 2/2 NSAID or H pylori
519. Fill out the following table comparing proximal vs distal small bowel obstruction
Proximal SBO Distal SBO
Vomiting
Abdominal distention

Proximal SBO Distal SBO


Vomiting Mucho No
Abdominal distention Minimal Significant

520. How are SBO diagnosed?


a. Abdominal plain films showing dilated loops of SI w/ air fluid levels proximal to the obstruction &
minimal gas in colon
b. Barium enema will r/o colonic obstruction
c. Upper GI series w/ follow-through of small amount of swallowed contrast to detect site of obstruction if
enema did not
521. How are these treated?
a. Both proximal & distal SBO result in severe dehydration which is the first step in tx. Specifically hypo-
Cl/K metabolic alkalosis due to vomiting & ↑ GI secretions proximal to the obstruction; correct w/ KCL
and 0.9% NS
b. Start w/ nonoperative management of NPO & NG suction to empty stomach
c. Abx
d. For constant pain & suspicion of strangulation must perform surgery
522. What would you suspect if the abdominal film shows air throughout the entire GI tract?
a. Ileus
523. How would you correct the metabolic alkalosis & the electrolyte abnormality?
a. Give KCl which will promote kidney secretion of K instead of H & will also correct the hypo-K/Cl
524. What is the most common type of melanoma?
a. Superficial spreading melanoma
525. What is used to determine how aggressive a melanoma is?
a. How deep/ thick it is since that suggests how invasive the melanoma is
526. What classification scheme is used for determining how invasive a melanoma is?
a. Breslow’s classification which uses millimeters of depth to classify melanomas
b. Clark’s level of classification was used in the past, Clark level >3 requires aggressive tx w/ excision &
chemotherapy
527. What is unique about invasive melanoma metastases?
a. Most cancers met to liver, lymph nodes, lung, brain & bones whereas invasive melanomas metastasize to
crazy stupid places where no other tumors will met to such as the wall of the duodenum or left ventricle.
528. What information in a vignette will suggest that you should perform embolectomy + fasciotomy
for a pt w/ sudden onset of calf pain & atrial fibrillation?
a. If there has been ischemia for > 8 hrs since there is high risk for compartment syndrome
529. A newborn is noted to have a moist medallion of mucosa occupying the lower abdominal wall
above the pubis & below the umbilicus. It is clear that urine is constantly draining from this anomaly.
What is the dx & tx?
a. This is extrophy of the urinary bladder where the pubis symphysis didn’t close.
b. Tx requires immediate surgery within the first days of life.
530. What is the most common type of breast cancer?
a. Infiltrating ductal carcinoma
531. How are infiltrating ductal carcinomas managed?
a. Lumpectomy or modified radical mastectomy + axillary sampling + post-operative radiation therapy
(unless pt is pregnant)
532. Almost all variants of breast cancers are managed the same w/ the exception of……(describe how
each is managed differently)
a. Inflammatory cancer of the breast requires pre-op radiation & neoadjuvant chemotherapy
b. In situ ductal carcinoma which is managed differently since if it is in situ does not have metastatic
potential therefore no axillary sampling is needed
533. What determines whether or not a pt w/ surgical resection of a breast cancer will be treated for
metastatic dz?
a. Any pt that has positive axillary nodes needs systemic therapy.
534. What is used to determine whether the pt will receive chemotherapy vs. hormonal therapy?
a. In premenopausal women we prefer to use chemotherapy
b. In post-menopausal women we prefer to use hormonal therapy
c. Hormonal therapy is given to any pts whose cancer is positive for estrogen or progesterone receptor
d. Any pt w/ distant metastases requires chemotherapy
535. 18 yo male c/o dull aching & fullness of the scrotum. PE shows soft left-sided scrotal swelling.
Transillumination test is negative. The scrotal swelling increases when the pt performs a Valsalva
maneuver but otherwise the PE is unremarkable. What is the most likely dx?
a. Varicocele which is a dilatation of the pampiniform plexus which would feel like a “bag of worms” on PE
b. A hydrocele which is fluid in the tunica vaginalis or a spermatocele will trans illuminate on PE
536. A 45 yo man c/o right shoulder pain & weakness after falling on an outstretched hand. When his
arms are passively abducted over his head & he is asked to slowly bring them down his right arm drops
rapidly at the midpoint of its descent. What is going on?
a. This is suggestive of a rotator cuff tear
537. What is the MCC of a winged scapula?
a. Iatrogenic injury to the long thoracic nerve (C5,6,7) during axillary lymphadenectomy
538. What are the two major brachial plexus injuries that may occur most commonly due to birth
trauma?
a. Erb’s palsy & Klumpke’s palsy
539. Fill out the following table comparing the two
Erbs palsy Klumpkes palsy
Nerve roots affected
Which is more common?
Clinical features

Erbs palsy Klumpkes palsy


Nerve roots affected C5-C6 Upper brachial plexus C7-C8, lower brachial plexus
Which is more common? More common Less common
Clinical features  “Waiter’s Tip”  Claw hand due to unopposed
 Flaccid arm held internal finger flexion & ↓ ability to
rotation w/ the elbow extended extend the elbow or flex the
& forearm pronated w/ wrist & wrist
fingers held in flexed position  Horner’s syndrome = ipsilateral
 Asymmetric Moro reflex ptosis, miosis & anhydrosis if
SNS fibers are involves

540. What is the classic sign suggesting rupture of the tendon of the long head of the biceps?
a. Popeye sign where the biceps muscle belly becomes prominent in the mid upper arm
541. What is the MCC of nosocomial bactermemia?
a. Coagulase neg staph such as staph epidermidis or saprophyticus
542. A 9 mo old chubby healthy looking boy has episodes of colicky abdominal pain that make him
double up & cry. The pain lasts up to one minute & is happy between the episodes. PE shows a vague
mass on the right side of the abdomen w/ bloody thick stools. What is the next step in management?
a. Currant jelly stool w/ colicky abdominal pain & an abdominal mass suggests intussusception which will
be managed w/ a barium or air enema that is both diagnostic & therapeutic
543. A 6 mo old is found to have occasional stridor & respiratory distress in which he assumes a
hyperextended position. The mother also notes that he has difficulty swallowing.
a. If the only sx were inspiratory distress/wheezing it would likely be tracheomalacia but since this pt also
has difficulty swallowing you must think of a vascular ring which is due to failure of the second fetal
aorta to degenerate.
544. How would you confirm the dx of a vascular ring?
a. Barium swallow which will show extrinsic compression of the esophagus
b. Bronchoscopy which rule out tracheomalacia which is normally diffuse & show the extrinsic
compression
545. What is the tx?
a. Surgical resection of the second vascular ring.
546. A 7 mo pregnant lady suddenly bleeds into the abdomen & goes into shock. What likely
happened?
a. This is likely rupture of a small hepatic or splenic aneurysm which happens most often during pregnancy
& late in the pregnancy
547. What is the MCC of ex-lap during pregnancy?
a. appendicitis
548. A child falls on an outstretched hand. This results in what type of fx?
a. Supracondylar fx of the humerus
549. How does this fx differ in children from adults?
a. In children we must monitor the vascular supply of that extremity w/ distal pulses, capillary filling or
Doppler studies since is a high incidence of vascular compromise
550. A 74 yo man w/ 3 mo h/o weight loss presents w/ lower back pain. You suspect cancer that
metastasized to his bones. What imaging modality will you order?
a. Radio nucleotide bone scan is the most sensitive test for bone metastases & it is therefore used for dx of
cancer in bones since CT scan & radiographs will not pick up this early in the dz
b. Although this test is very sensitive it is not very specific & positive results may be due to inflammation such
as arthritis or other causes therefore f/u on positive bone scan results w/ radiographs to check for arthritis
or other causes. If bone scan is negative then you can r/o cancer in the bones
551. A 55 yo HIV positive man has a fungating mass growing out of the anus & rock hard, large lymph
nodes on both sides of the groin. What is the most likely dx?
a. This is classic SCC rectal cancer. Unlike adenocarcinoma of the rectum, SCC metastasizes to both internal
abdominal lymph nodes as well as the groin. SCC is also more commonly seen in HIV positive
homosexuals.
552. What is the next step in management?
a. Bx to confirm the dx
553. Bx comes back positive for SCC what is the next step in therapy?
a. Neoadjuvant chemotherapy to reduce the size of the cancer since this is likely very extensive, once it is
smaller you are able to surgically resect the cancer.
554. T/F as a rule if a child suffers a fx through the growth plate an open reduction is required
a. True! If the growth place is in one piece then it can be pushed into place but if the growth plate is broken
in two pieces as a rule you need an open reduction & internal fixation in the OR.
555. What is the difference between the following two vignettes & how would you tx each?
a. A 44 yo woman is recovering from an episode of acute ascending cholangitis 2/2 to
choledocholithiasis. She develops fever & leukocytosis as well as some tenderness in the RUQ. US
reveal a liver abscess.
b. A 29 yo migrant worked develops fever & leukocytosis as well as RUQ tenderness; He has mild
jaundice & elevated alk phos. US of the liver show a normal biliary tree & a liver abscess.
Answer
c. There are 2 kinds of liver abscesses
i. Pyogenic liver abscess which is invariably a complication of biliary tract dz & it needs to be
drained
ii. This is an example of an amebic abscess which does not need to be drained but rather may be
treated w/ metronidazole; drainage will show anchovy paste pus.
iii. Do not try to drain some of the pus & send to culture because there are no ameba in the pus but
rather it is growing in the wall
556. Is it more common for GI bleeding to be from the upper or lower GI tract?
a. 75% is upper GI bleeding like the nose, throat, stomach or esophagus
b. If person is <40 yo there is overwhelming odds that the pt has an upper GI bleed
c. By virtue of being an old person the bleed may be from anywhere in the GI tract
557. Where is the most common lower GI tract bleeding?
a. The majority of lower GI tract bleeding is from the colon, very few are from the SI
558. T/F if a pt is vomiting blood it 100% means that the pt has an upper GI bleed
a. True! If a pt is vomiting blood it has to be an upper GI bleed
559. What is the first step in managing a pt that is vomiting blood?
a. Endoscopy to find the source & then stop the bleeding
560. A 54 yo black man w/ hx of smoking & drinking describes progressive dysphagia that began 3 mo
ago w/ difficulty swallowing meats, then other foods & now liquids. He points to where the food sticks.
What is the likely dx?
a. This is classic description of squamous cell carcinoma of the esophagus which is most common in black
men who smoke & drink.
561. What is the next step in the diagnostic workup?
a. The next step in the diagnostic workup is barium swallow not endoscopy!!! The reason for this is to
create a roadmap preceding the endoscopy thereby preventing accidental perforation of the esophagus
b. If the pt had a long h/o GERD it is more likely to be adenocarcinoma
562. A 14 yo presents to your clinic w/ obvious scoliosis. When she is asked to bend down you see her
right hemithorax protruding. Her Cobb angle is 35 degrees & she asks you whether or not she needs
surgery to prevent the progression of her scoliosis. She tells you she had menarche at age 11. What is
your answer?
a. Progression of scoliosis occurs only during growth or if the spinal curvature is >50 degrees. In
females growth ceases within 6 months of menarche therefore there should not be any more
progression.
b. 80% of growth occurs by the onset of menarche therefore if she still had not had menarche more aggressive
tx is needed
563. Explain how Cobb’s angle is used to decide treatment plan for a pt w/ scoliosis during growth
a. For 10-20° of scoliosis a f/u scoliosis film is obtained in 4-6 mo & assessed for progression. If ↑ >5° then
refer to an orthopedist.
b. For 20-40° bracing is indicated
c. For >40° of scoliosis surgery is indicated
d. After growth has concluded surgery is considered if scoliosis is >50°
564. 67 yo diabetic has an indolent un-healing ulcer at the heel of the foot. Why are these ulcers
common to this group of pts?
a. Diabetics have a high risk for chronic leg ulcers that do not heal due to PVD & neuropathy. These ulcers
tend to be found on pressure points such as the heel or the head of the first metatarsal.
565. Describe the pt that has an ulcer in the distal tip of the toes?
a. Male pts w/ cardiovascular/atherosclerotic occlusive dz who present w/ an ulcer on a distal tip of a toe
on a foot that lacks a pulse. This is an ischemic ulcer due to atherosclerotic dz. This may be due to
disease that affects the major vessels as opposed to small vessel dz
b. Large vessel dz is amendable to surgical therapy using bypass the occluded vessel
566. How do we determine whether or not a pt will benefit from the bypass?
a. Ultrasound (Doppler) to determine whether or not there is a pressure gradient along the major
vessels in that extremity. If a pressure gradient is found (ex; BP of 140 mmHg is in the arm, 140 mmHg in
the thigh & 70 mmHg in the foot) it suggests that a major vessel is occluded & he will benefit from a
bypass.
b. F/u on positive results w/ an arteriogram before bypass.
567. A 33 yo man has had 3 large bowel movements that he describes are made up entirely of dark red
blood. The last one was 20 mins ago. He is diaphoretic, pale w/ a HR of 110 bpm & a BP of 90/60. What
is the next step in managing this pt?
a. Although the pt is young & therefore the most likely site of bleeding is upper GI bleeding you should get
the first step is NOT endoscopy but rather placing a NG tube & suction. If we suction out blood we
confirm an upper GI bleed which will be followed w/ an endoscopy. The reason this was not melena
although it is an upper GI bleed is because when there is copious amount of blood it can be irritating to
the GI tract  faster transit time  less digestion of the blood.
568. Pt c/o sudden onset of severe abdominal pain. Pt is lying very still & is guarding the abdomen.
Radiograph shows air under the diaphragm. What is going on?
a. Sudden onset of severe abdominal pain w/ free air under the diaphragm is suggestive of perforation
which requires emergency laparotomy
569. What is the MCC cancer in the liver?
a. GI cancers that metastasize to the liver
570. What are the three main etiologies of jaundice & describe what labs characterize each
a. Hemolytic jaundice characterized by hyper-unconjugated/indirect bilirubinemia w/ normal LFTs
b. Hepatocellular jaundice due to lysis & inflammation for the hepatocytes characterized by elevated of
both conjugated & unconjugated bilirubin as well as ↑ ALT & AST
c. Biliary obstruction jaundice characterized by hyper-conjugated/direct-bilirubinemia & ↑↑ alk phos (most
likely on test since this requires surgical intervention)
571. What is the first step in working up a pt w/ jaundice & direct hyperbilirubinemia?
a. US of the biliary tree is the first step, if positive it will show dilated intra & maybe extra-hepatic biliary
ducts.
572. What would you think if the US showed a big distended thin wall gallbladder?
a. It is highly suggestive that the obstruction jaundice is due to malignancy of either the CBD, head of the
pancreas or ampulla of Vater
b. The fact that there are no gallstones excludes it as the etiology
573. A 14 yo woman has a firm rubbery moveable mass in her right breast that was first noticed one
year ago & has since grown to 6 cm in diameter. What is the next step in management?
a. This is most likely a fibroadenoma which are classically firm, rubbery moveable masses seen in young
women. This specifically is a giant juvenile fibroadenoma which grow very quickly. Next step is
confirmation w/ FNA or sonogram
b. Do not do a mammogram in pts < 20 yo
574. What is the tx?
a. Although normal fibroadenomas do not necessarily need to be taken out the giant ones need to be
excised.
575. A 40 yo man who has been suffering from UC for the past 22 years has had at least 40
hospitalizations for exacerbation. The pt weighs 90 lbs & c/o of abdominal pain, fever, & a very
distended transverse colon on KUB. Due to a recent relapse he has been placed on high dose steroids to
induce remission. What is the next step in management?
a. This is a classic presentation of a pt w/ all the indications for colon resection due to a long standing h/o
UC, severe malnutrition & toxic megacolon. The surgical resection of the GI tract from the rectum to the
cecum is curative (CD does not have a cure)
576. An adult w/ 20% burns presents to the ER. At what rate should you begin the infusion in this
adult?
a. In the adult w/ at least 20% burn begin w/ at least 1000cc/hr which we will continue by monitoring
urinary output & adjust accordingly
b. If asked at what rate begin w/ 1000cc/hr but if they want us to estimate use the formula
577. What is the normal hour urinary output in a 70kg man?
a. At least 1cc/kg/hr in adults therefore a normal 70kg adult should produce at least 70cc/hr
b. Increase UOP to 2cc/kg/hr if we try to protect the kidneys
578. What should you worry about in a pt that was electrocuted while trying to hook up a TV; he has a
small burn hole on his thigh & a small hole in the bottom of the foot.
a. Electrical burns are far more extensive than they actually appear & all the tissue inside his thigh is
probably burned which will require extensive debridement & probably amputation.
b. Monitor for the massive release of myoglobin & K from necrosis of tissue.
c. These pts also tend to present w/ posterior dislocation of shoulders & compression of vertebral bodies
due to massive contraction of muscles therefore possibly get radiographs
579. What are some sequelae of electrical burns?
a. Cataracts & demyelination of nerves
580. Fill out the following table
First degree Second degree Second degree deep Third degree
superficial partial partial thickness
thickness
Layers involved
Color
Wet or dry?
Pain?
Forms a scar?
Management

First degree Second degree superficial Second degree deep Third degree
partial thickness partial thickness
Layers Epidermis only Epidermis & superficial Epidermis & lower Complete destruction
involved dermis dermis of epidermis, dermis
& portion of
subcutaneous tissue
Color Red red Pale white White
Wet or dry? Moist Moist Dry & leathery
Pain? Yes Yes Yes NO SIR, nerve
endings are burned
Forms a scar? No No, forms a blister Yes Yes
Management Moisturizers & Analgesics (opiates) Analgesics (opiates) Skin grafting &
analgesics debridement of dead skin debridement of dead hydrotherapy
Do NOT remove formed skin Escharotomy may be
bullae, only remove bullae if Do NOT remove needed (surgical
ruptured formed bullae, only removal of
remove bullae if constricting scars)
ruptured

581. Hospitalization is required for partial thickness burns > __ % BSA & full thickness burns > __ %
BSA
a. Hospitalization is required for partial thickness burns > 10% of BSA & full thickness burns > 2% of BSA
582. After appropriate fluid resuscitation, how are burns managed?
a. Tetanus vaccination depending on pts vaccination status
b. Suitable cleaning & debridement
c. Silver-sulfadiazine (INN or silvadene) is a soothing white paste that protects very well against bacteria
& is first line therapy in burn pts
i. If we desire deep penetration of the topical, mafenide acetate has deep penetration & used on
areas w/ thick eschars but it hurts & may produce acidosis
ii. Burns near the eye are covered w/ triple abx ointment since silver-sulfadiazine is irritating to the
eye
d. After 2 weeks graft may be placed on areas that did not heal in extensive burns
e. Intensive nutritional support, preferably via the GI tract, w/ high calorie & high nitrogen since these pts
are in a tremendous catabolic state
f. Rehab begins on day one
583. 12 hrs after completion of multiple surgeries for blunt trauma in an average sized adult. Urinary
output is recorded in 3 consecutive hrs as 12 cc, 17 cc, & 9 cc. BP is 129/83 mmHg. What is on the ddx &
how would you differentiate them?
a. This pt has oliguria but we can tell he is not in shock due to the BP therefore the two most likely causes
are either dehydration (we are behind on his fluids) or acute renal failure.
b. We can differentiate the two by measuring UOP after administration of a bolus of IV fluids (500 cc
over 15 mins). If pt is dehydrated then the UOP will increase whereas if it is due to ARF then there
will not be a UOP. Another method (more likely on test) is to measure urinary Na concentration. If it is
due to dehydration the kidney attempts to reabsorb all Na therefore urinary Na will be <20
whereas in ARF the Na in the urine is >40.
c. Lastly we can calculate the FeNa which in ARF it will be >1%
584. 4 days post exploratory laparotomy for blunt abdominal trauma w/ resection & re-anastomosis
of damaged small bowel a pt has abdominal distention without abdominal pain. He has no bowel sounds
& has not passed flatulence. Radiographs show dilated loops of small bowel w/ air fluid levels. What is
happening here?
a. This is an example of paralytic ileus in an otherwise healthy pt, w/o pain who has not passed gas
585. What laboratory test would you want to perform?
a. Serum K determination since hypokalemia may perpetuate paralytic ileus
586. What would you assume would be the dx if this occurred one week post-surgery & the pt is
distended w/ no passing of gas?
a. More likely to be a mechanical intestinal obstruction 7 days post-op which is most commonly due to
adhesions or suture lines
587. How is the dx for mechanical intestinal obstruction?
a. Barium tag which is injection of a small amount of barium which injected through the NG tube which will
end up in the colon as long as there is no obstruction. Therefore take serial x-rays & make sure it does
not pool anywhere suggesting obstruction.
588. A 54 yo male is undergoing pre-operative evaluation for an elective repair of a large AAA has a
h/o severe progressive angina. What should you do next?
a. Must r/o & evaluate CAD & coronary revascularization w/ echocardiograph & stress test before the
elective surgery
589. A 40 yo obese woman has an indolent un-healing ulcer above her right medial malleolus. The
skin above it is thick & hyperpigmented & she has frequent episodes of cellulitis & varicose veins. What
is the cause of this ulcer?
a. This is classic for a venous stasis ulcer that is 2/2 to venous HTN & is always above the medial
malleolus in a pt w/ hyperpigmented & thickened skin.
590. How are these pts managed?
a. Compression stockings or special boots that help circulate the blood in the superficial venous veins.
591. Is the entry or exit wound bigger in diameter in gunshot wounds?
a. The exit wound is always bigger
592. A 32 yo male presents to the ER w/ c/o “breaking his dick” during intercourse. His penile shaft
has obvious hematoma w/ a normal appearing glans. What do you do?
a. This is a fracture of the penis which occurs to an erect penis suffering trauma (woman on top). First I
would get a retrograde urethrogram to insure no urethral injury.
b. There is rupture of the corpora cavernosa & tunica albuginea which are a surgical emergency since
impotence will ensue as arteriovenous shunts develop.
593. What is used to differentiate transudative from exudative pleural fluid?
a. Exudative fluid has at least one of the following:
i. Pleural protein/serum protein is >0.5
ii. Pleural LDH/ serum LDH is >0.6
iii. LDH > 2/3 the upper limit of normal serum LDH
594. What do you suspect if the pleural fluid has…..
1. Elevated fluid amylase?
2. Milk appearance
3. Bloody effusion
4. Purulent fluid
5. Exudative effusion that is primarily lymphocytic
ii. answers
1. Ruptured esophagus, pancreatitis or malignancy is suggested by presence of amylase
2. Milky appearance is a chylothorax which suggests lymph
3. Blood suggests malignancy
4. Purulent fluid suggests empyema (infection)
5. Lymphocytic effusion suggests TB
595. A 72 yo man w/ atrial fibrillation develops an acute abdomen that began 2 days ago. He has a
silent abdomen w/ diffuse tenderness & mild rebound. He has acidosis & he looks quite sick. He has a
trace of blood in the rectal exam. Radiographs shows distended small bowel until the middle of the
transverse colon. What is the likely dx?
a. Whenever you see acute abdomen in a very old pt it is either volvulus or mesenteric ischemia. Since this
pt has atrial fibrillation the most likely dx is mesenteric ischemia. Mesenteric ischemia is unique in its
presentation since it presents as acute abdomen w/ only traces of blood in the stool. The radiograph
showing distention up to the middle of the transverse colon is classic since that is the distribution of the
SMA.
596. What would be the first step if this pt c/o abdominal pain starting 30 mins ago?
a. Call vascular surgery to perform an arteriogram which will demonstrate an occluded SMA which may be
opened.
597. What is the most common site for colonic volvulus?
a. Sigmoid colon
598. How is volvulus diagnosed?
a. Plain abdominal radiographs
i. Sigmoid volvulus shows the omega loop sign indicated a dilated sigmoid colon
ii. Cecal volvulus shows a coffee bean sign indicating a large air fluid level in RLQ
b. Sigmoidoscopy is the preferred diagnostic & therapeutic test for sigmoid volvulus
c. Barium enema reveals bird beak which is the narrowing of the colon at site of volvulus; do not perform
a barium enema if strangulation is suspected!!
599. How is volvulus treated?
a. Sigmoid volvulus treated & diagnosed w/ sigmoidoscopy
b. Cecal volvulus requires emergency surgery.
600. What can be used to assess hepatic functional reserve in pts w/ cirrhosis?
a. Child’s classification estimates hepatic reserve in liver failure. It is used as a measure of disease severity
& a predictor of morbidity & mortality. It considers (A-BEAP) Ascites, Bilirubin, Encephalopathy,
Albumin levels & PT to calculate the prognosis.
601. What are the 2 MCC of cirrhosis in adults?
a. Alcoholic liver dz is the MCC of cirrhosis & chronic Hep B & C are the next most common causes
602. What is the diagnostic test for cirrhosis?
a. Liver bx
603. Name all the complications associated w/ liver cirrhosis
a. AC 9 H: Ascites, coagulopathy, Hyper-bilirubinemia, hyperammonemia, Hepatorenal dz, HCC,
hyperestrinism, hepatic encephalopathy, hypoalbuminemia, portal HTN, hypoglycemia
b. Portal HTN  esophageal & gastric varices which may  bleeding
c. Ascites due to portal HTN & hypoalbuminemia
d. Hepatic encephalopathy due to accumulation of ammonia & other toxic metabolites that are normally
removed by the liver
e. Hepatorenal syndrome which indicates end stage liver dz. This is where there is progressive renal
failure secondary to renal hypoperfusion resulting from vasoconstriction of renal vessels  functional
renal failure where the kidneys are normal clinical features are azotemia, hypotension, oliguria,
hyponatremia, low urine sodium]
f. Spontaneous bacterial peritonitis since the ascitic fluid is a great culture
g. Hyperestrinism which means hyper estrogenic hormone resulting in gynecomastia, testicular atrophy,
palmar erythema & spider angiomas
h. Coagulaopathy treated w/ FFP
i. HCC (elevated AFP)
604. What is used to confirm that ascites is indeed due to portal HTN rather than other etiologies?
a. Serum ascites albumin gradient. If it is > 1.1 g/dL is it likely due to portal HTN but if < 1.1 g/dL portal
HTN is unlikely
b. SAAG = (serum albumin) – (ascites albumin)
605. What is the tx for portal HTN?
a. TIPS – transjugular intrahepatic portal shunt which lowers portal pressure
b. Also available are porto-systemic shunts, octeride
606. Which is more common, esophageal or gastric varices?
a. Esophageal account for 90% of varices
607. What PE findings are common in pts w/ hepatic encephalopathy?
a. Asterixis which is a flapping tremor when pts have their arms extended & dorsiflexed
b. Hyperreflexia
c. Rigidity
d. Fetor hepaticus which is a musty odor of breath
608. What is the tx for hepatic encephalopathy?
a. Lactulose which prevents absorption of ammonia since metabolism by gut flora of lactulose favors
formation of NH4+ which is poorly absorbed thereby promoting excretion of ammonia
b. Neomycin which kills gut flora thereby decreasing ammonia production
c. Protein limited diet w/ up to 40 mg/day
609. What bacteria is MCC of SBP in pts w/ liver failure?
a. E. coli is MCC > Klebsiella & pneumococcus
610. How are pts w/ cirrhosis monitored?
a. Periodic lab testing every 3 months of LFT, renal function test, CBC, electrolytes & coagulation tests
b. Endoscopy to determine presence of esophageal varices
c. If HCC is suspected perform a bx
611. What is used in tx of bleeding esophageal varices?
a. Variceal ligation & banding which is the initial endoscopic treatment of choice; endoscopy is initial tx for
all pts vomiting blood
b. Endoscopic sclerotherpay
c. IV Octreotide causes splanchnic vasoconstriction reducing portal pressure
i. Second line IV vasopressin & NO vasoconstriction of mesenteric vessels dec portal pressure;
Nitro is given to reduce the effects of vasopressin on the coronary system
d. TIPS as long term management
612. T/F abstinence from alcohol for more than 2 years is required before a pt is eligible for
transplant
a. False! Abstinence for more than 6 month
613. A 42 yo woman drops her hot iron on her lap while doing laundry. She comes in w/ a shape of the
iron delineated on her upper thigh. The area is white, dry, leathery, & does not hurt. How should you tx
this pt?
a. For small area third degree burns we can excise the dead skin & place a skin graft. If this was more
extensive then we have to hospitalize the pt & wait 2-3 weeks until areas of skin do NOT heal & then
place skin graft
b. Only do immediate skin grafting for small third degree burns
614. 46 yo male after MVA presents w/ pelvic fx, blood at the meatus, rectal exam shows high riding
prostate. What is the dx & how would you confirm it?
a. Most likely posterior urethra injury dx w/ a retrograde urethrogram….AVOID placing a transurethral
Foley
615. How does the management for a posterior urethral injury compare to anterior?
a. Anterior urethral injury are immediately repaired whereas posterior surgery may be deferred 4 months
b. While waiting for surgery place a suprapubic catheter
616. What is the tx for a pt being tx for pneumonia that develops a purulent pleural effusion w/ a pH of
7.1?
a. The purulence suggests empyema & the pH<7.2 suggest that it is parapneumonic effusion therefore the
pt needs to get chest tubes placed to drain the fluid. Parapneumonic effusion is a pleural effusion that
arises as a result of pneumonia
b. Intrapleural injection of thrombolytic agents may help accelerate drainage
617. What must you always do after performing thoracocentesis?
a. Obtain a CXR following thoracocentesis or central line placement in order to ensure that you did not
cause a pneumothorax.
618. What is the cause of spontaneous pneumothorax?
a. Spontaneous rupture of pleural blebs (air filled sacs on the lung, usually in the apex) resulting in air
rushing into the pleural space  collapse
619. What is the first line tx for small asx spontaneous pneumothorax?
a. Observation only since most resolve in 10 days
b. If pneumothorax is large or pt is sx then administer oxygen since supplemental oxygen hastens the
resorption of air in the pleural space & chest tube placement
620. What is the classic pt w/ spontaneous pneumothorax?
a. Tall thin young men
621. T/F there is a high recurrence rate in these pts
a. True! Recurrence rate is 50% in 2 years
622. What are the two most common presentations of retinoblastoma?
a. Leukocoria & strabismus
623. What is the hallmark finding on imaging that is suggestive of retinoblastoma?
a. Calcification within the tumor which is identified on imaging studies is the hallmark of
retinoblastoma
624. A 72 yo c/o 1 week h/o bilateral leg pain when ambulating but pt denies any trauma. Pt also
admits that he has been unable to have an erection for the previous week even though using Viagra. On
PE you notice diminished bilateral lower pulses. What is going on?
a. This is an example of Leriche’s syndrome which is a triad of bilateral claudication, impotence &
absent/diminished femoral pulses due to an atheromatous occlusion of the distal aorta right above the
bifurcation of the iliacs.
625. A man was rescued from a burning building. His face & mouth appear burned. What is the next
step in managing this pt?
a. Must consider a respiratory burn due to inhalation of smoke which will compromise the airway
therefore intubate this pt to ensure airway
626. How would you confirm the respiratory burn?
a. Fiberoptic bronchoscopy
627. How do you judge the need for therapy or monitor the pt?
a. Blood gases….remember that these pts also breathed carbon monoxide therefore monitor
carboxyhemoglobin by sending blood
b. High CO levels create false high O2 sat readings
628. What is the tx for high levels of carboxyhemoglobin?
a. 100% oxygen
629. At what rate should you start the fluid resuscitation?
a. 1000 cc/hr & monitor UOP
630. What is the first step in trying to diagnose peripheral arterial disease (PAD)?
a. Ankle-to-brachial BP index…if <.9 then it suggests PAD
631. What do you do if a pt w/ classic description PAD but has a normal ABI?
a. Repeat ABI post exercise
b. If it is still normal it is suggestive of small vessel dz
632. How is PAD tx?
a. Aspirin & blood thinners
633. What is RQ measuring?
a. Resting quotient is the steady state ratio of CO2 produced to oxygen consumed per unit of time.
634. What does an RQ close to 1 suggest?
a. Carbohydrate excess in diet since carbs produce an RQ of 1
635. What is the RQ of protein & fats
a. RQ of fats is 0.7
b. RQ of protein is 0.8
636. What is deficient in pts w/ Wilson’s dz?
a. Ceruloplasmin which is the copper-binding protein necessary for copper excretion. This results in
copper accumulation in the liver & once the hepatocytes die they release the copper into systemic
circulation where they accumulate in the kidneys, basal ganglia & cornea
637. What are some CNS findings associated w/ Wilson’s dz?
a. EPS such as parkinsonian sx, chorea, drooling & ataxia
b. Psychiatric disturbances like depression, neuroses & personality changes
638. How is Wilson dz treated?
a. D-penicillamine which removes & detoxifies excess copper
b. Zinc which prevents uptake of dietary copper
c. Liver transplant for unresponsive pts
639. Pt was found to have a popliteal aneurysm what else should you check for?
a. 50% of pts w/ popliteal aneurysms have AAA
b. Popliteal aneursms are most common peripheral artery aneurysm. If pt is symptomatic they req surgery
640. A 53 yo T2DM presents w/ a 4 mo h/o leg pain when he walks. He states that it feels like his
calves cramp after he walks 50 meters or so & the pain improves w/ rest. He denies any rest pain. On PE
you notice that there is hair loss & thickened toenails as well as decreased pulses in his lower
extremities. What is the likely dx?
a. Peripheral vascular dz which is common in diabetics & smokers. This pt has intermittent claudication
which is reliably reproduced by the same walking distance & is relieved by stress which is classic for it
b. Femoral or popliteal dz causes calf claudication
c. Aortoiliac occlusive dz causes buttock & hip claudication & impotence in addition to calves.
641. How is the diagnosis of PVD confirmed?
a. Ankle-to-brachial index (ABI) which is the ratio of the SBP at the ankle to SBP at the arm
i. Normal AB >1.0
ii. Claudication <0.7
iii. Rest pain ABI < 0.4
b. Pulse volume recordings
c. Arteriography is the gold standard but it is only done if revascularization surgery is considered
642. What can cause a false ABI reading?
a. Pts w/ calcified arteries, especially those w/ DM, have false ABI since the vessels are not compressible
643. What is the tx for PVD?
a. Smoking cessation!!!
b. Exercise program where they walk to point of claudication, rest & continue walking for another cycle
c. Foot care
d. Control of hyperlipidemia, HTN, weight, DM to reduce atherosclerotic RF
e. Aspirin
644. What are the indications for surgical tx of PVD?
a. Resting pain
b. Ischemic ulceration w/ tissue necrosis
c. Severe sx refractory to medical tx
645. What is the surgical tx for PVD?
a. Bypass grafting has an immediate success rate of 90%
b. Angioplasty
646. Pt suffered 3rd degree burns on both arms, no burns on face so no suspicion for respiratory burn.
The burns have a circumferential, white leathery appearance. What complication do you need to
consider?
a. Pts w/ burns develop edema & since this pt has circumferential, non-pliable leathery skin it may result in
cutting off of circulation therefore you must monitor for distal pulses w/ ultrasound. If there is
significant decrease in blood flow then you should do escherotomy, will not require sedation since these
pts do not have pain fibers & no need for sterilization since its already not sterile.
b. This is what you worry about w/ circumferential burns
647. A 33 yo male presents after MVA w/ pelvic fx a Foley is placed & meets resistance. What is the
next step?
a. Take out Foley & perform a retrograde cystogram to evaluate urethral/bladder injury
b. Take radiographs of bladder w/ dye as well as post-void to see posterior extra peritoneal dye that would
be occluded w/ a bladder full of dye
648. Eisenmenger’s reaction?
a. This is a late complication in a minority of pts w/ pulmonary HTN resulting in a R  L shunt presenting
as cyanosis, DOE, SOB & chest pain
649. Define electrical alternans & what it suggests.
a. Electrical alternans refers to alternate beat variation in the direction of the ECG waveforms due to the
heart swinging like a pendulum within the pericardial space/fluid creating a motion artifact

650. What are the classic PE findings in cardiac tamponade?


a. Beck’s triad of hypotension, muffled heart sounds & JVD
b. Pulsus paradoxus which is a drop >10 mmHg in BP during inspiration is pathognomonic
651. What is used to confirm the dx of cardiac tamponade?
a. echocardiogram
652. A boy is bit by his neighbor’s dog when the kid was messing w/ him while the dog was eating.
How do you manage the kid & the dog?
a. The boys bite is irrigated well & then may be given Augmentin to prevent pasturella or campynophagia
(asplenia pts) infections
b. Since the bite was thought to be provoked then the dog may just be monitored for signs of rabies & at the
smallest indication of rabies he will have to be put down & brain inspection for rabies
c. In wild animals we have to put them down & check for rabies
653. When do we give pts rabies prophylaxis?
a. Bat bites or wild animal bites
b. This includes rabies vaccination & rabies immunoglobulin
654. A 64 yo male presents w/ sudden onset of pain in his right lower leg. On PE you cannot feel any
pulse on his right leg & the leg appears pale. Pt reports tingling & difficulty moving the leg. What is the
likely dx?
a. Acute artery occlusion of the femoral artery
655. What is the most common site of acute artery occlusion?
a. Femoral artery most commonly from an emboli
656. What is the most common source of emboli
a. The heart accounts for 85% of emboli w/ AFib being the MCC of emboli
657. How would you confirm the dx in this pt?
a. ECHO to look for MI or AFib
b. Arteriogram to define site of occlusion
658. How would you tx this pt?
a. IV Heparin & emergency surgical embolectomy via cutdown & Fogarty balloon
659. When is surgery indicated for closure of an ASD or VSD?
a. When pts are symptomatic or when pulmonary to systemic flow ratio is greater than 1.5 : 1
660. Pt arrives to hospital one hour after being bit by rattle snake. No local pain or swelling around
the bite marks in the leg. How do you manage this pt?
a. Up to 1/3 of all bites from venomous snakes do not contain venom. Pain, swelling & discoloration should
take place within 30 minutes therefore this pt only needs his wound cleaned, tetanus prophylaxis &
observation since we suspect he was not envenomed
661. What if the leg did in fact swell & change color?
a. This man needs tx therefore give him the specific anti-venom (a lot of it at least 5 viles) for the snake &
get typing & cross-match since the venom will later interfere w/ this & if there are later complications
we want the appropriate blood.
b. DO NOT cut or excise the area, no need to perform fasciotomy/steroids or elevation all you need is the anti-
venom
662. T/F smaller pts require MORE anti-venom
a. True!!!
663. The “square root sign” describing a rapid y descent in the jugular vein is suggestive of….
a. Constrictive pericarditis
664. A 44 yo woman breaks her arm when trying to pick up her groceries. What do you suspect?
a. This is a pathological fx therefore I suspect metastatic cancer to the bones radiograph is likely to show a
lytic bone lesion
b. Most likely from breast in women
665. What would be the most likely origin of the metastasis if pt was a male?
a. Lung cancer…not prostate
b. Prostate bone mets result in blastic lesions not lytic
666. A 64 yo male presents w/ a 2 day h/o fever & malaise. He is currently taking beta blockers, ACE
inhibitors, aspirin & statins since he suffered a MI 2 month ago. His CBC shows leukocytosis & on PE you
hear a friction rub. What is the likely dx?
a. This is an example of Dressel’s syndrome, also known as post myocardial infarction syndrome, which is
an immunological syndrome that presents weeks to months post-MI as fever, malaise, leukocytosis &
pericarditis
667. What is the most effective tx for this pt?
a. High dose aspirin…..not the baby aspirin he is taking post MI but real dose aspirin.
668. A 3 yo develops microscopic hematuria after falling from his tricycle. Are you worried?
a. Yes! Because this is too small of a trauma to cause hematuria therefore this suggests an underlying
congenital urological anomaly
669. T/F radiographs of bones always require 2 views
a. True! Always 2 views that are 90 degrees to each other including both the joint above & below
670. A 40 yo man who has a h/o chronic draining sinus in his lower leg after he had an episode of
osteomyelitis when he was 12. In the last few months he has developed an indolent, dirty looking ulcer
at the site w/ heaped tissue growth. Unlike in the past, this ulcer seems to be expanding rather than
healing. What is the likely dx & how would you confirm it?
a. A dirty looking expanding ulcer w/ heat up tissue growth at the edges in a pt who has a long hx of
recurrent ulcers in a site of chronic irritation is classic description for squamous cell carcinoma
developing in a long standing site of chronic irritation (Marganil’s ulcer)
b. Biopsy of the edge of the ulcer is the diagnostic test.
c. Although it should not be hard, do not confuse this for diabetic foot ulcer, ischemic ulcer or venous stasis
ulcer
671. After a fall a 24 yo presents w/ a distal clavicular fx. How is this tx?
a. Figure 8 harness or splint for 4 - 6 weeks
672. A 60 yo woman has a soft tissue mass that has been growing steady for 6 month, it is firm & fixed
into the thigh. What do you suspect?
a. Soft tissue sarcoma, we don’t know whether it is a liposarcoma, rhabdomyosarcoma, or fibrosarcoma…
but we can tell this is a soft tissue tumor.
673. What is the first step in managing this pt?
a. MRI to look at the tumor
674. T/F the next step is fine needle aspirate to confirm dx
a. False! To dx a sarcoma the pathologist needs a large piece of tissue which will complicate further
resection therefore avoid any invasive procedures but rather refer to a specialist
675. What are the different types of gallstones & w/ what is each associated with?
Cholesterol stones Pigmented stones Mixed stones
Color Yellow to green Black more common in
gallbladder
Brown more common in
bile ducts
Associated with…  Obesity, DM, Black stones Majority of stones
hyperlipidemia  Alcoholic liver
 Pregnancy, OCP cirrhosis
 Chron’s dz or ileal  Hemolysis ex. SS,
resection thalassemia,
 Advanced age hereditary
 Cirrhosis spherocytosis,
 CF mechanical heart
valves….
Brown stones
 Biliary tract infections

676. A black 66 yo male complains of back pain & a pathologic break of his hip. Imaging
studies revealed punched-out lytic lesions in his bones. CBC & chemistries indicate Hct of 33%,
elevated creatinine & hypercalcemia. Look at his imaging & PBS & state the most likely dx.

a. This patient has multiple myeloma which is the most common primary tumor within the
bone in the elderly (>40) & blacks are affected 2x more than whites.
i. The lytic bone lesions are characteristic of MM resulting in hypercalcemia due to the release
of the breakdown product into the blood.
ii. Anemia from chronic dz (inflammation  IL6  inc Hepcidin) w/ Rouleux formation (coin
stacks) & hyper viscosity
iii. Renal insufficiency due to damage of renal tubular cells & glomerular cells from Ig accumulation
iv. The “fried egg” appearance of the plasma cells is indicative of MM.
v. Think CRAB for MM: hyperCalcemia, Renal insufficiency & Rouleux, Anemia, Bone lesions
677. Bonus: What diuretic can be used to decrease serum calcium?
a. Furosemide is a Loop diuretic that Leaves calcium inside the tubes used for CHF & HT but it’s also great
for increasing the secretion of calcium
678. What is multiple myeloma?
a. Multiple myeloma is a plasma dyscrasia where there is proliferation of malignant mature plasma cells
resulting in secretion of a monoclonal immunoglobulin protein. These malignant cells produce a shit ton
of IgG (55%) or IgA (25%). Think CRAB for MM: hyperCalcemia, Renal insufficiency, Anemia, Bone
lesions
679. What lab tests help confirm the dx of multiple myeloma?
a. 24 hour urine collection where you then do urine protein electrophoresis detecting the Bence Jones
protein which is just the light kappa or lambda chain
b. Serum protein electrophoresis trying to detect the M spike
680. 55 yo woman falls in shower & hurts her right shoulder. She presents w/ her arm held close to
her body but rotated outwards as if she is about to shake hands. What is the likely dx & how do you
confirm it?
a. Anterior dislocation of the shoulder confirm the diagnosis w/ 2 view radiographs
b. Anterior dislocation is by far the more common dislocation of the shoulder
681. What nerve is commonly injured in pts w/ anterior dislocation of the humerus?
a. Axillary nerve
682. How does a posterior shoulder dislocation present?
a. Arm held close to body w/ internal rotation usually seen in pts who have massive generalized
contraction of muscles such as epileptic seizures or electrocutions
683. How is the dx confirmed?
a. Radiographs of the shoulder including scapula-lateral or axillary view
684. 32 yo female presents after MVA. Vitals are normal, no pelvic fx present. Pt has gross hematuria
& cystogram is negative. What is going on & what is the next step?
a. Hematuria without pelvic fx & normal cystogram suggests kidney damage which is associated w/ rib
fractures.
b. Next step in management is a CT scan w/ contrast looking for renal injuries
685. T/F the majority of pts w/ renal injury do NOT require surgery
a. True!
686. The previous pt’s CT scan showed renal injury that did not require surgery. 6 wks after d/c the pt
develops SOB & flank pain. What happened?
a. Although rare, some pts post renal injury may develop AV fistulas which overload circulation to the
kidneys  Renal failure
687. How would you confirm the dx?
a. Arteriogram
688. What if the previous pt developed HTN instead of the AV fistulas?
a. Likely developed renal artery stenosis
689. A 30 yo woman presents w/ progressively worsening SOB (pulmonary congestion) & has a
pansystolic murmur best heard at the apex radiating to the axilla. Standing worsens the murmur
intensity. What valvular heart dz does she most likely have?
a. This is classic presentation of mitral regurgitation due to mitral valve prolapse as indicated by the
hemodynamic changes & the fact that its best heard at the apex radiating to the axilla. The reason she
has SOB is because the regurgitation  congestion in LA  pulmonary edema.
690. S3 is a sign of decompensated heart failure w/ _____ overload whereas S4 is a sign of _____ overload
often detected in LVH or aortic stenosis
a. S3 is a sign of decompensated heart failure w/ VOLUME overload (eccentric hypertrophy) whereas S4 is
a sign of PRESSURE overload often seen w/ LVH (concentric) or aortic stenosis
691. A 30 yo woman presents w/ several week h/o mild RUQ fullness. US shows a mass in the liver but
AFP are normal. The diagnosis of hepatocellular adenoma (=hepatic adenoma) is made. What could
have caused this & is this malignant or benign?
a. Hepatocellular adenomas are benign liver tumors most often seen in young women (15-40) who are
taking oral contraceptives. Usually regress after OCP are stopped.
692. What is the most common type of benign liver tumor?
a. Cavernous hemangiomas which are vascular tumors
693. How are liver masses diagnosed?
a. Ultrasound or CT scan w/ contrast
b. If you suspect a cavernous hemangioma avoid doing a bx since it may cause rupture of the tumor
694. A 56 yo man has bloody bowel movements. The blood coats the outside of the stool & has been
present on & off for several weeks. For the past 2 month he has been constipated & his stool has become
of narrow caliber. What is the next step in management?
a. This is most likely left colon cancer which presents w/ narrowing of stool, change of bowel habits &
bright red blood in stool. The first study in a pt w/ this presentation is a flexible sigmoidoscopy (NOT
colonoscopy since its likely left colon cancer)
695. What is the next step after a positive CRC sigmoidoscopy?
a. CT scan to assess metastases
696. What must be done before operation for resection of the cancer?
a. Must get a full colonoscopy to assess for other primary tumors.
b. Baseline CEA
697. When is surgery contraindicated in closure of a PDA in an adult?
a. If there is severe pulmonary HTN or a right-to-left shunt then do NOT correct PDA
698. A 15 yo girl is stung by a bee during a picnic. 20 minutes after she develops urticarial rash,
hypotension & vomiting. How should you manage this pt?
a. Epinephrine to control the anaphylaxis reaction
b. Take out stingers carefully
699. A 40 yo obese mother of 5 children presents w/ progressive jaundice she noticed 4 wks ago. She
has elevated alk phos & minimally elevated AST/ALT. She has elevated conjugated bilirubinemia, what is
the next step in management?
a. This woman is the classic 4 Fs for gallstones & the first step in managing a pt w/ obstructive jaundice is
an US.
700. What would be the next step if the US shows a gallbladder w/ a thick wall & full of stones, but no
stones visible in the biliary tract?
a. The next step would be cholecystectomy for sx pts
701. What is the significance of calcifications in pulmonary masses?
a. A calcified pulmonary mass is more likely benign
b. Non-calcified pulmonary masses are more likely to be malignant
702. 6 yo boy has progressive limping & knee pain on the right side. He bears less weight & for less
time on the right side. Pt denies any trauma to that side. What do you suspect?
a. Legg-Perthes disease which commonly occurs around 6 years of age where there is avascular necrosis of
the capital femoral epiphysis
b. Hip pathology may produce knee pain but knee pain may not produce hip pain therefore when you hear
knee pain think pathology in knee or hip
c. No tx needed since it will revascularize
703. A 34 you male c/o chronic back pain. He reports progressive pain & stiffness, specifically
morning stiffness & pain that is worse at rest & improves w/ activity. Pt denies any hx of trauma. What is
the likely dx?
a. Since the pt is young & he has chronic progressive pain & stiffness that improves w/ activity the
most likely dx is ankylosing spondylitis
704. What do you expect radiographs to show?
a. Bamboo spine
705. What is the tx?
a. NSAIDs & physical therapy
706. What Ag is associated w/ ankylosing spondylitis?
a. HLA B27 which is also associated w/ psoriatic polyarthritis, IBD & uveitis
707. What is a Colle’s fx & who tends to get it?
a. Colle’s fx is a dorsally displaced dorsally angulated fracture of the distal radius commonly seen in older
ostoporortic women who fall on an outstretched hand.
b. In children supracondylar fx result from falling on an outstretched hand
708. What else are you worried about in an adult that falls on an outstretched hand?
a. Fracture of the scaphoid bone in the hand or rotator cuff tear.
709. How would you dx a fx of the scaphoid (carpal navicular)?
a. Pts complain of wrist pain & PE shows localized tenderness to palpation over the anatomic snuff box.
b. Radiographs will be positive only 3 weeks later therefore tx based on PE & hx & repeat radiograph 3
wks later
710. A 22 yo man has a colonoscopy because of rectal bleeding. A villous adenoma is found in the
rectum & several adenomatous polyps are found in the sigmoid. On PE you notice pigmented macules
surrounding the pts mouth & buccal mucosa. How should you tx his colon polyps?
a. This is a common presentation of Peutz-Jeghers syndrome which is an AD inherited syndrome
characterized by benign hamartomatous polyps & hyperpigmented macules on the lips & oral mucosa.
Since these are benign there is no need to take the polyps out.
b. In FAP & Gardner’s (osteomas, desmoid tumors) syndrome around 100% of the polyps become malignant
therefore resect all of them.
711. T/F you must resect all villous adenomas
a. True since 50% become malignant. Also remove all adenomatous polyps
712. What should you do if the pt was diagnosed with….
1. Juvenile polyposis syndrome
2. Peutz-Jeghers dz
3. Inflammatory polyps
4. Hyperplastic polyps
ii. Answer
1. No need to resect any of these since none of these are malignant
713. A 42 yo woman becomes confused, lethargic, c/o severe HA, has a grand-mal seizure & finally
goes into a coma 12 hours after a total hysterectomy. The nurse accidently gave her 500cc/hr of D5W
instead of 100. What is the likely dx?
a. This is acute water intoxication which is further confounded by the fact that post-surgery pts secrete
excess ADH in response to the stress
714. What test would you order to confirm the dx?
a. Serum Na concentration likely around 120
715. What is the management?
a. Very careful administration of 3% saline & mannitol….avoid quick correction since this may cause central
pontine myelolysis syndrome
b. First line tx for asx pts w/ hyponatremia is free water restriction
716. A 63 yo male w/ a 60 pack year of smoking & clinical evidence of COPD is undergoing pre-op
evaluation for an elective repair of AAA. He currently smokes one pack a day. We know this pt is a high
risk for surgery due to ventilation. What will be the best test to quantify his degree of risk?
a. Blood gases where we are looking for pCO2 which will be higher than normal
b. PFT looking for FEV1 which is a good measure of the ability to ventilate
717. How is it possible to improve his risk prior to surgery?
a. Cessation of smoking for 8 weeks…must stop for at least 8 weeks because after one week of smoking these
pts develop bronchorrhea which makes them an even higher risk.
b. Intensive pulmonary therapy such as incentive spirometry & inspiration of air
718. What is the tx for a pt c/o wrist pain & tenderness over the snuff box w/ a negative radiograph?
a. Thumb spica cast for possible scaphoid fx which will not be evident on radiographs
719. What are you worried about in the previous pt if the radiograph shows a displaced & angulated
scaphoid?
a. If the scaphoid shows a displaced & angulated scaphoid there is a very high rate of nonunion of that bone
which requires open reduction & surgery
720. 3 days post-op for tx of a 30% body surface area burn a 75 kg pt develops UOP of 250 cc/hr. How
should you adjust his fluids?
a. UOP should be around 1cc/kg/hr therefore this pt has massive diuresis. Although in the first 2 days we
would want to reduce the infusion for this pt massive diuresis is normal on the third day post burns
because all the fluid that was trapped under the burns is not making its way back to the vasculature
721. A 63 yo male presents w/ abdominal pain, blue/black toes & complaints of decreased UOP. He
underwent arteriogram last week for possible renal artery stenosis & he has a strong h/o
atherosclerosis. His labs show Cr of 2.4 that was 0.7 last week & an elevated BUN. What is going on?
a. This pt has new ischemia in his toes & acute renal failure as suggested by the decreased UOP & elevated
BUN/Cr. This can be explained by cholesterol embolization syndrome which could have been caused by
radiographic intervention such as the arteriogram causing showers of cholesterol crystals originating
from atherosclerotic plaques.
722. What is the first step in a pt presenting w/ acute onset of respiratory distress due to suspected
tension pneumothorax?
a. Immediately decompress the pleural space on the side w/ hyper resonance & decreased breath sounds
using a large bore needle or chest tube. DO NOT obtain a CXR if tension pneumothorax is suspected
723. A 42 yo female c/o RUQ pain after meals. She states the pain is worse at night & it sometimes
radiates pointing to her right scapula. Pt reports that she has been itchy all over & reports dark colored
urine. What is the likely dx?
a. Choleolithiasis commonly seen in the 4 Fs, Fat Female Fertile Forties.
724. How is choleolithiasis diagnosed?
a. RUQ ultrasound has a high sensitivity & specificity (>95%) for stones > 2mm
725. How would you tx this pt?
a. No treatment is necessary if pt is asx but elective cholecystectomy is done for pts w/ recurrent bouts of
biliary colic
726. What are some other signs of biliary tract obstruction?
a. Elevated Alk phos
b. Elevated conjugated bilirubin
c. Jaundice
d. Pruritus
e. Clay-colored stools
f. Dark urine
727. Describe Kussmaul’s sign?
a. Kussmaul’s sign is when JVD fails to decrease during inspiration suggesting constrictive pericarditis.
During inspiration there should be more room for the ventricles to dilate therefore increase diastolic
filling which should help drop the venous pressure & JVD but in pts w/ constrictive pericarditis there is
diastolic dysfunction therefore no fall in JVD.
b. Do not confuse this w/ Kussmaul’s respiration which is rapid deep breathing seen in DKA in an attempt to
lower pCO2 & correct the metabolic acidosis
728. A pt has a swollen knee after he was hit during soccer. He reports pain when you angle his knee
at 30° & you abduct. What does this suggest?
a. If the pain is during abduction of the knee at this angle it suggests damage to the medial collateral
ligament & meniscus since the abduction results in valgus stress
729. What are the 4 pharmacological tx options for systolic CHF & state the MOA & how they affect
mortality
MOA Effect on mortality Comment
Loop diuretic Inhibits Na reabsorption in NONE Used for symptomatic
(Furosemide) the loop of Henle thereby relief & first line along w/
reducing blood interstitial ACE inhibitors
volume  ↓ in pulmonary
edema
ACE inhibitors Result in venous & arterial Decrease mortality First line along w/
dilation  ↓ pre & furosemide but if pts
afterload develop cough then use
ARBs
Beta blockers ↓ HR/BP/contractility & Decrease mortality, Added to moderate CHF
myocardial oxygen specifically in post-MI on top of ACEI & LD. May
demand CHF slow down progression of
CHF by slowing down
tissue remodeling
Digitalis (cardiac Inhibit Na/K ATPase NONE - Useful in pts w/ EF <
glycosides) thereby destroying the 30% (severe CHF) or
gradient that the Na/Ca severe atrial fibrillation
efflux pump  ↑ - Serum levels should be
intracellular Ca  ↑ checked periodically due
contractility (inotropic to toxicity
agent)

730. What about diastolic CHF?


a. There are very few therapeutic options available & pts are treated symptomatically
731. Which beta blocker (BB) has been shown to have the best survival in tx of CHF?
a. The COMET trial compared carvedilol & metoprolol & showed that carvedilol has a significant
improvement in survival.
732. What are some signs of digoxin toxicity?
a. GI: N/V & anorexia
b. Cardiac: ectopic (ventricular beats), AV block, AFib
c. CNS: visual disturbances, disorientation
733. What is the overall 5 year mortality in pts w/ CHF?
a. 50%
734. What is considered hypertensive emergency?
a. A systolic BP > 220 or diastolic > 120 in addition to end organ damage requires immediate treatment.
b. Hypertensive urgency is a BP > 220 or > 120 systolic withOUT any organ damage
735. While rummaging around her attic a lady is bitten by a black spider. The pt develops vomiting,
severe abdominal muscle cramps & vital signs reveal HTN. What is happening here?
a. Bit by the black widow spider, may be confused as acute abdomen it is so significant
736. How do you tx this pt?
a. IV calcium gluconate & anti-venom
737. What is the leading cause of liver transplant in adults?
a. Hep C but NASH is becoming the #1
738. What is a major SE of bariatric surgery?
a. Vitamin malnutrition
739. A 22 yo gang member presents w/ inflammation & sharp cut over his right metacarpal after a
fight. How do you manage this?
a. This is a bite mark since the man likely hit a person in the face. Human bite marks must be properly
cleaned & tx w/ Augmentin since human bites are the worst bites we can have due to oral flora. If it
affects the joint it may destroy it therefore take it to the OR & massive debridement
740. A cirrhotic pt is bleeding from a duodenal ulcer. Surgical intervention is being considered. Pt has
encephalopathy, albumin 2.5, PT is 22 sec, & bilirubin is 3.5. Should you operate?
a. Do not operate!!! This pt clearly has hepatocellular dysfunction which excludes this pt from surgery since
they have a mortality rate approaching 100%
741. Which of the following pts will be a candidate for surgery?
i. A cirrhotic pt w/ blood ammonia 160 needs an operation.
ii. A cirrhotic pt w/ albumin of 1.8
iii. A cirrhotic pt w/ bilirubin >4
b. Answer
i. None of the previous pts will be able to undergo surgery since anyone of those lab values
suggests hepatocellular dysfunction which is a major contraindication to surgery
742. What is the MOA of death associated w/ operating on cirrhotic pts?
a. Operation on pts w/ these parameters results in high output cardiovascular failure 2/2 AV fistulas
forming which shunt the blood to the venous system thereby preventing perfusion of tissues
743. Fill out the following table
Primary sclerosing cholangitis (PSC) Primary biliary cirrhosis (PBC)
Abnormality
Clinical features
Complications
Dx
Tx
Comments

Primary sclerosing cholangitis (PSC) Primary biliary cirrhosis (PBC)


Abnormality Chronic idiopathic progressive dz of intra Chronic & progressive autoimmune
and/or extra-hepatic bile ducts characterized by cholestatic liver dz characterized by
thickening of the bile duct walls & narrowing of destruction of INTRAhepatic bile ducts
the lumen w/ portal inflammation & scarring
 May or may not progress to cirrhosis &
liver failure
Clinical  Insidious onset  Insidious onset
features  Jaundice due to direct hyperbilirubinemia  Pruritus early in course of dz
 Pruritus  Jaundice late in course of dz
 Fatigue/malaise/weight loss  RUQ discomfort
 Osteoporosis
Complications  Cholangiocarcinoma  Portal HTN
 Recurrent bouts of cholangitis (infxn)  End-stage liver failure
 2/2 biliary cirrhosis  Cirrhosis
 Portal HTN  Xanthomata
 Liver failure  Xanthelasmata
Dx ERCP & PTC showing bead-like structuring &  Cholestatic LFTs (↑↑ Alk P)
dilatations of the intra/extrahepatic ducts  Positive antimitochondrial Ab
 Labs show cholestatic LFTs (hallmark)
 Elevated cholesterol
 Elevated IgM
 Liver bx to confirm dx
Tx  ERCP or PTC to place stent & relieve  Ursodeoxychoolic acid (hydrophilic bile
obstruction acid) slows progression of dz
 Cholestyramine to relieve pruritus due to  Cholestyramine to relieve pruritus due
liver dz to liver dz
 Liver transplant is only cure  Liver transplant is only cure
Comments  Major risk factor for the development of  Autoimmune liver dz commonly
cholangiocarcinoma associated w/ UC (not CD)
 Most common in middle aged women

744. A 47 yo presents w/ RUQ pain that radiates to his scapula. The pt reports that he has a h/o
recurrent episodes of biliary colic. RUQ US was negative w/o any presence of gallstones therefore you
suspect biliary dyskinesia. Describe what biliary dyskinesia is & how you would confirm the dx.
a. Biliary dyskinesia refers to a motor dysfunction of the sphincter of Oddi which leads to recurrent
episodes of biliary colic without any evidence of gallstones on diagnostic studies.
b. Diagnosis is made by HIDA scan. Once the gallbladder is filled w/ labeled radionuclide, give
cholecystokinin (CCK) which will stimulate contraction & emptying of the gallbladder. If the ejection
fraction is low then dyskinesia is likely.
745. A 62 yo man describes epigastric & substernal pain that he cannot characterize well. At times his
description sounds like GERD at times it does not. US of the gallbladder, EKG & cardiac enzymes are
negative. How would you decide whether or not the pain is due to reflux?
a. Esophageal pH monitoring is best….although we can show reflux w/ X-ray & contrast we cannot tie the
pain w/ the reflux. w/ pH monitoring the pt can monitor when he has pain & associate that w/ the pH.
b. Reflux is usually a vague pain that is hard to characterized
746. What test would you order if the pt reported significant crushing retrosternal pain every time he
swallows?
a. This is more likely to be nut cracker syndrome which is uncoordinated contractions of the esophagus
which is best diagnosed w/ monometry which is a test used to assess esophageal motility
747. A 66 yo male has an upper GI endoscopy for scheduled checkup on chronic GERD. 6 hours after
the procedure he presents complaining of constant retrosternal pain that began shortly after he came
home. He looks very ill, fever of 104, diaphoretic & a respiratory rate of 30. There is a hint of
subcutaneous emphysema at the base of the neck. What is the next step?
a. This is most likely instrumental esophageal perforation of the esophagus. The next step is gastrograffin
swallow which is a water soluble medium that can be used as a contrast in the GI tract that is far safer
than barium if it extravagates
b. Avoid barium contrast since it will compound the injury by irritation the pleural or peritoneal cavity
through perforation
c. Avoid endoscopy since it may worsen the perforation
748. What is the next step if the gastrograffin study is negative?
a. Continue w/ a barium study since the gastrograffin is a poor contrast that gives bad images. If you have
to use the barium then be ready for quick surgery if it does confirm the perforation
749. Shortly after administration of succinylcholine a pt develops a fever of 104. What do you suspect?
a. Malignant hyperthermia which results in massive release of Ca in muscle tissue  rapid increase of
temperature
750. How is this treated?
a. IV dantrolene which is also a muscle relaxant which stops muscle activity which is responsible for the
hyperthermia
751. What must you monitor in these pts?
a. Monitor serum myoglobin & K since it may cause renal damage
752. What other agent most frequently causes malignant hyperthermia?
a. Halothane
753. What metabolic disturbance do you expect to find in a person who has thrown up for several days
(Cl, K, pH)
a. Hypocholermic, hypokalemic metabolic alkalosis since you lose H/Cl/K found in the gastric juice
754. What about in pts who have had persistent diarrhea?
a. Hyperchloremic, hypokalemic metabolic acidosis since you lose a lot of HCO3
755. How do you correct the metabolic alkalosis from vomiting?
a. By inhibiting H+ secretion in exchange for Na in the kidneys we can correct the metabolic alkalosis
therefore we prevent secretion of H which is done by providing a generous intake of KCl which will
enable the kidney to use the K & Cl in exchange for Na reabsorption thereby preferentially sparing the H
b. No more than 10 mEq of K an hour
756. We know that malnourished pts (sig % of weight loss & low albumin) are EXTREMLY high risk for
operation. How can we improve their risk?
a. Intense feeding over 7-10 days significantly lowers the risk associated w/ operation; preferably enteral
route vs. parenteral-.
757. T/F DKA is an absolute contraindication for surgery
a. True! Must correct acidosis & blood sugars before surgery
758. What do you suspect if a pt develops a fever of 101-102F 45 mins after surgery?
a. Sepsis, get blood cx & begin empiric tx
759. What is the MCC of nosocomial bacteremia?
a. Coagulase negative staph
760. A 34 yo female lawyer presents w/ c/o tenderness at the bottom of her foot. She reports that it
has been there for over a month & it hurts when she walks. On PE you find a tender spot at the bottom of
her foot between the third & fourth toe. What is the likely dx?
a. Morton neuroma which is inflammation of the common digital nerve at the third interspace between the
third & fourth toes. The neuroma is tender & caused by use of pointed, high heeled or cowboy shoes that
cause the toes to be bunched together.
761. What is the MCC of acute appendicitis?
a. 60% of the cases are due to hyperplasia of lymphoid tissue  obstruction of the lumen  stasis 
bacterial growth  inflammation  distention  compromise blood supply  ischemia  infarction or
necrosis if untreated  appendiceal perforation  peritonitis
b. 35% of cases are due to fecalith (fecal stones) causing obstruction
762. What are the clinical features of acute appendicitis?
a. Abdominal pain that later localized to McBurney’s point in the RLQ
b. Low grade fever
c. Leukocytosis
d. Anorexia
e. N/V
f. Rebound tenderness, guarding, ↓ bowel sounds
763. What are some PE maneuvers that would further support the dx of acute appendicitis?
a. Peritoneal signs such as guarding, rebound tenderness, diminished bowel sounds
b. Psoas sign is RLQ pain when the right thigh is extended
c. Obturator sign is pain is RLQ when right thigh is flexed & internally rotated while supine
764. When is the peak incidence of acute appendicitis?
a. In the teens to mid-20s; prognosis is far worse in infants & elderly since there is a higher rate of perforation
765. How is acute appendicitis diagnosed?
a. Acute appendicitis is a clinical dx which is supported by PE findings, Lab findings (mild leukocytosis) &
imaging studies such as US or CT scan
b. If suspicion is high & risk for perforation is as well then you can skip imaging & take the pt straight to the
OR.
766. A 64 yo female was BIBA to the ER after falling in the nursing home. When you come to examine
her you see that her left leg appears shortened & externally rotated. What are you worried about & how
would you confirm it?
a. Hip fx which will be dx using radiograph
767. What will be the tx if the radiograph shows displacement of the femoral head?
a. The likelihood that we will be able to salvage the femoral head is low therefore the treatment is to
remove the femoral head & place a metal prosthesis instead
768. What would be the tx if the radiograph shows an intertrochanteric femoral fx?
a. Open reduction & pinning of the bones
769. What are you worried about in pts w/ intertrochanteric femoral fx?
a. These pts have a very high likelihood of developing DVT therefore you must provide post-operative
anticoagulation
770. What do you expect in a pt who presents w/ hip pain & a shortened leg that is INTERNALLY
rotated after hitting his knee on the dashboard post-MVA?
a. This is an example of a posterior dislocation of the femur
771. How is this treated?
a. This is an orthopedic emergency since the blood supply to the femoral head runs posteriorly therefore if
this is not treated immediately there may be avascular necrosis so reduce this as soon as possible
772. What is a boxer’s fx?
a. Fx of the 4th & 5th metacarpal of the hand that did the punching
773. Pt presents w/ painful ulceration on arm. He was cleaning the attic yesterday & he thinks he
might have been bitten by a bug. How do you manage this?
a. This is an example of a brown recluse bite which you manage by skin debridement, tetanus prophylaxis
& possible skin graft
774. PE of a newborn reveals uneven gluteal skin folds but the Orlatani sign is negative. What do you
suspect & how would you confirm it?
a. Developmental dysplasia of the hip that is confirmed by ultrasound if the PE is not conclusive.
b. Do not use radiography since the bones have not solidified yet
775. How is DDH treated?
a. Place the child in a harness that keeps the femoral head inside the hip joint w/ abduction splinting
776. An obese 64 yo male c/o sudden onset of pain in his left big toe. On PE you see that he has
swelling, redness & exquisite tenderness in that toe but nowhere else. What is the likely dx?
a. Gout due to high serum uric acid which results in uric acid crystals depositing in the joint
777. What is the tx for this man?
a. For acute attacks indomethacin & colchicine (microtubule polymerization by binding tubulin &
preventing PMN migration)
b. For chronic tx use allopurinol (Xanthine oxidase inhibitor) & probenicid (↑ uric acid excretion by
inhibiting reabsorption)
778. A 72 yo man has lost 40 lb over 2 month period. The pt has a h/o anorexia & a vague epigastric
discomfort for 3 weeks. LFTs are normal. What is the next step in management?
a. This is obvious malignancy, therefore the next step in endoscopy & bx. Unlike suspected esophageal
malignancy, which requires a barium swallow first to create a map, the stomach has a large lumen
making it safer for endoscopy.
b. if pt also had jaundice & distended thin gallbladder it might be pancreatic cancer r/o w/ CT
779. A 66 yo male presents w/ jaundice that has been worsening over the past several weeks. He has a
total bili of 22 w/ 16 direct, AST/ALT are WNL, Alk phos is 6 x normal limit. He has lost 10 lbs in the past
two months but otherwise asx. US shows a dilated intrahepatic & a very distended thin walled empty
gallbladder. What is the dx?
a. We know this is obstructive jaundice & the thin distended empty gallbladder suggests malignancy
causing obstruction.
780. What is the next step in management of this pt?
a. CT scan is the next step since it will help rule out cancer of the head of the pancreas
781. What can you rule out if the CT scan comes back negative & what is the next step?
a. The CT scan will really only detect a moderately sized tumor of the head of the pancreas therefore if the
CT is negative then perform ERCP to detect presence of cholangiocarcinoma in the CBD or a duodenal
tumor blocking the ampulla of Vater.
782. Fill out the following table
Acute cholecystitis Biliary colic
Pain secondary to….
Pain persists for…..
Location of pain
PE signs
Dx made using…

Acute cholecystitis Biliary colic


Pain secondary to…. Obstruction of the cystic duct (not Contraction of the gallbladder
infection)  gallbladder wall against the obstructed cystic
inflammation duct…colicky pain that is worse
w/ fatty meals
Pain persists for….. Several days of Lasts only a few hours & is worse
persistent/constant pain w/ fatty meals
Location of pain RUQ or epigastrium pain that RUQ
radiates to the right shoulder or
scapula
PE signs  RUQ tenderness & rebound  Unlike cholecystitis which has
tenderness leukocytosis & fever biliary
 Murphy’s sign is colic is NOT an inflammatory
pathognomonic (inspiratory process therefore none of these
arrest during deep palpation of sx
RUQ)
 Hypoactive bowel sounds
 Low grade fever
 Leukocytosis
Dx made using… RUQ US showing a thickened RUQ US
gallbladder wall, pericholecystic
fluid, distended gall bladder or
stone
** chronic cholecystitis may develop w/ recurrent bouts of acute cholecystitis

783. What test is used for dx of acute cholecystitis when ultrasound results are inconclusive
a. Hepatoiminodiacetic acid (HIDA) radionuclide scan
784. Explain how the HIDA results are interpreted
a. A positive result is when 4 hours after injection of the radionuclide the gallbladder is NOT visualized
since the gallbladder is plugged up. If the gallbladder is NOT visualized then the diagnosis of acute
cholecystitis or cystic duct obstruction is confirmed; makes sense since bile needs to exit the liver through
the hepatic duct & fill up the gallbladder retrograde & if it is obstructed it won’t be able to fill it
b. In the absence of dz the gallbladder is visualized within 1 hour of the injection of the radioactive tracer
785. In what group of pts do we see acalculous cholecystitis?
a. Acute acalculous cholecystitis is usually idiopathic & seen in pts w/ severe underlying illness therefore
the dx is difficult since these pts have so many other medical problems so clinical features are less
apparent.
786. What is the tx for acalculous cholecystitis?
a. Emergent cholecystectomy is the tx of choice but for pts who are too ill perform percutaneous drainage
of the gallbladder w/ cholecystectomy
787. Fill out the following table comparing choleolithiasis to choledocholithiasis

Choleolithiasis Choledocholithiasis
Abnormality
Clinical features
Complications
Diagnosis
Treatment

Choleolithiasis Choledocholithiasis
Abnormality Stone in the gallbladder stone in the common bile duct
Clinical features Asx or biliary colic  Asx
 RUQ/epigastric pain
 Jaundice
Complications  Cholecystitis  Cholangitis
 Choleodocholithiasis  Obstructive jaundice
 Gallstone ileus  Pancreatitis (↑ lipase!!!)
 Malignancy  Biliary cirrhosis
Diagnosis RUQ ultrasound ERCP is the test of choice
Ultrasound/CT are NOT sensitive
Treatment No tx in most cases but elective Removal of stone via ERCP &
cholecystectomy within 72 hrs if sphincterotomy
severe or recurrent

788. A pt w/ known congenital heart defect is going to the dentist. What do you do?
a. Prophylactic amoxicillin for any pt w/ known heart defects
789. Name 2 drugs that may cause interstitial lung dz
a. Amiodarone (anti-arrhythmic)
b. Bleomycin which is an anti-cancer drug
c. Nitrofurantoin
d. Phenytoin
790. What PFT findings are suggestive of ILD?
a. Restrictive pattern where there is a ↓ in both FEV1 & FVC (more ↓) w/ a normal or increased FEV1/FVC
ratio. Diffusing capacity (DLCO) is decreased.
791. For what 2 ILD would you expect to see an abnormal UA?
a. Goodpastures dz which is IgG anti BM in the lungs & glomeruli
b. Wegener’s granulomatosis which is a classic triad of pneumonitis, sinusitis & glomerulonephritis due to
necrotizing granulomatous vasculitis
792. What type of hypersensitivity rxn is GP dz?
a. T2HS rxn where ab form against glomerular & alveolar BM resulting in hemorrhagic pneumonitis &
glomerulonephritis
793. How is Wegener’s granulomatosis dx?
a. Tissue bx showing necrotizing granulomas along w/ positive c-ANCA
794. Describe syndrome X
a. These patients have exetrional angina w/ evidence of myocardial ischemia on exercise testing BUT they
have a normal coronary arteriogram which does NOT show any evidence of disease. This is weird since
coronary arteriorgram is the definitive test for dx of CAD
795. Describe the mechanism behind the hyperpigmentation & edema in pts w/ chronic venous
insufficiency (CVI)
a. Normally the venous system in the legs is composed of superficial, deep & perforating veins. All have
valves which prevent retrograde flow & the perforating veins are responsible for moving blood form the
superficial veins to the deep veins but not vice versa. In CVI pts have:
i. Destruction of the venous valves in the deep venous system resulting in valvular incompetence &
accumulation of fluid in the legs due to gravity
ii. Destruction of the valves in the perforating veins resulting in movement of the blood from the
deep to the superficial (normally the other way)
iii. The result is ambulatory venous HTN which results in interstitial fluid accumulation  edema
& extravasation of plasma proteins & RBCs into the subcutaneous tissues resulting in brawny
induration & pigmentation of skin
796. What is the end result of CVI if untreated?
a. Eventually leads to reduced capillary blood flow & hypoxia of the tissues where ulcers form & tissue
death occurs.
797. Where do the venous ulcers most commonly occur?
a. Above the medial malleolus overlying the incompetent perforator veins
798. Describe the treatment management for CVI & venous ulcers in these pts?
a. Leg elevation & avoiding long periods of standing
b. Heavy weight elastic stockings
c. For ulcers use wet-to-dry saline dressing & Unna venous boot (external compressing stocking)
799. What is the tx for ulcers refractory to the previous tx
a. Split-thickness skin grafts
800. What are the 4 classic findings of DVT?
a. Lower extremity pain & swelling that is worse w/ walking & better w/ elevation
b. Homan’s sign which is pain in calf on dorsiflexion
c. Palpable cord
d. Fever
e. All of these signs have very low sensitivity & specificity since 50% of pts w/ these findings do NOT have a
DVT & 50% of pts w/ a DVT do not have these findings
801. What is used for dx of a DVT?
a. Ultrasound is the initial test for DVT
b. Venography is the most accurate test but is invasive & not really used
c. D-Dimer testing is a great screening tool since it is very sensitive but not very specific. If pt does NOT
have an elevated D-dimer then he does NOT have a DVT but if it is elevated it does not rule it
in…..therefore it’s a rule out test (Sensitive/SNOUT)
802. What would you do in a pt w/ classic findings of a DVT, elevated D-dimer but a negative
ultrasound?
a. Repeat ultrasound every 2 to 3 days for up to 2 weeks & if positive then begin anticoagulation
803. Describe the anticoagulation treatment for DVT
a. Heparin bolus & constant infusion to maintain the PTT at 1.5 times the aPTT
b. Once aPTT is therapeutic start warfarin (INR 2 -3) for 3 to 6 months while continuing heparin for 48
hours once INR is therapeutic range
c. Thrombolytic therapy speeds it up
804. What are the criteria for acute respiratory failure?
a. Hypoxemia which is characterized by PaO2 < 60mmHg
b. Hypercapnea PaCO2 > 50 mmHg which is due to hypoventilation
805. What are the two subtypes of respiratory failure?
a. Hypoxemic respiratory failure which is characterized by a low PaO2 w/ a low or normal PaCO2. The
SaO2 is <90% despite FiO2 > 0.6. Hypoxemic respiratory failure is due to dz process of the lung itself
b. Hypercarbic (ventilatory) respiratory failure which is failure of the alveoli to ventilate therefore CO2
retention
806. How will you determine the underlying mechanism of hypoxemia in a pt w/ respiratory distress?
a. PaCO2 levels will be elevated if the pt has V/Q mismatch or shunting (pneumonia, atelectasis) present
b. A-a gradient will be normal if it is due to hypoventilation or elevated due to mismatch
c. Response to supplemental O2 differentiates the two
807. What is the difference in how ventilation & oxygenation are monitored?
a. Ventilation is monitored by PaCO2 where we ↓ PaCO2 by either ↑ RR or TV
b. Oxygenation is monitored by O2 saturation & PaO2
c. Ventilation & oxygenation are unrelated!! A pt may be SaO2 100% by w/ very high PaCO2 & in ventilatory
failure
808. A healthy 23 yo male presents to the ER w/ a swollen dusky foot. He admits that he stepped on a
rusty nail 3 days ago. On PE the pt looks toxic you can feel gas in the tissue of his right foot. What is the
likely dx?
a. Gas gangrene likely due to Clostrdium perfringens
b. Gas gangrene does NOT require immunosuppression, it can happen to anyone
809. What is the next step in management for the pt?
a. You can collect fluid from the tissue & then get a gram stain which will help confirm the etiology (C per is
Gram (+))
b. Clostrdium perfringens is susceptible to penicillin therefore the first thing is large volume of IV
penicillin is used along w/ surgical debridement of the dead tissue & a hyperbaric chamber that will
help tissue regenerate.
810. A 63 yo presents to the ER w/ a 4 hr h/o constant, diffuse & severe abdominal pain. He has had
one episode of non-bloody vomiting since the pain started. KUB shows free air under the diaphragm.
What does this suggest?
a. Subdiaphragmatic free (intraperitoneal) air on abdominal x-ray suggests perforation of a hollow
abdominal viscus & acute abdomen.
811. What is the next step?
a. Requires emergent laparotomy
812. A 44 yo man has unequivocal signs of multiple liver metastases but no primary tumor has been
identified by multiple diagnostic studies of the abdomen & chest. The only abnormality of PE is a missing
toe that was removed at the age of 18 for a black mass growing. What is the likely dx?
a. This is an example of melanoma which may present w/ metastases 20 years down the road or
immediately after the dx. Melanoma does not follow the normal rules of metastases & it may present in
crazy random places such as the cardiac ventricles or wall of duodenum
813. Who is at highest risk for necrotizing enterocolitis?
a. Premature infants + problem feeding = NEC
814. How would you manage a preemie w/ suspected NEC?
a. Stop feeds, place IV fluids, order serial abdominal films & initiate systemic abx
b. If no perforation is present then bowel rest is the treatment w/ nasogastric decompression. If free air is
identified on radiograph or if the infant worsens then call surgery consult
815. When is surgery indicated for NEC?
a. Only when there is evidence of a perf such as pneumoperitoneum
816. A 45 yo male c/o feeling tired & sleepy all the time. He says his job seems more tiring to him & it
is difficult for him to get up in the morning. He also describes recent abdominal pain that is constant &
gnawing interfering w/ his sleep. His appetite is poor & he has lost 13 lb over the last month. PE is
significant for tenderness & fullness in the epigastrium. What is the likely dx?
a. Pancreatic cancer which presents insidiously w/ a combination of constant visceral epigastric pain
radiating to the back, jaundice due to extrahepatic biliary obstruction (head of pancreas) &
anorexia/weight loss.
i. Pancreatic cancer is more common in AA & rare before age 40 w/ peak incidence >65 yo.
b. Duodenal ulcers cause periodic epigastric pain relieved by meals
817. What is the most common site of pancreatic cancer?
a. The pancreatic head which would cause obstruction jaundice, US shows a thin distended gallbladder w/o
gallstones, CT helps confirm dx
818. What is the most clearly established risk factor for development of pancreatic cancer?
a. Smoking cigarettes
819. A 65 yo man has had 3 large bowel movements that he describes as made up entirely of dark red
blood. The last one was 20 mins ago. He is diaphoretic, pale w/ a HR of 110 bpm & a BP of 90/70. What
is the next step in managing this pt?
a. First thing is to resuscitate fluids & stabilize the pt
820. What is the next step after the pt is stabilized?
a. After the pt is stabilized place a nasogastric tube on suction looking for blood which if positive will be f/u
w/ endoscopy.
821. The NG tube brings up clear green non-bloody fluid, what does this tell us?
a. We know this pt has bleeding as we speak since he just had a bloody bowel movement. The fact that the
NG tube was negative for blood we can exclude a GI bleed from the stomach all the way up from the nose.
Since the fluid is green it means we sampled bile which suggests which helps us further rule out a
duodenal bleed there by confirming that this is indeed a lower GI bleed
b. if the fluid was white clear fluid (not green) then we would be able to r/o a bleed from the stomach all the
way up but not a duodenal bleed therefore the green fluid helps us further narrow on the location of the
bleed.
822. What is the next step in the previous pt?
a. Inspect the anus for possible hemorrhoids
b. If the pt is actively bleeding, which he is, we should NOT perform a colonoscopy since all we will see is
blood which will obscure the diagnostic field.
823. The pt’s recto-anal exam is negative, what is the next step?
a. Perform an emergency arteriogram which will provide the diagnosis if the pt still has significant bleeding
unit of blood every 4 hours ( >2 cc blood loss per minute)
i. Must have significant bleeding since otherwise an arteriogram will not show anything
b. If bleeding less than 2 cc per minute then wait for bleeding to stop & perform a colonoscopy later
c. Some physicians use the radionuclide tagged RBC transfusion which may show the site of bleeding
d. If pt does not have significant bleeding then just wait for a while & perform a colonoscopy later on
824. On the 5th post op day after a laparotomy it is noticed that large amounts of salmon colored clear
fluid is soaked in the wound dressings. How is this managed?
a. This is classic wound dehiscence where the wound is still held together but the lower layers failed to
come together. The fluid is peritoneal fluid stained w/ blood from the surgery. The management is
careful protection of the wound, keep the patient in bed & prevent too much movement or use of
abdominal binders. At a later date the pt will need to be re-operated on to prevent complications such as
hernias
b. The operation is not an emergency so you can wait some time
825. A 54 yo man has colicky abdominal pain & protracted vomiting for several days. He has
developed abdominal distention & has not had a bowel movement or passed any gas in 5 days. He has
high pitched loud bowel sounds that coincide w/ the abdominal pain. KUB shows air fluid levels. 5 yrs
ago he had exploratory laparotomy for a GSW. What is the next step of management?
a. This is classic presentation of mechanical obstruction likely secondary to the adhesions from the
previous surgery. These adhesions have been there all along for 5 years it’s just that the feces has
avoided falling into the trap of the adhesions. Now the feces has been trapped & therefore there is a
good chance that the feces can get out of it such as it has been able to for the past 5 years.
b. Typically we begin the management w/ NG suction, NPO & IV fluids giving the bowel time to evacuate
itself. While we are waiting we must monitor for strangulation which will result necrosis which will
have systemic sx.
826. What type of respiratory failure is seen in pts w/ ILD?
a. Pts w/ ILD have diffusion impairment therefore they have hypoxemia w/ hypocapnea since the CO2 is
still able to diffuse out.
827. How does acidosis due to hypercapnea affect CNS blood flow?
a. Acidosis due to hypercapnea results in vasodilation of cerebral vessels  ↑ blood flow  ↑ ICP 
papilledema, HA, AMS & eventually coma
828. Compare primary vs secondary stones in choledocholithiasis.
a. Primary stones originate in the CBD & tend to be pigmented stones (due to ↑ bilirubin)
b. Secondary stones originate in the gallbladder & pass into the CBD & tend to be cholesterol or mixed
stones. These account for 95% of the cases.
829. Which is more alarming to the physician a pt w/ biliary colic due to choleolithiasis or biliary colic
due to choledocholithiasis & why?
a. Onset of sx in choledocholithiasis is more alarming since it can signal the development of life-threatening
complications such as cholangitis & acute pancreatitis since it involves the common bile duct whereas
choleolithiasis does not.
830. What is used for dx of choledocholithiasis?
a. ERCP is the gold standard for diagnosis & treatment since ultrasound/CT are not sensitive for this
(50% sensitivity)
831. A 25 yo male suffering a MVA presents to the ER. Radiographs show a closed fx of the femoral
shaft, what is the appropriate tx?
a. As a rule tx for femoral shaft fx is intramedullary rod fixation, must properly anticoagulate these pts to
prevent DVT.
832. A 64 yo male presents w/ one wk h/o left arm pain & weakness. On ROS you find out he has lost
15 lb the past 2 month & he c/o SOB & diplopia. On PE you notice that he has left eye ptosis & miosis. Pt
has a 48 pack year h/o smoking. What is going on?
a. This is presentation of Pancoast tumor which is an apical lung tumor, typically squamous cell non-
small cell carcinoma, that causes compression of C8-T2 of the brachial plexus causing arm pain &
weakness & it is associated w/ Horner’s syndrome
833. What is a Monteggia fx?
a. Monteggia fx is a diaphyseal fracture of the proximal ulna w/ anterior dislocation of the radial head. This
is seen in people who raise their arm to protect themselves from a night stick or something…
834. A 53 yo male presents w/ sudden onset of severe abdominal pain. On PE the belly is soft, non-
tender & is quite normal. Pt has a previous h/o CAD & a MI. What do you suspect?
a. Severe abdominal pain w/ disproportionate physical findings is suggestive of acute mesenteric ischemia
835. What lab marker may help support the dx of acute mesenteric ischemia early on?
a. Elevated lactate levels & acidosis
836. What is used for definitive dx?
a. Mesenteric angiography is definitive diagnostic test & is used if the onset is recent
b. Thumb printing on barium enema due to thickened edematous mucosal folds also helps in dx
837. How is acute mesenteric ischemia treated?
a. Direct intra-arterial infusion of papaverine (vasodilator) into the SMA during arteriography is the
therapy of choice
b. Heparin anticoagulation may be used if it is of venous origin
c. If signs of peritonitis develop, surgery is indicated
838. What artery is most commonly affected in pts w/ acute mesenteric ischemia?
a. The SMA which supplies blood to the midgut (jejunum  middle 1/3 of transverse colon).
839. What is the MCC of acute mesenteric ischemia & name 3 other causes?
a. Arterial embolism accounts for 50% of cases almost all of cardiac origin; sudden onset of sx
b. Arterial thrombosis accounts for 25% & it is seen in pts w/ long standing atherosclerosis; usually
insidious onset
c. Nonoclucusive mesenteric ischemia is seen in pts w/ splanchnic vasoconstriction, commonly in very sick
elderly
d. Venous thrombosis < 10% of cases; sx present for several days of weeks w/ gradual worsening
840. How does chronic mesenteric ischemia present?
a. Abdominal angina, typically after a meal where there is increased demand for splanchnic blood flow
841. 64 yo woman w/ 2 week of jaundice has elevated conjugated hyperbilirubinemia, AST/ALT are
WNL there is sig elevated Alk phos. US demonstrates dilation of the intra/extrahepatic bile ducts & a
very distended, thin walled gallbladder. Pt has positive occult blood testing. What is the next step?
a. Obstructive jaundice w/ a thin distended gallbladder suggests malignancy. Normally the next step would
be a CT scan to r/o tumor of the head of the pancreas but since there is positive occult blood testing it
suggests a tumor of the duodenum that is also obstructing the ampulla of Vater therefore the next step
would be endoscopy.
842. A 48 yo woman w/ severe migratory necrolytic dermatitis for several years, unresponsive to tx is
referred to you by her dermatologist. Her skin has mild dermatitis & she is dx w/ DM after a oral glucose
challenge test showed blood glucose levels of 148. What is the likely dx?
a. Glucagonoma which is classic cause of migratory necrolytic dermatitis. The DM is mild & therefore the
biggest problem is the skin problem & these pts present to dermatologists rather than endocrinologist.
843. Define cholangitis
a. Inflammation of the bile ducts
b. Choledocholithiasis accounts for up to 60% of the cases
844. What is the classic presentation of pts w/ cholangitis
a. Charcot’s triad of RUQ pain + fever + jaundice which in reality is only present in 70% of cases
b. Reynold’s pentad which is Charcot’s triad plus septic shock & AMS
845. How is the diagnosis of cholangitis made?
a. RUQ US which is great for detecting gallstones & biliary tract dilatation but not sensitive in detecting
CBD stones
b. Lab studies showing hyperbilirubinemia, leukocytosis & mild elevation of serum transaminases
c. ERCP is the definitive test but should not be done during acute illness, but rather wait for it to resolve
(~48 hrs) & then proceed w/ PTC or ERCP.
846. How are pts w/ cholangitis managed?
a. IV fluids
b. Blood cultures
c. IV abx after blood cultures are obtained
d. Decompress CBD when pt is stable (24 – 48 hrs later)
847. What is the most serious & dreaded complication of acute cholangitis?
a. Hepatic abscess which has a very high mortality rate
848. What is the MCC of adult RDS?
a. Sepsis or septic shock is the highest RF for developing ARDS which is due to ↑ capillary permeability  ↑
A-a gradient & ineffective gas exchange that is NOT responsive to supplemental 100% oxygen
849. What is the most useful parameter in differentiating ARDS from cardiogenic pulmonary edema
which also presents w/ a similar CXR & hypoxemia that is refractory to oxygen supplementation?
a. Pulmonary capillary wedge pressure (PCWC) is placed in the pulmonary artery thereby indirectly
measuring LA pressure. If PCWP is < 18 mmHg then it r/o CHF
850. What is the most common type of carcinoma in the gallbladder?
a. Adenocarcinomas are most common & typically occur in the elderly & associated w/ gallstones in most
cases.
851. What is the prognosis of carcinoma of the gallbladder?
a. Poor prognosis, >90% of pts die within 1 year since it goes undetected until it is very advance
852. What is Dupuytrens contracture?
a. Increased tension of the palmar fascia resulting in inability to lay the hand flat on a desk. The ring finger
& the index finger are most commonly affected & it is much more common in men of Scandinavian
descent.
853. What is the tx?
a. Surgical relief of the tension in the fascia
854. How do you check for Airway w/ regards to the ABC?
a. Check for presence of gag or cough reflex or if the pt is talking w/ no problem it r/o pathology
855. What must you do after placing an endotracheal tube?
a. Listen for bilateral breath sounds & check a post-intubation CXR
856. Give at least 4 parameters to consider before extubating or weaning a pt from a ventilator
a. PaO2 >75
b. PaCO2 < 45
c. Adequate respiratory effort
d. Intact cough when suctioning secretions
e. O2 sat of >90% w/ PEEP of 5 cm H2O or less
f. Tidal volume > 5ml/kg
g. RR < 30 bpm
h. Negative inspiratory pressure < -20 cm H2O
857. Define pneumoconiosis
a. Accumulation of dust in the lung resulting in a tissue reaction to its presence
858. Give at least 3 causes of pneumoconiosis
a. Beryllium
b. Silicosis
c. Asbestos
d. Coal
859. Fill out the following table
Asbestosis Silicosis
Characterized by
Predilection for
How long till it develops after
exposure
Associated with
Classic CXR finding
Treatment
Sources

Asbestosis Silicosis
Characterized by Diffuse interstitial fibrosis of lungs Localized & nodular
peribronchial fibrosis
Predilection for Lower lobes Upper lobes
How long till it develops after >15 years after PROLONGED > 15 years after chronic exposure
exposure exposure
Associated with  ↑ risk for bronchogenic  ↑ risk for TB
carcinoma (smoking is
synergistic)
 Malignant mesothelioma
Classic CXR finding  Pleural plaques  Egg shell calcifications
Treatment  None Removal of exposure to silica
Sources  Ship building  Mining
 Car mechanic  Stone cutting
 Insulation worker  Glass manufacturing

860. Palpable non-tender gallbladder in a 67 yo AA male w/ vague dull abdominal pain is suggestive
of….
a. Pancreatic cancer which is most commonly seen in AA males around age 65. The palpable non-tender
gallbladder is the Corvoisier’s sign which is classic for cancer of the pancreas
861. How is pancreatic cancer diagnosed
a. CT scan is the preferred test for dx & assessment of dz spread but ERCP is the most sensitive & it may
help differentiate between other tumors which have a more favorable prognosis.
862. What tumor markers are associated w/ pancreatic cancer?
a. CA 19-9 is more sensitive & specific
b. CEA
863. A 44 yo female presents to your office for a regular checkup. She is found to be hypertensive
whereas last year her BP was normal. Lab studies show a serum Na of 151, HCO3 of 28, K of 2.1. She
denies taking any medication. What is the likely dx & the first step in confirming this dx?
a. Hyperaldosteronemia which presents as HTN, w/ a touch of hypernatremia & touch of metabolic
alkalosis w/ HYPOkalemia
b. The first step should be to measure serum aldosterone & renin since to confirm the dx you need to
have high aldosterone w/ low renin. Renin is the major stimulator of aldosterone secretion (NOT ACTH)
c. For this to be actually due to hyperaldosteronemia the pt must NOT be on diuretics since pts w/ HTN on
diuretics will present w/ HTN & hypokalemia
864. What are the 2 causes of hyperaldosteronemia?
a. Either adrenal hyperplasia or aldosterone adenoma
865. How do you differentiate between the two?
a. Adrenal hyperplasia still responds to physiologic stimuli such as ↑ aldosterone secretion when standing
up as opposed to laying down. Therefore if there is a difference in serum [aldosterone] w/ position then
it suggests adrenal hyperplasia whereas the adenomas are independent of position.
b. Repeat measurements of serum aldosterone & renin when laying down & standing up (higher levels in
hyperplasia)
866. Elderly pt hospitalized w/ pneumonia c/o progressively worsening RUQ. Labs show slightly
elevated AST/ALT & very high Alk phos. US shows thickening & inflammation of the gallbladder. Pt has
new onset fever of 105. What is likely going on & the next step in management?
a. This is an example of acute ascending cholangitis which present in elderly ill pts w/ high fevers & Alk
phos the next step is ERCP since this is an emergency. Also place pt on NPO, IV fluids/ abx & get blood
cultures
867. A newborn has not passed stool & has green vomit so a KUB was done & it shows the following.
What is the dx?
a. This picture is the classic “double bubble” which is due to duodenal atresia, malrotation or annular
pancreas
868. What chromosomal abnormality commonly has this problem?
a. Down syndrome
869. What is another cause of intestinal obstruction seen in pt’s w/ Down syndrome?
a. Hirschsprung disease which is lack of caudal neural crest migration therefore lack of the enteric NS to
the distal colon resulting in coarctation of a distal segment of the colon w/ proximal migration.
870. How is Hirschsprung dz diagnosed?
a. An appropriate x ray as well as a full thickness rectal biopsy showing lack of innervation
871. What if the pt has a double bubble sign as well as a few loops of distal bowel that have gas &
bilious vomiting on the first day of life?
a. May also be due to malrotation or annular pancreas on top of duodenal atresia
872. Which of the previous three is the most worrisome?
a. The malrotation is most worrisome since there is a high likelihood that this bowel will strangulate &
become necrotic.
b. Every time you see double bubble sign do not forget to think about malrotation
873. How will you confirm the dx of malrotation in a newborn w/ bilious vomiting?
a. Barium enema or swallow
b. Barium enema is safer but not as diagnostic whereas swallow is riskier since the baby is vomiting & may
aspirate.
874. T/F if you accidently find gallstones in an asx pts you do not remove them
a. True! Very low rate of conversion of asx to sx gallstones
875. A 54 yo man has colicky abdominal pain & protracted vomiting for several days. He has
developed abdominal distention & has not had a bowel movement or passed any gas in 5 days. He has
high pitched loud bowel sounds that coincide w/ the abdominal pain. KUB shows air fluid levels. 5 yrs
ago he had exploratory laparotomy for a GSW. 6 hours after being hospitalized & placed on NG suction &
IV fluids, he developed fever, leukocytosis & abdominal/rebound tenderness. What is going on & what is
the next step of management?
a. This is an example of mechanical obstruction that progressed to bowel strangulation. The next step is
emergency laparotomy
876. Pt w/ hx of LLQ pain & rectal bleeding presents to ER w/ fever & urine stained w/ poop coming
out of his penis. What is going on?
a. Diverticulitis  colovesical fistula; confirm dx w/ cystoscopy
877. About 45 minutes after completion of cystoscopy a pt develops chills & a fever of 104F. What is
the likely dx & the management?
a. The development of a high fever this close to an invasive procedure is suggestive of bacteremia.
b. Management requires blood cultures & then empiric abx until susceptibilities come back.
878. 54 yo man has colicky abdominal pain & protracted vomiting for several days. He has developed
abdominal distention & has not had a bowel movement or passed any gas in 5 days. He has high pitched
loud bowel sounds that coincide w/ the abdominal pain. KUB shows air fluid levels & distended loops.
On PE a growing mass is noted & he explains that he used to be able to push it back at will but the past
couple of days has not been able to do so. Labs indicate that he is dehydrated w/ BUN/Cr ratio of 26 & Na
levels of 153. What is the next step of management?
a. This is an example of an incarcerated & potentially strangulated (peritoneal & systemic sx) hernia. The
next step in management is correction of dehydration & then surgery. Unlike adhesions that only
require surgery if there is suspected strangulation, all hernias that cause obstruction will be operated on
since once it is fixed, unlike adhesions, it will not recur.
b. The first step before surgery is correct his metabolic abnormality so in this case this man will likely need
around 4 liters of fluid (3 mEq of Na = 1 L)
879. A 63 yo male w/ h/o aortic graft surgery presents w/ c/o blood in his stool. What is the next step
in management?
a. Although rare, in a pt w/ a GI bleed & a h/o aortic graft surgery you have to r/o aortoenteric fistula
which is a rare but lethal cause of GI bleeding. Get an endoscopy during the small window of opportunity
to prevent death.
880. For each of the following pts state what test you should order first
1. Pt c/o bloody vomiting
2. Pt c/o bright red blood in stool
3. Pt c/o dark, tarry, foul-smelling stool
4. Pt was found to have positive occult blood in stool
ii. Answers
1. Upper GI endoscopy is initial test for hematemesis
2. For hematochezia first r/o upper GI w/ NG tube suction & anorectal causes such as
hemorrhoids & then get a colonoscopy if negative
3. For melena first get a NG suction if neg  upper endoscopy since melena suggests an
upper GI bleed but if negative get a colonoscopy
4. Colonoscopy is the initial test in most cases of occult blood since cancer is the main
concern, especially if pt is >50 yo
881. What does coffee grounds emesis suggest about the etiology of the bleed?
a. Upper GI bleeding as well as a slow rate of bleeding which allows enough time for vomitus to transform
882. What is the most common site of carcinoid tumors?
a. The appendix
883. What are carcinoid tumors?
a. Carcinoid tumors originate from neuroendocrine cells that secrete serotonin
884. What would be the presentation of carcinoid tumors in the test?
a. Carcinoid syndrome which is due to excess serotonin secretion resulting in cutaneous flushing, diarrhea,
sweating, wheezing, abdominal pain & heart valve dysfunction. The presence of carcinoid syndrome
suggests metastasis to the liver!!
b. In reality only 10% of pts w/ carcinoid tumors experience carcinoid syndrome
885. How are these diagnosed?
a. Urine 5-HIAA
886. What is the tx for carcinoid tumor?
a. Surgical resection
887. What vitamin deficiency may be seen in pts w/ carcinoid syndrome?
a. Niacin (Vit B3) deficiency which results in Pellegra (3 Ds = dementia, dermatitis & diarrhea) 2/2 too ↑ 5-
HT secretion which is made from tryptophan & niacin
888. Granulomatous vasculitis & eosinophilia in a pt w/ h/o asthma
a. Churg-Strauss syndrome (AKA eosinophilic granulomatosis w/ polyangiitis) typically presents w/
pulmonary infiltrates, rash & eosinophilia.
889. How is the dx of CS made?
a. Significant blood eosinophilia along w/ positive P-ANCA
890. A 32 yo carpenter accidently drives a small nail into the pulp of his index finger but he pays no
attention to the injury at the time. 2 days later he presents to the ER w/ pain in that finger, fever & all the
signs of an abscess in the pulp of the index finger. What is the name of this dx & how is it managed?
a. This is called Felon which is a surgical emergency since it requires emergency decompression
because the pulp of the finger has fascia trabecular made for closed spaces where the pus has nowhere to
go therefore it can compress the vasculature  necrosis of the tissue.
891. A 63 yo male w/ Alzheimer’s dz who lives in a nursing home is operated on for a fx of the femoral
head. On the fifth post-op day it is noted that his abdomen is grossly distended, tense but not tender. He
has occasional bowel sounds. A KUB shows dilation of the colon & distal small bowel. Barium injection
& serial radiographs shows that the contrast travels throughout the whole GI tract. What is going on?
a. This is an example of Ogilvie’s syndrome which is where there are the signs, sx & radiographic evidence
of large bowel obstruction but no mechanical obstruction is found. May be due to recent surgery/trauma
& medications such as narcotics or anticholinergics & seen in elderly pts who have other severe
comorbidities. Dx is made when mechanical obstruction of colon is excluded.
b. Tx includes decompression immediately
c. The fact that the barium traveled all the way through helps rule out obstruction
d. The fact that the pt has occasional bowel sounds helps rule out paralytic ileus
892. What is the next step in managing this pt?
a. Colonoscopy which will help move fecal matter which will allow the gas to be released thereby
decompressing the tract
b. Furthermore colonoscopy will further help rule out mechanical obstruction.
c. Can place a long rectal tube which will help continue exit of the gas.
893. Whenever there is colonic distention w/ a colon diameter exceeding ___ then there is risk of
impending rupture leading to peritonitis
a. >10 cm; tx is to decompress immediately
894. How can you decompress the colon?
a. Gentle enemas or nasogastric suction
b. Colonoscopic decompression
c. Surgical decompression is last resort
895. A 43 yo male has difficulty extending his wrist after breaking his arm when he fell down the
stairs. Radiographs show an oblique fx of the diaphysis of the humerus. What is going on?
a. Injury to the radial nerve which innervates the extensors travels in a groove on the posterior side of the
humerus.
896. What is the next step in management?
a. First step is to attempt to reduce the fx & see if it helps w/ the nerve.
897. A different pt comes in w/ the same oblique fracture of the humerus & normal extensor function.
After reducing the fx & placing the pt in a cast the pt later develops radial nerve paralysis. What does
this mean & how is it managed?
a. This suggests that the nerve is trapped between the bones in the fracture. Management requires OPEN
surgery to free the nerve & correctly reduce the fx
898. What are the two classic types of COPD & explain each?
a. Emphysema & chronic bronchitis
i. Chronic bronchitis is a clinical dx of chronic cough productive of sputum for at least 3 months per
year for at least 2 consecutive years. There is enlargement of the mucous glands & smooth
muscle hyperplasia resulting in excess mucus & obstruction
ii. Emphysema is a pathologic dx where there is permanent enlargement of air spaces distal to
terminal bronchioles due to destruction of alveolar walls
iii. The two often coexist & pure emphysema or pure chronic bronchitis is rare
899. What is the biggest RF for COPD?
a. Tobacco smoke is seen in 90% of cases
900. What is the cause of COPD in pts who do not smoke?
a. Alpha-1 antitrypsin deficiency resulting in unopposed action of trypsin which digests the alveolar walls
901. Compare emphysema seen in smokers to alpha-1 anti-trypsin deficiency

Smoking emphysema Alpha-1 antitrypsin emphysema


Type Centrolobular Panlobular
Destruction of… Limited to respiratory bronchioles Destruction in both proximal & distal
which are the proximal acini acini
Area of lung most affected Upper lung zones where smoke Lung bases when the fluid
accumulates accumulates

902. What are some signs of COPD?


a. Prolonged forced expiratory time; can be as long as 6 seconds of full exhalation of vital capacity which
is evidence of obstruction
b. Hyperresonance on percussion
c. End expiratory wheezing suggesting intrathoracic obstruction
d. Tachypnea, tachycardia
e. Cyanosis
f. Dyspnea
g. Use of accessory muscles
903. What test is used as a SCREENING test for pulmonary obstruction?
a. Peak expiratory flow (PEF) rate using a peak flow meter. If < 350L/min it suggests obstruction
904. What is used as the definitive diagnostic test for COPD?
a. PFT
905. What PFT finding is suggestive of COPD?
a. ↓ FEV1 & ↓ FEV1/FVC ratio < 80% which suggests obstruction; also spooning of the curve & ↑ TLC &
residual volume indicating air trapping
i. FEV1/FVC ratio < 50% suggests severe dz
ii. FEV1 is the amount of air that can be forced out in one second therefore the lower the FEV1 the
harder it is to breath
b. Restrictive PFT is ↓ of both FEV1 & FVC w/ a normal ratio
906. Fill out the following table
Pink puffer Blue bloater
Predominance of emphysema or
CB?
Fat vs skinny
PE findings
Respiratory rate
Pt in distress & using accessory
muscles?

Pink puffer Blue bloater


Predominance of emphysema or Emphysema Chronic bronchitis
CB?
Fat vs skinny Skinny due to ↑ E expenditure of Fat & blue due to ↑ pCO2
breathing
PE findings Barrel chest (↑ AP diameter) Chronic cough & sputum
When sitting pts tend to lean production
forward Signs of cor pulmonale
Respiratory rate Tachypnea w/ prolonged Respiratory rate is normal
expiration through pursed lips
Pt in distress & using accessory Yes No
muscles?

907. A man suffers a MVA in Houston. When EMS arrives he is unconscious w/ a BP of 84/53. What is
the first thing they should do?
a. This man is in shock & the most likely reason is hemorrhagic shock due to trauma. The first thing is to
address the ABCs & then find the bleed & stop it in the OR setting. Once you get him to the OR give two
16 gauge IV in the arm w/ NS or Ringer’s solution. Do not pick the answer that says stabilize the pt at the
scene if there is a trauma center in the area.
908. What vasopressors may be given if the pt remains hypotensive despite fluids?
a. Dopamine is the first line vasopressor & NE is used in more severe/resistant cases
b. Dobutamine may be also used since it will increase CO; used specifically for cardiogenic shock post MI
909. What surgical intervention is indicated for pts w/ cardiogenic shock since it increases CO &
myocardial oxygenation?
a. Intra-aortic balloon pump (IABP) which is a pump that is inserted into the descending aorta & it inflates
& deflates opposite to the heart therefore immediately before cardiac systole the balloon deflates there
by reducing afterload & during diastole it inflates therefore increasing afterload & pushing blood into
coronary circulation therefore the net effect is enhanced myocardial oxygenation & increased CO.
910. How are COPD pts clinically monitored?
a. Serial FEV1 measurements (highest predictive value)
b. Pulse ox
c. Exercise tolerance
911. What are used as first line txs in COPD?
a. Smoking cessation!!!
b. Inhaled beta 2 agonists and/or anticholinergics are first line
c. Oxygen therapy is used in pts w/ COPD & chronic hypoxemia since oxygen is a potent pulmonary
vasodilator & therefore helps in tx of the pulmonary HTN that is seen in these pts
d. Inhaled corticosteroids have NOT been shown to help in long term tx unlike in asthma there whey are first
line therapy. Systemic corticosteroids are used in acute COPD exacerbation in pts who need to be
hospitalized
912. What is the MCC of death in the ICU?
a. Septic shock
913. What metabolic finding is seen in all pts w/ shock?
a. Lactic acidosis due to under perfusion of tissues
914. What renal/CNS/cardiac finding is seen in all pts in shock?
a. Oliguria/anuria
b. Tachycardia
c. AMS
915. What is the most useful parameter used for monitoring the effectiveness of treatment for shock?
a. Urine output
916. Shock in a pt w/ fever & warm skin/flushing is suggestive of….
a. Septic shock or vasomotor shock where there is severe vasodilation
b. Hypovolemic shock is associated w/ peripheral vasoconstriction & cool skin
917. What other type of shock presents w/ warm, well-perfused skin?
a. Neurogenic shock which results from failure of the SNS to maintain adequate vascular tone
918. What is a common syndrome that presents before septic shock
a. Systemic Inflammatory Response Syndrome (SIRS)  sepsis  septic shock
b. SIRS is characterized by 2 or more of:
i. Fever > 38 or hypothermia < 36
ii. Tachycardia (>90)
iii. Hyperventilation or PaCO2 < 32
iv. Leukocytosis > 12,000
c. Sepsis is SIRS plus positive blood cultures
919. A 67 yo male presents w/ fever, LLQ abdominal pain. Pt states that for the past several years he
has had episodes of blood in his stool but they would stop spontaneously. CBC shows leukocytosis. What
do you suspect?
a. Diverticulitis which is an infection of a diverticulosis most commonly found in the LLQ sigmoid area.
b. Diverticulosis is a common cause of lower GI bleeds whereas it is uncommon in diverticulitis
920. How would you confirm the dx?
a. CT scan w/ oral & IV contrast
b. Barium enema & colonoscopy are contraindicated in acute diverticulitis due to risk of perforation!!
Although these are used for dx of diverticulosis.
921. What is the tx for diverticulitis?
a. IV abx w/ bowel rest (NPO)
b. For recurrent episodes surgery is recommended once acute inflammation has resolved
922. A football player gets hit straight in his right leg & he suffers posterior dislocation of that knee.
What is the importance of this clinical scenario?
a. Right behind the knee joint is the popliteal artery which is unique since it has very little collateral
circulation therefore if it is not quickly recognized, by checking for intact distal pulses it can result in loss
of blood supply to the lower leg & need for amputation.
b. The answer to these vignettes is checking distal pulses & quickly reducing the dislocation
923. What is a porcelain gallbladder referring to?
a. Porcelain gallbladder refers to a gallbladder w/ intramural calcifications
924. What is the tx?
a. Prophylactic cholecystectomy is recommended since 50% of pts w/ porcelain gallbladder eventually
develop cancer of the gallbladder, most commonly adenocarcinoma which has >90% mortality rate
925. Abdominal pain, hypotension & abdominal pulsatile mass?
a. Ruptured AAA
926. Abdominal pain “out of proportion” to abdominal exam?
a. Mesenteric ischemia
927. Elderly woman w/ pain down inner aspect of thigh?
a. Obturator hernia (Howship-Romberg sign)
928. Arm pain & syncope w/ arm movement?
a. Subclavian steal syndrome
929. Increase in creatinine on ACE inhibitors
a. RAS
930. Child w/ MIDLINE neck mass…
a. Thyroglossal duct cyst
931. Child w/ LATERAL neck mass…
a. Branchial cleft cyst
932. Crush injury & dark urine?
a. Myoglobinuria
933. Emesis, chest pain radiating to back & mediastinal air?
a. Boerhaave’s syndrome which is rupture of the esophagus 2/2 to vomiting resulting in leaking of fluid
into the mediastinum  mediastinitis & chest pain
934. What is the MCC of kidney transplant?
a. diabetes
935. If a choice of left or right donor kidney is available which is preferred?
a. The left since the renal vein is longer
936. Why are native kidneys kept in pts who receive new ones?
a. Increased morbidity if removed
937. What is an indication for removal of native kidneys?
a. Uncontrollable HTN
b. Ongoing renal sepsis
938. What is the red flag that indicates kidney rejection?
a. ↑ in Creatinine
939. What is used for monitoring liver transplant rejection?
a. Serum bilirubin
b. LFTs
940. A 43 yo male c/o worsening severe pain over his lower leg 3 hours after placement of a cast for an
ankle fracture. What is the next step in management?
a. REMOVE THE CAST due to possible compartment syndrome…it does not matter if the person is a whiner
because compartment syndrome is a surgical emergency that requires a fasciotomy
941. Does HLA matching matter in organ transplantation?
a. With immunosuppression the effect is largely obscured BUT for HLA-B & HLA-DR matching is the most
important, especially when it comes to renal allografting.
b. HLA crossmatching is most important for kidney & pancreas transplants!!!
942. Lower GI bleed & a positive technetium pertechnetate scan….
a. Meckel’s diverticulum
943. Skin flushing, diarrhea & right-sided heart failure…
a. Carcinoid syndrome which is massive release of serotonin from neuroendocrine cell cancer, usually mets
in the liver where the primary tumor is in the appendix resulting in diarrhea, flushing & right side heart
valve malfunction
944. How is this diagnosed?
a. CT or US of appendix & elevated urine 5-HIAA
945. A newborn infant has respiratory distress w/ prominent inspiratory wheezing when supine but
not when sitting up. What is the likely dx?
a. Tracheomalacia, inspiratory wheezing suggests EXTRA-thoracic obstruction
946. What would be the first step in management if previous newborn also had difficulty swallowing?
a. If pt also had difficulty swallowing then it would suggest a vascular ring 2/2 failure of second fetal aorta
to degenerate
b. Barium swallow which will show an extrinsic compression
c. Bronchoscopy which will r/o tracheomalacia & show segmental tracheal compression
947. T/F in all L to R cardiac shunts there is increased pulmonary markings on CXR
a. True since we are increasing pulmonary flow which may eventually result in pulmonary HTN
948. During a school PE a 12 yo girl is found to have a murmur. She has a hx of frequent colds & URI,
when you examine her you hear a fixed split second heart sound (S2). What is the dx?
a. ASD which is classically heard as a fixed split S2 & is seen in asx young pts, does not become sx until 30s
949. What is the next step in working up this pt?
a. The first step in all congenital heart problems is an echocardiogram.
950. A 3 mo boy is hospitalized for FTT. He has a loud pansystolic murmur best heard at the left
sternal border. What is the likely dx?
a. VSD which is heard as pansystolic murmur & will be sx (FTT) unlike ASD
951. What is the next step in management?
a. Echocardiogram!!! Always the answer as the first step for evaluation of non-emergent cardiac cases
b. This will in fact require surgical repair since a significant L to R shunt will cause irreversible pulmonary
HTN & damage
952. What information, if given on exam, w/ regards to a VSD will change the course of tx?
a. If the echocardiogram shows a very low VSD that is in muscle will close by itself therefore the answer
will not be surgical repair but rather watchful waiting.
b. Don’t forget these pts will require prophylactic abx (amoxicillin)
953. Institutionalized (nursing home), abdominal pain, vomiting & distention, w/ proximal colonic
dilation
a. Sigmoid volvulus
b. Dx & tx w/ sigmoidoscopy
954. Infant w/ projectile vomiting…
a. Pyloric stenosis…oral erythromycin increases the risk
955. Newborn w/ failure to pass meconium in first 24 hrs
a. Hirschprung’s or CF
956. Infant w/ bilious vomiting
a. Malrotation or atresia
957. newborn w/ abdominal defect & umbilical cord on sac
a. omphalocele which is associated w/ other congenital malformations
b. gastroschisis is paraumbilical & is NOT associated w/ other abnormalities
958. Pulmonary capillary wedge pressure of 16, CXR w/ bilateral pulmonary infiltrates & PaO2:FO2
ratio < 200….
a. ARDS
959. Increased peak airway pressure, low urine output & urinary bladder pressure >25mmHg 2 days
after ex lap
a. Abdominal compartment syndrome which will expand & cause↑ intra-thoracic pressure as well as
compression of kidneys & bladder
960. Newborn w/ inability to “pass a NGT”
a. esophageal atresia ; mucho salivation
961. Traumatic blinding in one eye followed by blindness in the contralateral eye 2 weeks later
a. sympathetic ophthalmia thought to be 2/2 to autoimmune rxn; occurs after trauma which exposes the
immune system to ag that are normally not seen by the immune system
962. traumatic head injury, conscious in ER followed by unconsciousness
a. epidural hematoma’s classic lucid interval
963. What are the two types of esophageal cancers?
a. Squamous cell carcinoma
b. Adenocarcinoma
964. What are the clinical features associated w/ esophageal cancer?
a. Dysphagia initially to solids & later to liquids
b. Weight loss secondary to anorexia & dysphagia
c. Odynophagia which is pain w/ swallowing
d. Hematemesis
e. Hoarseness if there is impingement of the recurrent laryngeal nerve
f. Chest pain
965. Fill out the following table comparing the previous 2
Squamous cell carcinoma of the Adenocarcinoma of the
esophagus esophagus
Which is more common
Location along esophagus
Risk factors
AA or whites?

Squamous cell carcinoma of the Adenocarcinoma of the


esophagus esophagus
Which is more common more common Less common
Location along esophagus Upper & mid thoracic Distal third of esophagus as GE
junction
Risk factors  Alcohol  GERD & Barrett’s esophagus
 Tobacco use Esophageal adenocarcinoma is a
 Diet complication 2/2 to chronic
 HPB GERD where there is columnar
 Plummer-Vinson syndrome metaplasia of the squamous
epithelium
AA or whites? More common in AA More common in Caucasian males
(5:1)

966. How are these diagnosed


a. Barium swallow helps evaluate the dysphagia but upper endoscopy w/ bx & brush cytology is required
for confirming the diagnosis
967. What imaging is used for staging of esophageal cancer?
a. Transesophageal ultrasound as well as a full metastatic workup including CT scan, CXR & bone scans
968. What is cardiac index?
a. Cardiac index is an expression of cardiac output as it relates to the pts body surface area
969. How should you decide the next step in correcting CO/CI in a post-operative pt who just had open
heart surgery w/ a CI of 1.7 (low)?
a. The two ways we can increase the CO is by either increasing the contractility w/ ionotropic agents or
increase preload w/ fluids. This is decided based on the end diastolic volume which if…
i. If EDV is high then it suggests that we should use ionotropic agents
ii. If EDV is low then correct w/ fluids
970. Cardiac catheterization shows a square root sign & equalization of pressure in all ventricles &
atria. This is suggestive of…
a. Chronic constrictive pericarditis
971. A 3 day old premature baby has trouble feeding & pulmonary congestion. PE shows bounding
peripheral pulses & a continuous machine like murmur. What is the dx?
a. This is classic PDA which is described as a continuous machine like murmur w/ bounding pulses. Plus
unlike many other cardiac anomalies PDA tend to present w/ complications early in life (1-3 days)
972. How are these pts managed?
a. First step is echocardiogram to evaluate whether or not you should close the duct
b. Second step will be either indomethacin to stimulate closure or surgical closure
c. Indomethacin tx takes several days therefore should not be used in pts w/ congestive failure therefore this
pt will require surgical intervention
973. What is Plummer-Vinson syndrome?
a. This is a classic presentation consisting of:
i. Upper esophageal webs causing dysphagia
ii. Iron deficiency anemia
iii. Koilonychias which are spoon-shaped fingernails
iv. Atrophic oral mucosa
974. Why are pts w/ PV syndrome monitored?
a. 10% of pts develop SCC (upper 2/3) esophageal cancer
975. What is the dx if a pt has distal esophageal webs?
a. Schatzki’s rings are distal esophageal webs (unlike PV syndrome which are proximal) that cause
dysphagia
976. What is a common cause of Schatzki’s rings?
a. Ingestion of alkali or acids during previous suicide attempts
977. Which is more dangerous ingestion of alkali or acid?
a. Alkali is more dangerous since it causes liquifactive necrosis which may be transmural whereas acid is
coagulative necrosis that is less likely to cause full thickness necrosis.
b. Amiodarone is second line treatment if ejection fraction is normal

You might also like